Download as pdf or txt
Download as pdf or txt
You are on page 1of 396

M-Ki

Dr.Sabreen O. Jebur
2015-2019/August 23

698.One of the following is not correct about stem cells and location:

A. Interfollicular epidermis

B. Palmoplantar epidermis

C. Sebaceos gland

D. Melanocytes(bulge)

E. Hair follicul (bulge)

698.PUVA differs from photodynamic therapy in that:?

A. Chromophore is an exogenous drug

B. Specific wavelengths of light are used to activate photosensitizer within tissue

C. Photosensitizer can be topical or systemic

D. Topical psoralen is active in its native form as administered

E. Broad range of potential dermatologic applications

698.Stratum corneum has feu cornified cells in:

A. 8 Week of EGA

B. 12 Week of EGA

C. 16 Week of EGA

D. 20 Week of EGA

E. 24 Week of EGA
1. One of the following is least likely to cause blue – Grey discoloration of the nails:

A. AZT

B. Wilson’s disease

C. Argyria

D. Minocycline

E. Amiodarone

160. The strateum corneum is composed of Few layers of cornified cells by:

A. 24 weeks of estimated gestational age

B. 20 weeks of estimated gestational age

C. 16 weeks of estimated gestational age

D. 12 weeks of estimated gestational age

E. 8 weeks of estimated gestational age

698.What first initial test should be performed in an infant showing cutis congenital with an open
fontanelle:

A. Total body CT scan

B. Blood glucose level

C. Cranial ultrasound

D. Spinal angiography

E. Cranial MRA
E

698.One of the following is correct in Atopic Dermatitis Inheritance:

A. The inherited individuals are heterozygous for the gene.

B. 25% chance to passing it to their children.

C. Both parents are carriers and healthy.


D. Consanguinity increase the risk.

E. None of the above.


E

698.Tumor suppressor gene located in the mitochondria is:

A. Myc

B. Fos

C. Rb

D. P53

E. Bcl-2
E

1. In which of the following sites the apocrine sweat gland is not present:

A. forehead

B. External auditory canal.

C. Eyelids.

D. Axillae.

E. Anogenital region
A

159. Major skin Angiogenesis factor is:

A. Placental growth factor

B. Platelet derived growth factor

C. Angiopoiten-2

D. Thrombospondin- 1

E. Keratinocyte growth factor

698.The most important function of postoperative wound dressing:

A. Provides a barrier to infection


B. Decreases tension on the wound

C. Creates a moist environment

D. Prevents suture dehiscence

E. Immobilizes the wound


E

698.The chemical mediators that plays a role in cross-linking of wound healing?

A. Kinin

B. Serotonin

C. Histamine

D. Prostaglandins

E. Complement
B

698.Electrocoagulation :

A. A moderately damped current is applied in a monoterminal manner

B. The current is of lower amperage and higher voltage than that utilized for
Electrocodessication

C. Because this type of current penetrates less deeply, it has the potential for less tissue
destruction

D. Electrocoagulation is not effective for ttt of superficial telangiectasias. Unwanted hair


(Electrocoepilation) and ingrown toenails.

E. It is the preferred modality for ttt small and uncomplicated primary BCC and SCC
E

1. Which of the following stains is most helpful for diagnosing Cryptococcus?

A. Warthin-starry

B. Fontana-Masson

C. Alcian blue

D. Giemsa

E. Von kossa
D

1. Which of the following stains is most helpful for diagnosing Cryptococcus?

A. Warthin-starry

B. Fontana-Masson

C. Alcian blue

D. Giemsa

E. Von kossa

157. The number of keratinocytes associated with melanocytes to form the Epidermal melanin
unit is:

V. 3

W. 13

X. 23

Y. 33

Z. 36

5
1042. One of the following is a commun cause of Erythroderma in adults:

A. Cutaneous T-cell lymphoma

B. PRP

C. Pemphigus Foliacius

D. Atopic Dermatitis

E. Congenital ichtyosis
4

1044. When considering the mechanism of action, which treatment used for onychomycosis
inhibits the epoxidation, an early step in the formation of ergosterol in the fungal cell:

A. Itraconazole

B. Ciclopirox olamine

C. Ketoconazole
D. Griseofulvine

E. Terbinafine
E

1043. One of the following would exacerbate perioral dermatitis:

A. Sunlight

B. Triamcinolone

C. Bactroban

D. Clotrimazole

E. Metoprolol
B

155. Paraphenylene diamine (PPD) allergen is present in :

A. Nail polish

B. Condom

C. Ear ring

D. Hair dyes

E. Dental fillings
D

156. One of the followings is not involved in adaptive immune response:

M. IL-2

N. IL-4.

O. IL-12

P. TNF-β

Q. IL-23.
4

1041. Which of the following retinoids has the LONGEST elimination half life:

A. Isotretinoin
B. Alitretinoin

C. Etretinate

D. Acitretin

E. Bexarotene
C

698.Complications of bacterial endocarditis include one of the following:

A. Renal abcesses

B. Splenic rupture

C. Immunocomplex glomerulonephritis

D. Heart failure

E. DIC
C

698.When examining a pathology slide with subepidermal blisters with eosinophils, Which of the
following is the most probable diagnosis:

A. PV

B. EBA

C. DH

D. Linear IgA Dermatosis

E. BP
E

153. A 45-year-old woman wants to remove her tattoo, which laser would be most appropriate
for the treatment of her red tattoo?

A. Long pulsed Nd: YAG laser

B. Q-switched 1064 nm Nd: YAG Laser

C. Q-switched 532 KTP

D. Q-switched 755 nm Laser

E. Q-switched Ruby laser


C
154. In One of the following conditions ANA is usually negative?

A. Sjogrens syndrome

B. Polyarteritis nodosa

C. Scleroderma

D. Infective endocarditis

E. Rheumatoid arthritis
D

1. The maximum dose (in mg for a 70kg man) of Mepivacaine for local anesthetics is:

A. 350

B. 300

C. 400

D. 200

E. 175
B

698.What is the initial mainstay therapy for severe Pemphigus Vulgaris:

A. Topical corticosteroid

B. Rituximab

C. Systemic corticosteroid

D. Minocycline

E. Azathioprine
C

698.The primary focus of pulmonary tuberculosis is characteristic by:

A. Cavitation

B. Occurs in the apex of the lung

C. Is mostly asymptomatic

D. Is complicated by military tuberculosis


E. Presents with haemoptasis
C

1036. Psoriasis has many features involving the nails. Which one is most frequently seen:

A. Discoloration

B. Nail plate dystrophy

C. Onycholysis

D. Pitting

E. Subungueal hyperkeratosis
D

152. One of the following Lasers is ablative ?

G. Erbium Yag

H. Q switched KTP 532

I. Alexandrite

J. IPL

K. Pulsed – dye
A

698.Which of the following vitamine deficiencies has no mucocutaneous findings:

A. B1

B. B5

C. B3

D. B6

E. B7
B

150. Which of the following suturing types for wound healing closure has highest tensile
strength?

m. staples
n. adhesive tapes

o. tissue glues

p. laser welding

q. photochemical welding
1

698.Regarding skin Prick test, One of the following is not true:

A. Cheap

B. Results are available in 45 mn

C. Can be used with ad hoc allergens (e.g. fresh, foods, drugs)

D. Cannot be done in patients receiving antihistamines and calcium channel blockers

E. Requires skilled staff for reproducible Results


A

698.A 65 y old man with atypical pneumonia caused by mycoplasma pneumonia will most likely
be treated with:

A. Benzyle pencillin

B. Methicillin

C. Erythromycin

D. Primaquine

E. Thiabendazole
C

149. Which of the following is not correct about laboratory diagnosis of neurosyphilis :

G. Reacher blood and CSF serologies which are almost always positive

H. CSF abnormalities

I. Specific anti treponemal antibodies in CSF is mandatory and proof the diagnosis

J. CSF-IgG index >0,7 is indicative of IgG in brain

K. Intrathecal T pallidum antibody index >100 is indicative of specific antibody in CNS


3
1032. One of the following histopathologic findings is characteristic for dg of BCCarcinoma:

A. Solid lobules of basaloid cells with jigsaw-puzzle appearance and thickened basement
membrane

B. Masses of basaloid cells with shadow keratinization and foreign body reaction

C. Solid lobules of basaloid cells composed of individual nodules of 2 cell type

D. Basaloid lesion with mature and immature hair follicle formation.

E. Solid lobules of basaloid cells with sebaceous differentiation


3

1031. The elastic fibres of the dermis are characteristic by:

A. A central core of elastin

B. A peripheral part of fibronectin

C. Elaunin fibres anchor the basement membrane

D. Oxytalin is abundant in reticular dermis

E. Mature fibres are restricted to middermis


A

1030. The following is true about the dermis:?

A. Undergoes an obvious sequence of differentiation

B. Its structure and organization are not predictable with regard to depth

C. Its matrix components undergo remodeling in normal skin

D. Remodeling of matrix is not affected by external factors

E. Pathological factors do not affect its organization


3

147. Which of the following Graft types has no contraindication:

A. Full-thickness skin graft

B. Split- thickness skin graft

C. Perichondrial cutaneous graft

D. Composite
E. Free cartilage

148. Which type of wound dressings is indicated for clean, non-infected, recalcitrant chronic
wounds:

AA. Alginates

BB. Hydrogels

CC. Films

DD. Collagens

EE. Foams

1029. The so called veil cells are located around:?

A. Sebaceous gland

B. Hair follicules

C. Lymphatics

D. Arterioles

E. Capillaries
D

698.One of the following is true about corpuscular skin receptors:

A. Are composed of a fibrous core and a capsule

B. They mediate pain

C. They form touch domes

D. Meissner”s corpuscule is situated in the deep dermis

E. Paccini corpuscule is located in the papillary dermis


A

698.One of the following proteoglycans is a component of the dermo-epidermal junction?

A. Decorin

B. Versican
C. Syndecan

D. Chondroitin sulphate

E. Perlecan
E

145.A 36-year-old female presents with a generalized, morbilliform rash associated with fever,
malaise, myalgia, and sore throat and was given antibiotics by her primary care physician. She
has since developed a morbilliform rash. What infection did she most likely suffer from and what
antibiotic did she most likely received?

f. EBV infection-ampicillin

g. CMV infection-clindamycin

h. EBV infection-clindamycin

i. CMV infection-amoxycillin

j. Streptococcal infection-amoxycillin
1

1. In HIV-positive patient, which of the following feature might not indicate seroconversion?

A. Fever

B. Cervical lymphadenopathy

C. Maculopapular skin rash over the extremities

D. Acute diarrhea

E. Myalgia

698.The elastic tissue of the dermis is associated in :

A. Horizontal bundles

B. Transverse bundles

C. Vertical bundles

D. Continuous network
E. Irregular loose fibres
D

698.One of the following is true about dermal collagens:

A. Constitue 60% of skin dry weight

B. Type I account for 50%

C. Type III surrounds structures with basal lamina

D. Type III fibrils are smaller in diameter than Type I

E. Fibrils are visible by light microscopy


D

698.Which part of the nail unit most closely resembles the epiderm histologically?

A. Lunula

B. Cuticle

C. Nail Bed

D. Nail Plate

E. Hyponychium
E

All of the following pertain to Odland bodies EXCEPT: 1. Contain squalene


2. Are found intracellularly in upper level keratinocytes
3. Discharge their contents into the extracellular space at the junction of the granular and
cornified layers
4. Establish a barrier to water loss
5. Mediate stratum corneum adhesion in conjunction with filaggrin.
A

143. The Janus kinase inhibitor is :

A. FDA – approved for treatment of psoriasis in adults

B. Tofacitinib and ruxolitinib are pregnancy category C

C. Tofacitinib should be avoided in patients with severe renal impairement

D. Caution is advised in patient at increased risk of gastrointestinal perforation

E. Melanoma skin cancers carcinomas have occurred in patients treated with Janus kinase
inhibitors
E

144. One of the following is false regarding the use of anti-TNFa biological agents?

A. Contraindication in patient with severe heart failure

B. Live attenuated virus should be avoided

C. PDD testing is apre-request in all patient

D. Neutralizing antibodies can reduce efficacy in patient on etanercept

E. ANA may become positive in patient receiving adalimumab


D

141. Complete replacement of toe nail requires:

A. 3 months

B. 6 months

C. 12 months

D. 15 months

E. 18 months
E

698.The most important proteoglycan in the dermis:

A. Versican

B. Perlecan

C. Biglycan

D. Keratocan

E. Syndecan
A

698.One of the following is not correct about nail plate :

A. Keratin account for 80% of dry weight of nail plate

B. Contain hard and soft Keratin

C. Less permeable to water than is the epidermis


D. It Contain lipids (primarily cholesterol) and minerals (Fe, Zn, Ca)

E. Calcium does not influence the hardness of the nail


C

142. Lamina densa does not have the following adhesion proteins:

A. Laminin 111

B. Laminin 311

C. Nidogen

D. Laminin 332

E. Laminin 511
A

698.Which of the following is false, about catagen:

A. It divided into 6 substages

B. Catagen last about 2 w in human regardless of the site and follicule type

C. Characterized by a nearly 50% reduction in the volume of the demal papilla

D. Melanocytes in the hair bulb also begin to down-regulate Melanogene

E. There is cessation of mitotic activity within the matrix cells of the bulb
A

1. Which of the following is microfibrillar collagen?

A. COL III

B. COL IV

C. COL V

D. COL VI

E. COL VII
C

698.Which of the following is a fibril-forming collagen??

A. COL.I

B. COL IV
C. COL V

D. COL VI

E. COL VII
D

698.Sweat ducts are not affected in:?

A. Lichen Planus

B. Ichtyosis

C. Psoriasis

D. Palmoplantar Pustulosis

E. Atopic Dermatitis
B

698.Intraepidermal blister at the granual layer is not seen in:

A. Friction blister

B. Pemphigus foliates

C. Subcorneal pustular dermatosis

D. Herpes Virus infection

E. SSSS
D

1. Which of the following is microfibrillar collagen?

A. COL III

B. COL IV

C. COL V

D. COL VI

E. COL VII
C

Which of the following occlusive ingredients is present in topical moisturizers:


A. Squalene

B. Ceramics

C. Cholesterol

D. Fatty acids

E. Glycerin
A

138. One of the following is not a site of expression of the POMC gene except:

A. Adrenals

B. Pituitary gland

C. Keratinocyte

D. Testis

E. Endothelium
A

1. The nomber of Melanocyte in the genital region:

A. 850/mm2

B. 900/mm2

C. 1000/mm2

D. 1300/mm2

E. 1500/mm2
E
1015. Regarding nail development , proximal nail fold is visible at:?

A. 8 weeks of EGA

B. 10 week of EGA

C. 13 week of EGA

D. 15 week of EGA

E. 18 week of EGA
C
Generalized pigmentation occurs in :
A. Xeroderma Pigmentosum

B. Tattooing

C. Iron deficiency

D. Addison’s disease

E. Cronn’s synd
D

698.One of the following statement regarding bullous Impetigo is true?

A. Impetigo is transmitted via fomites

B. Impetigo most commonly occurs in childs> 6 y of age

C. Peak incidence is in winter

D. Non-contact sports like running, increase the risk of transmition

E. Streptococcus pyogenes is more often the implicated bacterial species in Impetigo


A

698.Hepatitis B:

A. Has an incubation period of 15 – 45 days

B. Is spread by ingestion of contaminated seafood

C. Most patients recover completely

D. 50% become healthy carriers

E. Is a SSRNA virus.
D

698.Body louse is a vector for:

A. Endemic typhus

B. Epidemic typhus

C. Yellow fever

D. HIV

E. All of the above


B

Bone morphogenic proteins (BMPs) are express by cells:


A. Without fibroblast growth factors (FGFs)
B. With FGFs.
C. With activation of Wnt signaling.
D. Without activation of Wnt signaling.
E. With activation of molecular signals.
A

698.Tinea manuum :

A. Dermatophyte infections on the dorsal aspect of the hand

B. Dermatophyte infection of the palm and interdigital spaces have a clinical presentation similar
to Tinea corporis

C. The reason for the two different clinical pictures is thought to be related to the lack of
sebaceous glands on the palms

D. Interdigital type Tinea pedis is often present in patients with Tinea manuum

E. Tinea manuum is usually non-inflammatory and often bilateral


C

1. Desmoplakin disruption is not seen in:

A. Lethal acantholytic epidermolysis bullosa

B. Striate palmoplantar keratoderma

C. ED/skin fragility

D. PPK with left ventricular cardiomyopathy

E. Autosomal dominant arrhythmogenic right ventricular cardiomyopathy

Shakir c
Makki c
134. Which of the following stain is used for capsule of Cryptococcus neoformans?

A. PAS

B. Mucicarmine

C. Gomori methamine silver

D. Giemsa
E. Fite- faraco
B
One of the followings is not correct about ivermectin:
A. It is a macrocyclic lactone derived from ivermectin B1
B. It blocks the transmission of signals from interneurones to excitatory sensory neurons
C. It is effective against sarcoptes sabiei
D. It is used for treatment larva migrans
E. Side effects include fever, itching and headache
2

One of the following is not correct about pattern of immunoglobulins and their clinical
association:
A. Raised IgG, Ig A and IgM low= IgG myeloma waldenströms
B. Raised IgM, Ig G and Ig A Low= macroglobulinemia
C. Ig G, Ig A and Ig M low= Ig G or Ig E myeloma.
D. Very low Ig G, raised Ig A and Ig M = sjögren’s syndrome
E. Very high Ig M, normal Ig G or Ig A= primary biliary cirrhosis.
D

The biologic-false- positive results in non-treponemal tests include the followings EXCEPT:
A. Pregnancy
B. Drug abuse
C. Hepatic cirrhosis
D. Endemic treponematosis
E. Idiopathic, familial
D

Criteria for HIV seropositivity include the followings EXCEPT:


A. Repeatedly positive ELISA
B. Positive westen blot study
C. Reactivity against viral protein marker p24,gp41
D. Reactivity against viral protein marker gp 120/160
E. Reactivity against viral protein marker p51
E

Granulomas as clinical features suggesting an irritant or toxic etiology may be caused by the
followings EXCEPT:
A. Cotton fibers
B. Talc
C. Beryllium
D. Silica
E. Woods

698.The skin is thicker in palms and soles because of:

A. A thicker stratum cornum


B. The presence of special sense organs

C. Abundance of sweat glands

D. Presence of an additional cellular layer

E. Frequent friction with external factors


D

1. The epidermal basement membrane proteins derived from keratinocytes do not include:

A. BPAg2

B. α 6 β 4 integrin

C. CD151 tetraspan

D. Nidogen

E. Heparan sulfate proteoglycans


BM : Basal + fibroblasts
Basal —> BPag1,BPag2,integrins,laminin,tetraspan,plectin,HSPG,coll 4,7
Fibroblasts—> Nidogen,coll 4,7
4

136. Regarding BPAg2 , Which of the following is incorrect:

A. Occurs in 2 forms 180 kDa and 120 kDa extracellular domain

a. Proteolytic processing of BPAg2 is mediated by “sheddases” of (ADAM) family

B. Phosphorylation of BPAg2 by ecto-casein kinase 2 enhanse its Proteolysis by TACE

C. The cytoplasmic domain of BPAg2 associates with BPAg1, integrin subunit β 4 and plectin 5

D. The first extracellular segment of BPAg2 (NC 16) bind integrin subunit α 6
3
Inhibit

1008. A 42 y old housekeeper has a chronic hand dermatitis and is complaining of severe
pruritus. One of the following is not class I corticosteroid???

A. Halobetasol o.o5% cream

B. Fluocinonide o.1% cream

C. Clobetasol o.o5% lotion

D. Diflorasone diacetate o.o5% oint


E. Mometasone o.1% cream
E👍

698.A 5 y old boy, followed for molluscum contagiosum, presents with an accidental 1 degree
burn. His only medication is cimetidine. He received a prescription for silver sulfadiazine cream.
One of the following is true?

A. The use of silver sulfadiazine cream is contraindicated due to the child’s age.

B. The use of silver sulfadiazine cream is contraindicated in patients with molluscum


contagiosum.

C. when silver sulfadiazine cream is used in conjunction with cimetidine, there is increased risk
of leukopenia

D. silver sulfadiazine should only be used for 3d degree burns.

E. None of the above are true.


C

698.One of the following is a Third generation antihistamine:

A. Hydroxyzine

B. Loratidine

C. Levocertizine

D. Triproline

E. Ciproheptadine
C👍

1. Which of the following is RNA – double stranded ?

A. Echovirus

B. Coxackie A

C. Hepatitis A

D. Rotavirus

E. Respiratory syncythial virus’s


D

132. Which of the following acts on CD20:


A. Basiliximab

B. Ofatumumab

C. Ranibizumab

D. Denosumab

E. Palivizumab
B

Epstein- barr virus:

A. Aproximately 75% of adolescents and young adults with primary EBV infection develop
infectious mononucleousis

B. EBV is commonly associated with B-cell lymphoma(50%

C. The virus enters cells by specifically binding to cell surface complement receptors CR4

D. The humoral immune response, although it generates antibodies to a variety of EBV


antigens, appear to be protective

E. Splenomegaly is found in > 50% of patients and can be accompanied by hepatomegaly


E

698.Which of the following topical antimicrobials is used in treatment of decubitus ulcer:

A. Azelaic acid

B. Erythromycin

C. Benzol peroxide

D. Clindamycine

E. Metronidazole
C

698.Higher risk for toxicity from glucocorticoid therapy does not occur in Which of the following ?

A. Female patients

B. Patients with rheumatoid arthritis

C. Patients with dermatomysitis

D. Patients with hyperalbuminemia


E. Patients who smoke

698.One of the following is not true about PCR:

A. PCR is simple and rapid.

B. It is low sensitive in amplifying low amounts of DNA

C. The total number of PCR products after n cycles

D. PCR can be used to label DNA with radioactive

E. PCR can be used for rapid haplotype analysis


2

Regarding pathogenesis of TB, one of the following factors is not involved:

A. MMP9

B. MMP1

C. MMP3

D. ESAT 6

E. ESAT 3
E

1. The quantity of ointment required to treat 3 – 5 year child leg and foot is:

A. 1.5 FTUs

B. 2 FTUs

C. 3 FTUs

D. 4 FTUs

E. 5 FTUs
C

1001. The following causes vasodilatation in acute inflammation:

A. Nitric oxide

B. Leukotriene B4
C. Prostacycline

D. IL8

E. Fibronectin
A

1000. Fat embolism:

A. Complicates fatty liver

B. Occurs within 6 heur following fracture

C. Diagnostic is done by finding fat globules in the urine

D. Occurs in decompression sichness

E. Causes thrombocytosis
B
G. Which of the following advers effects is most likely to limit the maximum dosage of foscarnet
?

1. Cardiac arrhythmias

2. Hepatotoxicity

3. Nephrotoxicity

4. neurotoxicity

5. pulmonary fibrosis
3

IV IG is not used in :

A. Kawasaki disease

B. Erythrodermic psorisis

C. Pemphigus Vulgaris

D. Dermatomyositis

E. Necrotizing fasciitis
B
Raindrops on a dusty road appearance is seen in a toxicity to:

A. Antimony
B. Beryllium

C. Arsenic

D. Bismuth

E. Chromium
C
The hypodermis is condidered amongst fonctions to be an endocrine organ. The hormonal it
release is:

A. Amylase

B. Lipase

C. Glucocidase

D. Fructocidase

E. Leptin
E
Retinoid topical preparation pass into the keratinocytes through the process of :

A. Attachment to special cell wall receptors

B. Phagocytosis

C. Liquefaction of the cell membrane

D. Osmosis

E. Diffusion gradient
E

One of the following drugs is not reported to exacerbate psoriasis:


A. Antimalarials
B. B-blockers
C. Carbamazepine
D. Ibuprofen
E. Cimetidine
E

Calcium absprption:

A. Is not affected by age

B. Is stimulated by hydrochloric acid


C. Occurs in distal ileum

D. Is increased by phosphate in small intestine

E. Is delayed by bile
B

The lipids in Odlands body are secreted in the form of :

A. Cholesterol

B. Ceremides

C. Sphingolipids

D. Glycosylated lipids

E. Triglycerides
Which of the following is not true:

A. Melanocytes are derived from neural crest.

B. The different stages of Melanosomes correlate with the degree of Melanization.

C. Melanosomes that are involved in brown Melanin synthesis are spherical

D. Keratinocytes produce GF which are mitogenic to Melanocytes

E. Proliferation of Melanocytes rely on vit. D synthesis within the Epidermis.

Regarding features of premature skin, One of the following is not true:

A. Skin thickness : o.9 mm

B. Epidermal thickness: 20-25microns

C. Stratum cornum thickness: 9-10 microns

D. Size of collagen fiber bundles are small

E. Few mature melanosomes

Which of the following is the most single characteristic finding in child abuse:

A. bruises

B. blunt trauma

C. pinch marks
D. buckles

E. loop marks

In patients with acute measles, One of the following is recommended to prevent significant
morbidity and mortality?

1. Vitamin K

2. Selenium

3. Vitamin C

4. Vitamin B12

5. Vitamin A

Highest density of hairs is usually in:

A. Forehead

B. Eyebrow

C. Scalp

D. Chin

E. Upper lip
A

Complete replacement of toenail requires:

A. 3 months

B. 6 months

C. 12 months

D. 15 months

E. 18 months
E

Which one of the following melanocortin receptors is majorly distributed in cortex?

A. MC1R
B. MC2R

C. MC3R

D. MC4R

E. MC5R
4

What anesthetic should be avoided in a patient with an amide allergy for a punch biopsy?

A. Lidocaine

B. Benzocaine.

C. Cocaine .

D. Procaine

E. Tetracaine
A

Ptychotropism is the affinity of lesions to develop on:

1. Mucous membranes

2. Skin folds

3. Extensor surfaces

4. Acral regions

5. Keratinized structures (Nails and Hairs)


2

One of the following is not true about CD1a:

A. It is the most useful marker for detecting human Langerhans cells.

B. It is exclusively expressed on Langerhans cells in inflamed tissue only

C. It does not apply to HLA-DR

D. It does not exist in the murine system.

E. It is exclusively expressed on Langerhans cells in normal and inflamed tissue.


B
Which of the following is involved in stimulating adaptive immunity:
A. IL-4

B. IL-2

C. IL-23

D. IL-12

E. IFN-beta
B

Dermoscopically, a uniform pigment network is characteristic for:

A. Melanocytic nevus of acral skin

B. Dermal nevus

C. Halo nevus

D. Junctional nevus

E. Solar lentigines
D

Dermoscopically, circle in a circle is characteristic for:

A. Superficial spreading melanoma

B. Nodular melanoma

C. Lentigo maligna Melanoma

D. Acral lentigenous Melanoma

E. Unclassifiable melanoma
3

The classical and alternative pathway meet at complement component:

A. c3

B. c4

C. c4b

D. c5

E. factor d
A

A complement component which is strongly chemotactic for neutrophils is:

A. C3

B. C3b

C. C5a

D. C5b

E. C9
C

When performing a salt – split DIF of a skin biopsy from a patient with BP, WHAT IS the
expected result?

A. Granular IgA in the dermal papillae

B. Intercellular IgG antibodies located in the epidermis

C. Linear IgG antibodies located on the epidermal side of the BM

D. Linear IgA antibodies located on the BM

E. Linear IgG antibodies located on the dermal side of the BM


C
O
The synonym tuberculosis luposa is characteristic for the following type of tuberculosis:

A. Lupus vulgaris

B. Tuberculosis

C. Scrofuloderma

D. Tuberculosis verrucosa cutis

E. Tuberculosis gumma
A

In HPV, the gene which activates telomerase is:

A. E1

B. E2

C. E4
D. E6

E. E7
D
One of the following is not true about IgG:

A. It is the most abundant Ig.

B. It is the major Ig of the innate immune

C. IgG 1 and IgG3 are potent activators of the classic pathway

D. IgG2 is less effective

E. Most of the autoimmune dermatosis which autoantibodies are mediated by IgG, most often
2

There are 5 types of human Ig antibodies, IgA, IgD, IgE, IgG, IgM. Each of them has a typical
physical conformation. What type of physical conformation does the IgM usually have?

A. Monomeric

B. Dimeric

C. Trimeric

D. Pentameric

E. Hexameric
D.

Complement component C3 is cleaved by:

A. C3b

B. C3bBb

C. Factor B

D. Factor D

E. Factor H
B

Interferon beta is produced by:

A. Fibroblast
B. Macrophage

C. Activated T cell

D. B – cell

E. All of the above


A

The virus causing hand, foot and mouth dis belongs to what family of viruses?

A. Poxviridae

B. Picornaviridae

C. Herpesviridae

D. Paramyxoviridae

E. Papillomaviridae
B

Which of the following cause primary infection of lymphatics?

A. Gardnerella vaginalis

B. Neisseria gonorrheae

C. Trichomonas vaginalis

D. Chlamydia trachomatis

E. Mycoplasma genitalium
D

Which of the following has a protective role in urticarial:

A. Leukotriene C4

B. TNF-β

C. Prostaglandins E2

D. Platelet-activating factor

E. IL-4
3

Nocireceptors repond to:


A. Temperature changes

B. Harmful stimuli

C. Chemical solutions

D. Mechanical forces

E. Light
2

Which of the following antimicrobial peptides is expressed in sebaceous gland:

A. Antileukoprotase

B. Cathelicidin

C. Psoriasinx

D. β- defensing 1

E. β- defensing 2
Not expressed :: A

What type of virus is the causative agent of Rubella?

A. Poxvirus

B. Parvovirus

C. Togavirus

D. Orthomyxovirus

E. Picornavirus
3

A pregnant lady was referred from the antenatal care to dermatology clinic with the diagnosis of
severe pruritus gravidarum. What are the type of lesions that consolidate the diagnosis?

A. Excoriations

B. Oozing plaques of dermatitis.

C. Urticated papules and plaques.

D. Erythematous follicular papules.


E. Blisters.

An 18-year old male presents to the emergency room with palpable purpura on the legs, arthritis
and abdominal pain following upper respiratory tract infection. Which of the following is most
likely to be found in this patient?

A. Eosinophilia.

B. Aphthous ulcer.

C. Proteinuria

D. Tophi.

E. Alopecia totalis.
C
.Which one of the following signs belongs to neurofibromatosis?

A. Crowe sign

B. Leser-Trelat sigh.

C.Trousseau sign.

D. Hertoghe sign.

E. Ausputz.
A

An elderly patient presents with recurrent attack of painful unilateral papulovesicular lesions on
the trunk with a dermatomal distribution. What is the next appropriate step?

A. Swabs for bacterial culture.

B. Neurological clinical evaluation.

C. Topical acyclovir and rest at home.

D. Investigation for underlying malignancy

E. Punch biopsy.
D

A patient develops polyarteritis nodosa with orchitis. What is the likely etiology?

A. Hepatitis B

B. HIV
C. Mumps

D. EBV

E. HPV
A
Which of the following laser therapy targets has the longest thermal relaxation time?

A. Melanosomes

B. Capillaries

C. Leg vein

D. Hair follicules

E. Tattoo ink particles


3

Latent desquamation phenomenon means:

A. The scale appears as heaped up column of scale

B. The scale observed only after scraping of the lesion

C. Surface excavation of the epidermis resulting from scratching

D. Scales are small and branny

E. Scales split of from the epidermis in sheets


2

One of the following metal salts may cause persistent folliculitis:

A. Beryllium.

B. Calcium oxide.

C. Cobalt salt.

D. Arsenic compounds

E. Copper salts.
4
Palamo-planter keratoderma is associated with the following type of pityriasis rubra pilarios
(PRP):

A. I
B. II.

C. III.

D. IV

E. V.
4
.Raynaud Disease Characteristically:

A. Occurs almost entirely in young male cigarette smokers.

B. Is associated with atherosclerosis.

C. Occurs primarily in tropical climates.

D. Is the result of a vasospastic reaction

E. Involves elastic arteries.


4
Hertoghe sign is not seen in:

A. Lepromatous leprosy

B. Hypothyroidism

C. Lupus Vulgaris

D. Systemic sclerosis

E. Atopic dermatitis
3

Which of the following is true about hepatitis B virus infection:

A. Vertical transmission is rare.

B. Only 10% of infected persons have spontaneous remission.

C. Hepatitis vaccination should be used after acute infection.

D. The presence of hepatitis e antigen indicates high infectivity

E. It can lead to development of hepatocellular carcinoma


D

Which of the following is not a feature of anxiety neurosis:


S. Confusion.

T. Intial insomnia.

U. Panic attacks

V. Tremor.

W. Frequency of micturition.
3

Premature babies are not prone to one of the following:

A. Iron deficiency anemia.

B. Neonatal jaundice.

C. Hyaline membrane diseas.

D. Birth trauma.

E. Rickets
5

For experimental studies, the best growth culture for treponema palladium is:

A. Rat bladder

B. Monkey esophagi

C. Rabbit testicules

D. Guinea pig stomach

E. Treponema palladium Cannot be cultivated


3

Infantile kala - azar is most commonly caused by :

A. L. donovani

B. L .amazonensis

C. L . infantum

D. L. major

E. L. tropica
3
Which of the following listed facts is incorrect during patient education of Herpes Zoster?

P. Most people recover over several weeks.

Q. Workup for immunodeficiency is unnecessary

R. All adults age 50 and older should get a vaccine.

S. The vaccine is live attenuated virus.

T. This is not sexually transmitted.


2

Which of the following is characteristic symptom of hypomania:

P. Irritability.

Q. Inability to concentrate.

R. Exaggerated feeling of well-being

S. Increased sleepiness.

T. Tiredness.
3

Which of the following neonatal conditions does not require treatment: ?

A. Milia.

B. Mongolian spots

C. Umbilical granuloma.

D. Subconjunctival haemorrhage.

E. Erythema toxicum.
2👍 👍

The most potent cytokine involved in cidal activity against leishmaniasis is:

A. IL-2

B. INF-gamma

C. IL-4

D. TNF-α
E. INF-α
2👍 👍

The prominent cell type infected by HHV7 is:

A. Endothelial cell

B. Keratinocyte

C. T – cell

D. B – cell

E. Macrophage
C

Which of the following is true regarding scarlet fever?

A. The incubation period is usually short, lasting only a few days

B. Forchheimer spots, seen on the tongue and gums, are signs of this disease.

C. The typical rash spares folds of skin, which are known as Pastia lines.

D. Mediated by pyogenic exotoxin E.

E. The rash typically appears on the dorsal hands and spreads to others parts of the body.
A

Which of the following is false regarding impetigo?

P. Caused by superficial gram-positive organisms.

Q. Pediculosis capitis is a risk factor for development of this condition.

R. These infections may be followed are sometimes followed by acute glomerulonephritis.

S. Topical antibiotic monotherapy is appropriate.

T. Topical polymyxin would be an effective therapy


5

Regarding intrauterine HSV infection, which of the following findings would least likely be
present in the affected newborn during the neonatal period?

A. Microcephaly.
B. Cutaneous lesions.

C. Chorioretinitis.

D. Sezures.

E. Sepsis
A

A patient presents with Fixed Drug Eruption. Which of the following drug is most likely the culprit
of this reaction :
F. Antifungals
G. Penicillin
H. Albuterol
I. Trimethoprim - sulfamethoxazole
J. Bleomycin
4

The toxin produced in SSSS cleaves the epidermis by reacting with the following molecule:

A. Desmoglein – 1

B. Democollin

C. BP Ag 180

D. α 6 – β 4- Integrin

E. plectin
A
. Non inflammatory types of tinea capitis is caused by :?
a. Microsporum Gypseum
b. Trichophyton Violaceum
c. Trichophyton Mentagraphytes
d. Microsporum Canis
e. Trichophyton rubrum
B
One of the following is alkylamine class of antihistamine:
a. Triprolidine HCL
b. Diphenhydramine HCL
c. Hydroxyzine HCL
d. Azatadine maleate
e. Promethazine HCL
A
One of the following is not correct about the side effect of amphotericin B :
a. Hypersensitivity reaction including anaphylaxis
b. Febrile reaction
c. Microchromic anemia
d. Decreased potassium and magnesium serum level
e. Renal dysfonction
C

Phage 2 type 71 staphylococcus aureus is not found in patient with :


a. Lichen plan
b. Psoriatic scales
c. Atopic dermatitis
d. Dermatomysitis
e. Hemodialysis
B

Geophilic superficial dermatophytes:

A. Are incapable of infecting humans

B. Spread by spore formation

C. Their spores are short lived in fomites

D. Incapable of inciting an inflammatory response

E. Most common member is m canis


B

The prospective epidermis originate from:


A. A surface area of the late gastrula
B. A surface area of the early gastrula
C. Neural crest
D. Medulla
E. Mesoderm
B

Which of the following is true regarding Anthrax?

A. Is likely caused by a gram negative spore forming rod.

B. The causative agent produces two types of exotoxins

C. Azithromycin is a first line treatment.

D. There are two clinical forms of this disease: inhalational a cutaneous.

E. None of the above.


2

Which of the following is not true about systemic lupus erythromatosus:


A. It is more common in women.

B. Remissions is commonly occur in pregnancy

C. Anti-DNA antibodies are absent in drug induced lupus.

D. Antimalarial can reduce frequency of exacerbations.

Renal involvement carries the worest prognosis


2

Main inflammatory mediator in inflammatory phase of wound healing is:

A. IL-6

B. IL-12.

C. TGF-alpha.

D. PDGF.

E. IL-4.
A

One of the following is medium- sized vasculitis

A. Henoch – schonlein purpura

B. Urticarial vasculitis

C. Polyarteritis nodosa

D. Churg-straus syndrome

E. Temporal arteritis
C

Hertoghe sign is thinning or loss of the outer third of the eyebrow and is seen in:

A. Chronic eczema

B. Hyperthyroidism

C. SLE

D. Systemic sclerosis

E. PCT
A

One of the following cytokines has beneficial effect on wound healing:

A. IL-1.

B. TGF-B

C. IL-8.

D. IL-6.

E. TNF.
B

High-frequency electrosurgery method does not include:


A. Electrolysis
B. Electrodesiccation.
C. Electrofulguration.
D. Electrosection.
E. Electrocoagulation.
A

Cyclobutane pyrimidine dimers are produced by which of the following processes?

A. Normal flora interaction with corneocytes.

B. Ultraviolet light interaction with the skin

C. Breaking down of corneodesmosomes.

D. Antigen presentation to lymphocytes.

E. Wound healing.
B

Verrucous carcinoma is probably best considered as:


F. A typical SCC that happens to display a warty clinical morphology
G. A melanoma with a warty clinical morphology
H. A distinct clinicopathologic variant of low-grade SCC
I. A verrucous form of malignant acanthosis nigricans
J. A distinct variant of high -grade SCC
3

The perception of the color of the skin is related to which wavelength region of the following?

A. 200-290 nm.

B. 290-320 nm.
C. 320-400 nm.

D. 400-700 nm

E. 700-1200 nm.
D

Imiquimod increases production of Which cytokine ?

A. IL - 1

B. IL -3

C. IL -5

D. TGF - beta

E. IFN –alpha
E

What is a collodian preparation ?

A. Semi- solid

B. Oil – in - water

C. Water- in - Oil

D. Hydroalcoholic liquid

E. Cellulose nitrate in organic solvent

The best drug for treatment of cytomegalovirus (CMV) infection is:

A. Valacyclovir.

B. Ganciclovir

C. Foscarent.

D. Acyclovir.

E. Cidofovir.
2

Which of the following is an emulsifier?


A. Urea
B. Lanolin
C. Glycerin
D. Propylene glycol
E. Pyrrolidone carboxylic acid
B

Regarding telogen:

A. Telogen hairs are radiosensitive.

B. Telogen duration is relatively fixed for a particular body site.

C. The percentage of hairs in telogen is independent of body site.

D. In general, body hair has a lower percentage of hairs in telogen than scalp hair.

E. A and B
E

Which of these dermatophytes causes Tinea capitis without hair loss?

A. Trichphyton violacium

B. Microsporum lanosum

C. Epidermophyton floccusum

D. Trichophyton gypsum

E. Trichophyton rubrum
1

One of the following is not a clinical feature of secondary syphilis:

A. Piyriasis rosea-like.

B. Generalized papulosquamous.

C. Pink papules.

D. Bullous

E. Lesions of the palms and soles can have a collarette of scale.


D

Side effect of Bleomycin includes:

A. Raynaud’s phenomenon
B. Sedation.

C. Discoloration.

D. Xerosis.

E. Ocular irritation
A

One of the following is not correct about colchicine:

P. Prevents microtubule assembly.

Q. Decreases neutrophilic chemotaxis.

R. Treatment of neutrophilic dermatitis.

S. Pregnancy category B.

T. Side effects commonly gastrointestinal dermatosis.


4

Which of the following is the action of Glutathione?

P. Degrader of keratin filaments.

Q. Solvent for the cornified envelop.

R. Antioxidant

S. Sebum emulsifier.

T. Stabilizer for gap junction.


3

Alertnate morning glucocorticosteriod therapy is not useful in reduce risk in which of the
following:

K. Myopathy.

L. Growth suppression.

M. Cataract

N. Hypertention.

O. Opportunistic infection.
3
The longest peak effect of interferons is seen in :

A. Interferon alpha2a

B. Interferon a N3

C. Interferon Alphacon-1

D. Interferon 1b

E. Interferon Beta1b
1

One of the following is not a feature of sweat secreted by apocrine glands:

G. Sterile

H. Odorless

I. Dilute

J. PH 5 – 6.5

K. Continuous secretion
3 viscous

One of the following is not a contraindication for topical Tazarotene:

A. Unstable plaque psoriasis in phase of progression

B. Leukopenia

C. Erythrodermic psoriasis

D. Allergic contact dermatitis to tazarotene

E. Pregnancy or lactation
2

One of the following drugs not cause exanthematous eruption:

P. Aminopenicillin

Q. Sulfonamides

R. Tetracyclines
S. Anticonvulstants

T. Allopurinol
3

One of the following drugs is not an inducer of psoriasis:

A. Lithium

B. B-blockers

C. Antidepressants

D. IFNs

E. Antimalarial
3

Which of the following lasers, the target chromophor is vascular :

A. Argon

B. Erb- YAG

C. Pulsed dye

D. Alexandrite

E. Co2
3

Which of the following lasers is not used for treatment of nevi ?

G. Q switched ND :YAG

H. Long pulsed ruby

A. Q switched Alexandrite

I. KTP

J. Diode
4

One of the following is not bactericidal:

A. Aminoglycosides.
B. Penicillins.

C. Sulphonamides

D. Quinolones.
3

One of the following is not true about lepromatous leprosy:

A. Numerous lesions are present macules, papules, nodules.

B. Asymmetrical distribution

C. Vague border and difficult to distinguish between normal and affected skin.

D. Sensation is not affected

E. Many bacilli could be found in skin lesion.


2

Positive hair perforation test is caused by:

A. T. mentagrophytes

B. M. audouinii.

C. T. schoenleinii.

D. T. concentricum.

E. T. rubrum.
1
Which of the following is an important sweat component?

h. Lactate

i. Uroconic acid.

j. Phosphates.

k. Pyrovates.

l. Citric acid
1

Sphingosine is thought to play a role in the regulation of basal cells division. It is produced by:

g. Basal cells.
h. Fibroblasts.

i. Spinous and granular cells

j. The corneal layer.

k. Merkel cells.

3
One of the following is a zoophilic dermatophyte:

P. T.rubrum.

Q. T.tansurans.

R. T.concentricum.

S. T.verrucosum

T. Epidermophyton floccosum.
4

Heck’s disease is caused by:

A. HPV 3.

B. HPV 13

C. HPV 10.

D. HPV 16.

E. HPV 11.
2

Which of the following is not a tick-born disease:

P. Lyme disease.

Q. Rocky mountain spotted fever.

R. Babesiosis.

S. Human monocyticehrlichiosis.

T. Yellow fever
5 mosquito

Dermoscopically, a lattice-like pattern is characteristic for:


a. Recurrent melanocytic nevus

b. Pigmented spindle cell nevus

c. Melanocytic nevus of acral skin

d. Atypical melanocytic nevus

e. Medium – sized congenital nevus


3

The typical laser pulse duration for terminal hair follicule is:

a. 3 – 100 ms

b. 0,4 – 20 ms

c. 0,1s

d. 10 – 100 ns

e. 10 ns
1

A patient of herpes zoster developed granuloma annulare at the same site. What is this
phenomenon/sign?

Q. Pathergy.

R. Koebner.

S. Reverse Koebner.

T. Autosensitization.

U. Isotopic
5

One of the following is not correct about Favus:

P. It caused by T.schoenleinii.

Q. Hyphae and air spaces are observed within the hair shaft.

R. By Wood’s light there is bluish-white fluorescence.

S. Scutula is characteristic clinical feature.


T. Non scarring alopecia may develop in chronic infections
5

The development of HIV-1 antibodies is determined by:

L. Viral RNA.

M. Viral DNA.

N. Western blot

O. PCR.

P. P24.
3

Which disease is transmitted by affected males to their daugters but not to their sons?

A. Phenylketonuria

B. Incontinentia pigmenti

C. Acrodermatitis enteropathica

D. Anhidrotic ED

E. Hereditary haemorrhagic telangiectasia


2

What refer to Cheiroarthropathy?

A. Pachydermodactyly

B. Diabetic thick skin

C. Restrictive dermopathy

D. Vinyl chloride induced scleroderma

E. Nephrogenic systemic sclerosis


2

The main permeability barrier in the lamina densa is:

A. Heparin sulphate proteoglycan

B. Collagen IV.
C. Laminin 5.

D. Nidogen.

E. Alpha-6-beta-4 integrin
A

One of the following is affected by loss- of – function mutations within the filaggrin gene:

A. Lamina lucida

B. Lamina densa

C. Hemidesmosome

D. Keratohyaline granule

E. Membrane coating granule


D

Which of the following is not a feature of Nisseria gonorrhoae:

F. Is a Gram negative bacteria.

G. Grows in hemoglobin – containing medium.

H. It needs 3-5% CO2.

I. It has polysaccharide capsule

J. It canot tolerate dryness and low temperature.


4

The mode of inheritance in which both parents are not affected is:

A. AR

B. AD

C. X- linked recessive

D. X – linked dominant

E. A semidominant
A

The major reservoir of Leishmania major is:

A. Human.
B. Dogs.

C. Foxes.

D. Rodents

E. Hydraxes.
4

Which one of the following accelerates nail growth:

P. Night.

Q. Old age.

R. Summer

S. Fever.

T. Ca, Vit D.
3

Melanocortin MC-4 receptor is found in:

A. Pripherial tissue.

B. Placenta.

C. Adrenal cortex.

D. Melanocyte.

E. Brain

Stage II melanosome includes:

A. Spherical with no melanin deposition.

B. Oval with minimal melanin deposition and high tyrosinase activity

C. Oval with high melanin deposition and high tyrosinase activity.

D. Oval with moderate melanin deposition and high tyrosinase activity.

E. Oval with high melanin deposition and minimal tyrosinase activity.


2
By dermatoscopy, melanin located at the papillary dermis appear as:

A. Black

B. White

C. Brown

D. Slate blue

E. Red – black
4

Superantigen T – cell interaction lead to activation of :

A. 5 – 30% of entire circulatory T – cell population

B. O.o1% of entire circulatory T – cell population

C. 50% of entire circulatory T – cell population

D. 75% of entire circulatory T – cell population

E. Almost the entire circulatory T – cell population


1

Cyanosis:

A. Is yellowish discoloration of the sclera.

B. Is produced by high pCO2.

C. Can easily occur in Anemia

D. Occurs more with polycythemia

A. Is due to high oxygenated.


4

Which of the following substances has the greatest permeability across the basement
membrane:

A. Glucose.

B. Albumin.

C. Aliphatic Alcohol.
D. Dextran.

E. Sodium Chloride

One of the following is not correct about stem cell:

A. Self renewal is unlimited.

B. Potential for differentiation is multipotent.

C. Cycling in normal epidermis is slow

D. Proliferative potential is high.

E. Growth in culture is small abortive clone.

Wood’s lamp filter is made of:

A. Tin and chromium oxide

B. Nickel hydride and barium silicate

C. Nickel oxide and barium carbonate

D. Nickel oxide and barium silicate

E. Silicon dioxide
4

Which of the following antifungal is category D in pregnancy ?

A. Terbinafine

B. Voriconazole

C. Amphotericine B

D. Flucytosine

E. Caspofungin acetate
2

. One of the followings is not a form of sycosis:


A. Tuberculoid
B. Barbea
C. Lupoid
D. Mycotic
E. Herpetic
A

The arterial supply for a paramedian forehead flap is the?

A. Supraorbital artery

B. supratrochlear artery

C. angular artery

D. infratrochlear artery

E. infraorbital artery
B

Regarding calcium homeostasis:

A. Plasma Calsium Level is 5 Mmoles Per Liter.

B. Calsium is mainly transported bound to albumin

C. Calsium requires vitamin C for its absorption.

D. 50% of Calsium in plasma is bound to phosphates.

E. Vegetables contain considerable amounts of calcium.


2

Dopamine:

A. Is secreted from the anterior pituitary gland.

B. Stimulates prolactin secretion.

C. Is the precursor for adrenaline and nor adrenaline

D. Transported to the pituitary gland via axons

E. Can be released by parasympathetic nerves.


3

In cases of shock, which of the following compensatory factor is responsible for increasing blood
pressure:

A. Atrial stretch receptors.

B. Barao receptor reflex.


C. Bainbridge reflex.

D. Carotid body chemo receptors

E. Ischemic brain response.


B

Which of the following muscles is not innervated by the temporal branch of facial nerve:

g. Frontalis muscle

h. Corrugator supercilii Muscle

i. Orbicularis oculim (upper portion )

j. Orbicularis oculim (lower portion )

k. Auricular muscle ( anterior and superior portion)


4

Irrevesible cellular injury does not cause:

A. Progressive loss of phospholipids.

B. Decreased activity of Na+ / K+ AT pase causing cellular swelling.

C. Damage to the cellular cytoskeleton.

D. The presence of reactive O2 species.

E. The loss of the intracellular amino acids glycine and L-alanine


5

One of the following markers is ( – ) in blood vessels :

a. Laminin

b. Collagen type XVIII

c. CD34

d. VEGFR-2

e. VEGFR-3
E

One of the following is non- acute porphyria:


a. Acute intermittent porphyria

b. Variegate porphyria

c. Hereditary coproporphyria

d. PCT

e. ALA-D deficiency porphyria


D

The best dressing type for achieving mild hemostasis is:

A. Alginates
B. Collagens
C. Petroleum gauze
D. Hydrocolloids
E. Foams
A 👍 t 145-4

Regarding the cellular mechanisms of vascular leakage, which of the following is INCORRECT:

K. The ‘immediate transient response’ Only Occurs in Venules of 20-6- µm Diameter.

L. Sunburn Can Aause a Delayed and Prolonged Vascular Leakage.

M. The Endothelial Injury Caused By Leukocytes is Due to Free Radical-induced Damage.

N. The Cytoskeletal and Junctional Retraction Mechanism is Mediated by The Complement


System

O. The Most Common Mechanism For Increased Vascular Permeability is “Endothelial


Contraction” (“immediate transient response”)
4

Free Radicals:

A. Are not a byproduct of metabolism.

B. Can only form in the presence of oxygen

C. May arise by absorption of radiant energy.

D. Are removed with the aid of vitamin A derivatives.

E. Contain an extra protein that may bind to and destroy organic molecules.
2

The arterial supply for a paramedian forehead flap is the?

A. Supraorbital artery

B. supratrochlear artery

C. angular artery

D. infratrochlear artery

E. infraorbital artery
B

Dermoscopically, a globular pattern with hyphae-like structures is characteristic for:

a. Recurrent melanocytic nevus

b. Dermal nevus

c. halo nevus

d. Atypical melanocytic nevus

e. Medium – sized congenital nevus


5

Dermoscopically, a globular pattern with hyphae-like structures is characteristic for:

a. Recurrent melanocytic nevus

b. Dermal nevus

c. halo nevus

d. Atypical melanocytic nevus

e. Medium – sized congenital nevus


D

Melanocytes first appear in:

Q. Upper extremities.

R. Trunk.

S. Head
T. Genitalia.

U. Axilla.
3

The first embryonic hair follicles appear in the:

A. The scalp.

B. Public area.

C. Armpits.

D. Eyebrows

E. Chest.

Histamine exerts its effect during inflammation by:

Q. Vasoconstriction of post Capillary sphincters

R. Constriction of Large Arteries.

S. Acting on H2 Receptors on Mast Cells.

T. Causing Venular Endothelial Contraction.

U. Its Direct Effect on Macrophages.


1

Basal keratins are expressed in the embryo by:

A. The 5th Week

B. The 14th Week.

C. The 25th Week.

D. The 3rd Month.

E. The 4th Month.


B

In the embryo when the periderm is lost it is replaced by:

A. Stratum Corneum
B. Basal Layer.

C. Transient Layer.

D. Granular Layer.

E. Basement Membrane.
A

One of the following diseases is a TH2 response:

A. Multiple sclerosis.

B. Allergic contact dermatitis.

C. leprosy.

D. Atopic dermatitis

E. Tuberclosis.

The most likely cause of loss of shoulder abduction with wasting, paresthesia and weakness
after excision of an epidermal inclusion cyst of the side of neck is due to injury to the?

A. Spinal accessory nerve

B. Branchial plexus

C. Greater auricular nerve

D. 3d cervical nerve

E. Lateral thoracic nerve


A

For the color (green), select the associated tattoo substance ?

A. Cadmium sulphide

B. Cobaltous aluminate

C. Carbon

D. Iron oxides

E. Chromium sesquioxide
E

One of the following is not a member of IL-10 family:

A. IL-10.

B. IL-12

C. IL-20.

D. IL-22.

E. IL-24.
B

Which of the following Toll-like receptors is involved in recognizing flagellin:

A. TLR-2.

B. TLR-3.

C. TLR-4.

D. TLR-5

E. TLR-6.
D

Barrier function abnormalities in childhood Atopic Dermatitis does not include:

A. Increased transepidermal water loss

B. Increased stratum corneum moisture content

C. Increased permeability to hydrophilic substance

D. Decreased lipids

E. Decreased ceramides
B

Silk is preferable to nylon as the suture material for the ocular area because it :

A. Has greater tensible strength

B. Has less potential for sensitization

C. Is absorbable
D. Is less traumatic to tissues

E. Is less likely to cause infection


D

Which one of the following cytokines is a macrophage activator:

P. IFN Y

Q. IL-2

R. IL-4.

S. TNF-alpha.

T. IL-12.
1

Porphyria cutanea tarda can be a marker for:

A. Neurofibromatosis

B. Erythrodermic Psoriasis

C. Heriditary hemochromatosis

D. Paraneoplastic pemphigus

E. Diabetes Mellitus
C

The cornefied envelope is:‫مهم‬

A. A 15 – 20 nm insoluable layer of protein


B. Attached to the inner side of plasma membrane through corneosomes
C. Overlaid by a 5 nm lipid envelope
D. Its precursors are synthesized in the basal layer
E. Its Construction is carried out in 2 steps
C

One of the following type of HPV does not associate with common palmar , planter myrmecial
and mosaic warts:
A. HPV 1
B. HPV 2
C. HPV 3
D. HPV 27
E. HPV 57
3
Susceptibility of patients with atopic dermatitis to colonization and infections with
staphylococcus aureus is best explained by which of the following:

A. High expression of Th-1 cytokines in involved skin

B. Low expression of Th-2 cytokines in involved skin

C. High expression of beta-defensins in involved skin

D. Low expression of canthelicidin in involved skin

E. Low expression of penicillin-binding proteins in involved skin


D

Reduction of Langerhans’ cells in the epidermis results by which of the following?

A. UV exposure

B. Frictional trauma.

C. Dermatophyte infection.

D. Application of dinitrochlorobenzene.

E. Mycobacbacterial infection.
A

The epidermal basement membrane is permeable to:


A. Proteins
B. Proteoglycans
C. All types of cells
D. Small molecules
E. Lipid molecules
D

One of the following phenotypic markers is not observed in migrating Langerhans cells:
A. Birbeck granules
B. Langerin
C. MHC class II
D. CD1a
E. CCR6
E

One of the following is not used in the treatment of acne :


A. Retinoids
B. Dapsone
C. Tetracycline
D. Anti -androgen
E. Steroids
E

One of the following infection may cause extreme prematurity:

A. Aspergillus
B. Streptococcus A
C. HSV
D. Haemophilus influenza
E. Streptococcus B
A table 34-1

One of the following is cyclooxygenase inhibitor:


A. Cyclosporine
B. Aspirin
C. NASDs
D. Methotrexate
E. Sulfsilamides
C

The cytokine which predominant in chronic atopic dermatitis lesion is:

A. IL-12

B. IL-4.

C. IL-5.

D. IL-13.

E. IL-6.
A

Antigen presenting cells (APC) activation is not associated with induction of the following
cytokine:

A. IL-1gamma

B. IL-6.

C. IL-12.

D. Chemokine’s.

E. MHC class I&II.


A
Patient presented with 6 months history of a large abcess in his mouth that spread to mandible.
The area has been surgically debrided. What treatment would you recommend :

A. Penicillin

B. Fluconazole

C. Itraconazole

D. Ivermectin

E. Gancyclovir
A

One possible complication in a 3 year old child with measles is:

A. Encephalomyelitis

B. Heart block

C. Wide spread venous thrombosis

D. Iritis

E. Hemolytic anemia
A

.Induction of adaptive immune response is not dependent on the following cytokine:

A. IL-2

B. IL-4.

C. IL-13.

D. IL-6.

E. IL-23.
D

One of the followings is not a feature of leukocyte adhesion with transmigration:

A. Migration of leukocytes as a result of stasis of blood flow in the microvasculature.

B. Interaction of integrins with immunoglobulin found on endothelial cells.

C. The selectins found only on endothelial cells

D. Icam-1 and vcam-1 adhesive molecules on the endothelial cells.


E. Neutrophils in the first 6-24 hours of acute inflammation.
C

In the complement system, which one of the following is not true?

A. C3a, C5a cause vasodilation, and increased vascular permeability.

B. C5a is a powerful chemotactic agent.

C. C5a activates the cyclo-oxygenase pathway of arachidonic acid metabolism

D. C5 can be activated by kallikrein.

E. C3b, C3bi can act as opsonins.


C

A nuclear or cytoplasmic staining with IgG is seen in DIF in which of the following :

A. LP

B. PR

C. Subacute cutaneous LE

D. Bullous SLE

E. Paraneoplasic pemphigus
C👍 👍 👍 CME

Which of the following may be used to specifically identify Tuberculosis infection :

A. ADH

B. TNF- alpha

C. CRP

D. Alpha -1 antitrypsine

E. Interferon gamma
E

Which of the following cytokines is produced by Th -1 cells:

A. IL-5.

B. IL-13.
C. IL-2

D. IL-15.

E. IL-4.
C

The nail bed differs from normal skin histologically in:

A. Lack of spinous layer.

B. Thick basal layer.

C. Thick spinous layer.

D. Lack of granular layer

E. Thick granular layer.


D

The most sensitive marker for melanoma is:

F. S100

G. HMB45.

H. Tyrosinase.

I. MART 1/ Melanin A

J. P16.
1

In which of the following filaggrin break-down amino acids are involved?

A. Corneodesmolysis.

B. White blood cell chemotaxis to the epidermis.

C. Skin antimicrobial defense.

D. Vasodilatation.

E. Water retention in the epidermis


E

The best suture technique for closing dead space and decreasing wound diameter is:
A. Buried butterfly suture

B. Running subcuticular suture

C. Purse string suture

D. Vertical mattress suture

E. Pulley stitch suture


D makki

Q – switched ruby laser is not a suitable modality of treatment for one of the following
pigmented lesions :

A. Epidermal nevus

B. Nevus spilus

C. Nevus of ota

D. Becker’s nevus

E. Café-au-lait macule
A

In which one of the following glycosaminoglycans there is no core protein?

A. Dermatan.

B. Chondroitin.

C. Keratan.

D. Heparan.

E. Hyaluronan
E

One of the following is not correct about special stain in dermatology?

G. Crystal violet stain amyloid purple with blue back ground.

H. Fontanamasson stain melanin black.

I. Massons’strichome stain smooth muscle green

J. Pearls iron stain hemosiderin blue.


K. Suden black stain lipid black.
3

Which cell surface component of T cell leads to T cell activation as part of “signal 2”
component?

g. CD 28

h. B7

i. CD 80

j. TNF-Alpha receptor

k. CD1a
1

Epstein – Barr virus utilizes Which cell surface receptor to gain access to cell:

a. CR2

b. B7

c. T- cell receptor

d. TLR 7

e. MHC II
A

Which one of the following options is microfilaments of 7-10 nm diameter?

A. Keratins

B. Collagens I and III.

C. Elastin.

D. Fubulins.

E. Collagen IV.
A

. A 10 y old male with a history of mild to moderate eczema has numerous molluscum
contagiosum lesions, which of the followings is true:

A. This entity usually present as a large tender nodule


B. Is malignant tumor with 30 – 40 % rate of metastasis
C. The eosinophilic granules in the nucleus consist of HPV virions
D. The 1st line treatment is excision
E. May be treated by topical retinoids
E

Fibrillin is:

A. A macroprotein.

B. Distributed Closely Parallel to Collagen Fibres.

C. Assists in Destruction of Elastin.

D. Plays an Adhesive Role between Matrix Compnents

E. Abundant in Reticular Layer.


D

The essential factor for cells to move out of the basal layer is:

A. Dissolution of The Hemidesmosome.

B. Dissolution of The Internal Plaque.

C. Downregulation of Collagen VII.

D. Upregulation of Plectin.

E. Downregulation of Integrin Alpha6beta4


E

Which of the following B cell receptors is involved in immunoglobin isotype switching:

a. CD 40

b. CD 19

c. CD 20

d. CD154

e. CD 22
A

The gene responsible for Goltz syndrome is :

a. Myosin Va

b. PORCN
c. Endoglin

d. BP Ag2

e. PAX3
B

The labeling index equals the proportion of cells in which phase:

A. The Mitotic.

B. S phase

C. G1.

D. G2.

E. G0.
B

One of the following is not correct about stains most cells red:

A. Choroacetate esterase stains myeloid cells and most cells red.

B. Gomori methenamine silver stains fugal cell walls black.

C. Gram (Brown0Bernn) stain gram-positive bacteria red

D. Pagoda stains amyloid orange.

E. Truant stains acid-fast organisms yellow-green by fourescence microscopy.


C

In which of the following sites, apocrine gland is not present:

A. External auditory canal.

B. Forehead

C. Anogenital region.

D. Eyelids.

E. Axillae.
B

WHICH One of the following is not a mediator for itch?

A. Claudin 1
B. Acetylcholine

C. Calcitonine gene related peptide

D. Opioid peptide

E. Prostaglandin E
A

The most common form of psoriatic arthritis is :

A. Distal interphalangeal arthritis.

B. Arthritis mutilans.

C. Spondylitis.

D. Arthritis indistinguishable from rheumatoid arthritis.

E. Asymmetric oligoarthritis
E

Mark the WRONG statement about pityriasis aminatacea:

A. It can be a manifestation of sebrrhoeic dermatitis of the scalp.

B. It can be due to psoriasis.

C. It can be a manifestation of eczema.

D. It can be caused by fungal infection.

E. The average age incidence is 5 -40 years


E

The best dressing for WOUNDS after laser, dermabrasion or chemical peels is:

P. Alginates

Q. hydrogels

R. films

S. Hydrocolloids

T. Foams
2👍
T 145-4
In WHICH One of the following conditions Maculae caeruleae are seen ?

A. Jelly fish bites

B. Pediculosis pubis

C. Multiple mosquito bites

D. Sexually transmitted vulvo-vaginal candidosis

E. Sexually transmitted Sanogenital warts in children


B

The short (< 4 weeks) latency period for drug-induced psoriasis is characteristic for the following
drug:

A. Antimalarials

B. ACE inhibitors

C. Lithium

D. β – blockers

E. Terbinafine
5

Which of the following is implicated in the pathogenesis of keratinocytes formation?

A. IL-1

B. IL-2

C. IL-3

D. IFN-gamma

E. TGF-beta
5

P –acnes can induce monocytes that activate:

A. TLR-4

B. IL-8

C. TNF δ
D. CDI b

E. Histone H4
D

One of the following is NOT CORRECT about hair cycle:

A. Thyroxin promotes growth.

B. Premature to telogen induction is induced by plucking of telogen hair shaft

C. Estrogen promotes anagen.

D. Corticosteroid retard anagen onset.

E. Anagen duration of terminal hair on legs is 19-26 weeks.


B

The structural assembly of WHICH one of the following collagen is transmembrane ?

A. Collagen IV

B. Collagen V

C. Collagen VIII

D. Collagen XIV

E. Collagen XVII
E

The pseudo darier sign occurs in:

A. Smooth muscle hamartoma

B. mastocytoma

C. neurofibroma

D. nevoid hypertrichosis

E. hemi hypertrophy
A

One of the following is not immunocompromised host diseases associated with CMV infection:
A. Cutaneous Vasculitis
B. Ulcers
C. Verrucous plaques
D. Nodules and hyperpigmented plaques
E. Erythema nodosum
E

The main target organism for valganciclovir is:


K. HSV 1
L. HSV 2
M. VZV
N. Resistant HSV
O. CMV
5
which one is not effective for treatment of CMV :
a. CIDOFOVIR
b. acyclovir
c. foscarnet
D. ganciclovir
E. valganciclovir
B

One of the following is photomechanical destruction method for hair removal:

A. PDL (80 nm)

B. Long pulsed ND:YAG Laser (1064 nm)

C. Q- switched ND: YAG Laser

D. Intense pulsed light

E. Normal mode ruby Laser (694nm)


B

One of the following nerves will respond slowest to local anesthesia?

A. Unmyelinated C- fibers carrying pain sensation

B. intermediate fibers carrying sensation of heat and cold

C. A-type fibers that carry pressure sensation

D. Motor neurons that innervate the skeletal musculature

E. Autonomic nerves that innervate smooth muscles

During cryotherapy, what minimum temperature is required for melanocyte destruction?


A. -5 degree C

B. - 25 degree C

C. - 50 degree C

D. - 100 degree C

E. -196 degree C
A

Which one of the following is not true about drug interactions of systemic antiviral agents:

A. Amphotericin increase serum acyclovir level

B. Cimetidine decrease rate of conversion of valacyclovir to acyclovir

C. Famcyclovir decrease digoxin levels

D. Interferon may worsen potential neurotoxicity of acyclovir

E. Zidovudine increase serum level of acyclovir and valacyclovir


C

One of the following is not true about mechanism of action of azathioprine:

A. Reduction of NK cell function

B. Reduction of CD8+ T cells

C. Inhibition of lymphocyte proliferation

D. Impaired neutrophil chemotaxis

E. Activation of antibody responses


D

One of the following is a 4th generation cephalosporin?

A. Cefapime

B. Cephalexin

C. Cefaclor

D. Cefotaxime

E. Ceftriaxone
A

Cleave of the epidermis takes place below what layer in bullous impetigo?

A. Stratum corneum

B. Lamina lucida

C. Stratum granulosum

D. Stratum spinosum

E. Basal layer
A

One of the following statement regarding the metabolism of local anesthetics is not true?

A. Amides are metabolized by the CYP450 system

B. Esters are metabolized by plasma pseudocholinesterase

C. Esters are excreted by the kidneys

D. Amides are excreted by the liver into the entero-hepatic circulation

E. PABA is a metabolic by product of Ester anesthetics


D

Pseudo-aneurysm as cutaneous sign of drug abuse is associated with:

A. I.V. cocaine

B. Arterial injection

C. Barbiturate

D. Heroin

E. Marijuana
B

In lipoid proteinosis the genetic defect is in:

A. Elastin.

B. Laminin B1.

C. Extracellular matrix protein -1


D. Fibrillin 1.

E. Collagen XVII.
C

One of the following scenarios warrants eradication of staphylococcus carriage through twice
daily application of mupirocin to the nares for 5 – 10 days?

A. Non – bullous impetigo

B. Bullous impetigo

C. Recurrent folliculitis

D. Gram – folliculitis

E. Hot tub folliculitis


C

One of the following treatment is not appropriate for methicillin resistant staphylococcus aureus
(MRSA) infection?

A. TMP-SMX

B. Minocycline

C. Clindamycin

D. Mupirocin

E. Linezolid
D

The highest risk of fetal varicella in pregnant women would be associated with one of the
following scenarios?

A. 30-Week pregnant woman who developed herpes zoster.

B. 20- Week pregnant woman who developed herpes zoster.

C. 30- Week pregnant woman who developed a varicella infection

D. 20-Week pregnant woman who developed a varicella infection

E. The above scenarios place the fetus at equal risk for neonatal varicella syndrome
D

Large vessel vasculitis is


A. Kawasaki disease

B. Takayasu ‘ arteritis

C. Churg- straus syndrome

D. Essential cryoglobulinemia

E. Wegener’ granulomatosis
B

One of the following is not common Phototoxic agent:

A. Griseofulvin

B. Furosemide

C. Naproxen

D. 8- methoxy psoralen

E. Doxycycline
A photoallergic not photo toxic table 87-5

One of the following is most likely to cause FDE?

A. Antifungals

B. Penicillin

C. Albuterol

D. Trimethoprim-sulfamethoxasole

E. Bleomycin
D

One of the following is not responsible for systemic corticosteroid induced osteoporosis:

A. Secondary hyperparathyroidism and bone resorption

B. Increased gastrointestinal absorption of calcium

C. Increased osteoclast activity

D. Decreased osteoclast activity

E. Increased renal excretion of calcium


D makki

About delusional parasitosis, One of the following is not true:

A. It is a false and fixed believe.

B. It is a part of schizophrenia

C. It is a hypochondriac in nature

D. The onset is at mid 50 – 60 years

E. Treated by pimozide
B

Moccasin type fungal infection is most commonly caused by:

A. Epidermphyton floccosum

B. Microsporum gypseum

C. Trichophyton rubrum

D. Trichophyton mentagrophytes

E. Microsporum canis
C

The genotype HLA-DQ2 has associated with:

A. Paraneoplastic disease

B. DH

C. Epidermolysis bullosa simplex

D. JEB

E. Bullous SLE
B

One of the following Lasers is ablative:

A. Co2

B. Q switched ND : YAG 532

C. Diode
D. KTP

E. Pulsed – dye laser


A

Outbreaks of dermatitis and folliculitis associated with swimming pools and hot tubs are often
caused by One of the following?

A. Listeria

B. Pseudomonas

C. Streptococcus

D. Shigella

E. Staphylococcus

Squalene epoxides enzyme is inhibited by:

A. Griseofulvin

B. Terbinafine

C. Fluconazole

D. Itraconazole

E. Amphotericin
2

Erythema multiform, Erythema nodosum, epidermolysis bullosa acquisita , together can be


caused by:

A. Sarcoidosis

B. Crohn disease

C. B- hemolytic streptococcal infections

D. ACE inhibitors

E. Mycoplasma infection
B

One of the following CYTOKINES is not released by Th0:


A. INF- beta

B. IL-2

C. TNF- alpha

D. IL-10

E. IL-13
1

One of the following induces the release of inflammatory mediators from mast cells:
A. C2a
B. C3b
C. C4a
D. C5b
E. C2b
C

One of the following is not correct about giant cells:

A. ARE found in both infective and non- infective granuloma

B. Are always derived from macrophages

C. Pathological feature of xanthomas and xanthogranulomas

D. Seen in herpes infections

E. Seen in spitz nevi


2

One of the following is not correct about mast cells:

A. They possess a central round to ovoid dark staining nucleus

B. The cytoplasm contain small granules

C. In normal skin can occur around blood vessels with dendritic morphology

D. They stain methachromatically with alcian blue

E. Commonly seen in various benign and malignant tumors


4

Which of the following types of acute porphyria is characterized by high urinary porphobilinogen
and high fecal protoporphyrin:
A. Erythropoitic protoporphyria

B. Acute intermittent porphyria

C. Variegata porphyria

D. Congenital Erythropoitic porphyria

E. Hepatoerythropoitic porphyria
C

One of the following is surface suture for face:‫جديد‬

A. 4.o or 5.0 nylon or polypropylene

B. 5.o or 6.0 nylon or polypropylene

C. 3.o or 4.0 nylon or polypropylene

D. 5.o silk or polyester

E. 4.o or 5.0 polyglactin or poliglecaprone


2

The common site of primary chancre in male is:

A. Prepuce

B. Glans penis

C. Penile shaft

D. Scrotum

E. Coronal sulcus
2 acc to fitz.
5 acc to makki

A young woman with acne taking an oral medication developed polyarthritis and raised liver
enzymes test. The most likely drug she was taking is:

A. Minocycline

B. Erythromycin

C. Oxytetracycline
D. Trimethoprim

E. Isotretinoin
5

One of the following herpetic virus is resistant to acyclovir?

A. HS genitalis

B. EBV

C. VZ

D. CMV

E. HSV
D

One of the followings is not FDA- approved indication for treatment of the following clinical
diseases by interferones:

A. Condyloma acuminata

B. AIDS –associated Kaposi, s sarcoma

C. Melanoma

D. Actinic keratosis

E. Chronic granulomatous disease


D

The site of action of antifungal drugs class polyens is :

A. Blocks DNA synthesis

B. Disrupts mitotic spindle

C. Binds cell membrane sterols

D. Interferes with cell membrane synthesis via inhibition of squalene epoxidase

E. Interferences with cell wall synthesis via inhibition of beta (1, 3) – D- GLUCAN synthesis
C

One of the following statements is not correct about azathioprine:

A. It has immunosuppressive and anti-inflammatory effects


B. It is capaple of influencing both humoral and cell mediated immune functions

C. It inhibits B -cell proliferation

D. It suppresses IgG and IgM formation

E. It has inhibitory effects on suppressor or helper T- cells more than B- cells


E

Granulysin:

A. Is produced by epithelial cells at sites of chronic inflammation

B. Belongs to the saposin family of proteins

C. Is found in human beings, plants and insects

D. Is structurally similar to defensins

E. None of the above


E

The best stain to demonstrate collagen is:

A. Scarlet red

B. Toluidine blue

C. GMS

D. Masson trichrome

E. hyaluronidase
D

One of the following lipids is not produced by sebaceous glands:

A. Triglycerides

B. Phospholipids

C. Esterified cholesterol

D. Free cholesterol

E. Waxes
B

Basal cell liquefaction is not a feature of :


A. LP

B. Follicular LP

C. Parapsoriasis

D. Poikiloderma atrophicans vasculare

E. LE
C

Langerhans cells cannot be identified with One of the following stains:

A. ATP

B. Aminopeptidase

C. OKT6

D. Argentaffin

E. Gold chloride
D

One of the followings is NOT CORRECT regarding penciclover:‫جديد‬

A. Mechanism of action similar to acyclovir

B. It reaches higher intracellular concentration than acyclovir

C. It is less potent than acyclovir because it does not cause chain termination of viral DNA

D. It has a poor bioavailability than acyclovir

E. It is available in tablets and topical forms


5 only topical

One of the following is NOT a major site for a distant metastases to scalp:

A. Breast

B. Ovary

C. Lung

D. Stomach
E. Pancreas
B

One of the following is NOT CORRECT about early indeterminate leprosy:

A. The first clinical feature is hyperpigmented nodule

B. The first clinical sign is numbness

C. The first Lesion is hypopigmented macule

D. Peripheral nerves not enlarged

E. Plaques and nodules do not occur


1

Histopathological reaction of the type of tuberculoid granuloma with caseation may be csused
by One of the following foreign substances :

A. Berylium

B. Paraffin

C. Talc

D. Tatoo ink

E. Silicone
1

One of the following is not a feature of cytology atypia:

A. Pleomorphism

B. Hyperchromasia

C. Infiltrative growth

D. Abnormal mitosis

E. Prominent nucleoli
C

Which of the following in pediculosis capitis is not true:

A. More common in girls

B. Uncommon in African – americans


C. Oral ivermectin is used to treat resistant cases

D. The parasire feed on hair keratin

E. Blow – dryers allow lice to become airborn


4

One of the following statements about treponema palladium is not true:

A. It has regular tight spirals

B. Limited capacity for DNA repair

C. Inability to survive outside an animal host

D. Can be cultured in vitro for extended time period

E. It has periplasmic flagella


4

One of the following drugs does not cause secondrary Raynaud's phenomenon:

A. Imipramine

B. Bleomycin

C. Amphetamines

D. Oral countraceptive

E. Nifidipine
5

Which one of the following is 3d line treatment of physical urticaria:


A. Chlorpheneramine
B. Desloratidine
C. IV. IG. infusion
D. Prednisone
E. Thyroxine
3

One of the following drugs is not a cause of small vessel vasculitis‫جديد‬

A. Opiates

B. barbiturates
C. Amphetamines

D. Cocaine

E. Marijuana
2

One of the following dermatophytes is not ectotrix type:

A. M. ferruginum

B. M. distortum

C. T. gourvilli

D. M.gypseum

E. M.audouini
C

Regarding the arterial pulse and pulse rate:

A. The Pulse at the wrist is due to the flow of blood through the artery

B. The radial Pulse is synchronous with the 1st heart sound

C. Hardening and tortuousity of the radial artery is an indication of atheroma elsewhere

D. Sinus bradycardia may result from a rising intracranial pressure

E. Pressure on the carotid Sinus may not reduce the rate in atrial flutter.
D

Pitting edema

A. May not be clinically detectable until the extra-cellular fluid volume has increased by 1% or
more

B. Not developed in a paralyzed patient

C. In cardiac failure is mainly due the rise in venous pressure

D. Develops in all normal subjects from time to time

E. Is a characteristic feature of myxedema.


C

Which One of the following disorders is the most likely to be associated with helicobacter pylori
infection:
A. Non- ulcer dyspepsia

B. Reflux esophagitis

C. Coeliac dis

D. Gastric lymphoma

E. Achalasia of the cardia


D

One of the following is not correct regarding mycophenolate mophetil: ‫جديد‬

A. Mycophenolic acid is reactivated by B-GIU- curonidase in the epidermis

B. Selectively inhibit the enzyme inosine monophosphate dehydrogenase.

C. The most common side effect is GI symptoms.

D. It is indicated for both pemphigus vulgaris and psoriasis.

E. Both cyclosporine and tacrolimas are increasing mycophenolic acid Level


5

HIV receptor on the host cell is:

A. Plasma membrane

B. Mitochondria

C. CD+4

D. Ribosomes

E. Plasmids
C

Viruses multiply by:

A. Binary fission

B. Mitosis

C. Meiosis

D. A complex life cycle


E. Conidia
D

Which one of the following dermatophytes causes endothrix type of hair invasion?

A. Microsporum audouinii

B. Microsporum canis

C. Trichophyton verrucosum

D. Trichophyton mentagrophytes

E. Trichophyton violaceum
E

Which of the following is not an action of cyclosporine therapy? ‫جديد‬

A. Inhibition of microphage inhibition migration.

B. Decrease synthesis and release of IL-1 by Macrophage.

C. Decrease synthesis and release of IL-2.

D. Inhibition of interferon production

E. Inhibition of suppresor T cells


E

One of the following is not a feature of Lidocaine toxicity:

A. Ototoxicity

B. Nystagmus.

C. Seizure.

D. Slurred speech.

E. Perioral numbness.
A

The cytotoxic T-Lymphocytes skill target cells by:

A. Antibody and complement

B. Generation of reactive oxygen species


C. Activation of monocytes

D. Liberation of perforin and Activation of apoptosis

E. TNF- alpha
4

Most of human connective tissues are derived mainly from

A. Ectoderm

B. Trophoblast

C. Embryoblast

D. Mesoderm

E. Endoderm
D

One of the following type of HPV does not associate with common palmar , planter myrmecial
and mosaic warts:
L. HPV 1
M. HPV 2
N. HPV 3
O. HPV 27
P. HPV 57
3

Vitamin B 12:

A. Is a potent anti-oxidant

B. Its deficiency causes pellagra

C. Vegetable seeds are major source

D. Is not stored in the body

E. Requires the intrinsic factor for absorption


E

.A swelling at the site of trauma histologically shows RBCs and haemosiderin-containing


macrophage. The best hemodynamic disturbance that matches this pathology is:

A. Shock

B. Edema
C. Thrombosis

D. Hemorrhage

E. Congestion
D

Prostaglandins are derived from:

A. Lysosomes

B. Golgi apparatus

C. Mitochondria

D. Phospholipids of cell membrane

E. As a constituent of the cytoplasm


D

Hyperemia of acute inflammation is due to:

A. Interleukin- 1

B. Fibrinogen

C. Indreased hydrostatic pressure

D. C5a

E. Emigration of leucocyte
A

Nephrotoxicity and genital ulcers are side effects of one of these :

A. Valacyclovir.

B. Gancyclovir.

C. Famcilovir.

D. Foscarnet

A. Acyclovir
D

Anti-Jo-1 antibodies are directed against


A. Topoisomerase

B. Gyrase

C. Histidyl transfer RNA synthetase

D. Phospholipase

E. Lysyl oxidase
C

Which of the following statements is true?

A. Calcitonin gene-related peptide and substance P are increased in the alopecia areata (AA)
scalp

B. Calcitonin gene-related peptide and substance P are decreased in AA scalp

C. Interleukins 1 and 1 inhibit hair growth in vitro

D. Tumor necrosis factor alfa enhances hair growth in culture

E. B and C
5

The pseudo-darier signs occurs in :

A. Smooth muscle hamartoma

B. Mastocytoma

C. Neurofibroma

D. Nevoid hypertrichosis

E. Hemihypertrophy
1

The epithelial cells are derived from

A. Monocytes

B. Lymphocytes

C. Astrocytes

D. Eosinocytes

E. Basocytes
5

Autonomic nervous systeme:

A. Restore homeostasis

B. Neuropathy lead to complete smooth muscles paralysis

C. Adrenaline is released by the most of post ganglionic sympathetic fibers

D. Sympathetic and parasympathetic always counteract each other

E. Consists of afferent from visceral receptors and efferent to smooth muscles


D

A test which detects the presence of a multiple number of antigens:

A. Complement fixation test

B. Passive latex agglutination

C. Direct fluorescent antibody test

D. Indirect fluorescent antibody test

E. Immuno-electrophoresis
E

The specific test for the diagnosis of neurosyphilis is :

A. TPHA

B. MHA-TP

C. SPHA

D. FTA-ABs

E. FTA-ABs 19s-IgM
C

The major chromophore in skin for the generation of single oxygen is:

A. DNA

B. Keratin

C. Urocanic acid
D. Selenium

E. Collagen
C

ACTIVATION of C5a results in:

A. Opsonization

B. Increased inflammation

C. Endotoxin shock

D. Activation of interferon

E. Activation of NK cells
B

Which complement component is composed of three separate proteins:

A. C1

B. C2

C. C3

D. C4

E. C5
A

The recommended daily dose of griseofulvin in cutaneous fungal infection is:

A. 10 mg/kg body weight

B. 50 mg/kg body weight

C. 2 mg/kg body weight

D. 100 mg/kg body weight

E. 20 mg/kg body weight


E

Which of the following lasers has crystal medium ?

A. Ruby
B. Argon

C. Carbon dioxide

D. Copper vapor

E. Rhodamine dye
A

Which of the following lasers has the shortest depth of penetration ? ‫جديد‬

A. KTP

B. Ruby

C. CO2

D. Excimer

E. Erbium : YAG
5

One of these structure is not a regular component of the papillary dermis:

A. Capillary

B. Venules

C. Blind ended lymphatics

D. Elastic fine fibres

E. Reticulin fibres
E

The inner plaque of the hemidesmosomes contains:

A. Fibronectin

B. Tenascin

C. BP antigen

D. Thrombospondin

E. E- selectin
3
The structures that compose the dermal papilla are:

A. Type IV collagen

B. Anchoring fibrils and plaques

C. Anchoring fibrils, blood vessels and lymphatics

D. 2 different Type of collagen, elastic mesh, blood vessels, lymphatics and nerves

E. Elastic fibrils , blood vessels and lymphatics


D

One of the following chemicals is not associated with the development of non-melanoma skin
cancer: ‫جديد‬

A. Mineral oil

B. Coal tar

C. Psoralen ( plus UVA)

D. Nitrogen mustard

E. Salicylic acid
5

Suture “4-0” vicryl compared to “5-0” vicryl has:‫جديد‬

A. More tensil strength

B. Increased suture memory

C. Smaller suture diameter

D. Smaller needle

E. Increased knot security


1

One of the following is derived from neural crest:

A. Keratinocytes

B. Merkel cells

C. Langerhans cells
D. Face dermis

E. Endothelial cells
4👍

The greatest density of mast cells is found in the:

A. Stratum spinosum

B. Stratum basale

C. Reticular dermis

D. Subcutaneous fat

E. Papillary dermis
5

Interferons production is usually induced by which type of Toll-like receptors (TLRs):

A. TLR3

B. TLR5

C. TLR7

D. TLR8

E. TLR9
A👍 👍 👍
Bolognia 82

Which one of the following local anaesthetics is used in pregnancy?

A. Lidocaine

B. Mepivacaine

C. Etidocaine

D. Ropivacaine

E. Tetracaine
A

One of the following drugs is the least likely to cause morbilliform eruption‫جديد‬
A. Opiates

B. Cocaine

C. Barbiturates

D. Amphetamines

E. Marijuana
B

.Regarding Th3 cells, One of the following is not true:

A. Th3 cells are a poorly characterized type of CD4+ Th cell.

B. Primerily secretes TNF-beta

C. Provides help for IgA production

D. Has suppressive properties against Th1

E. Has suppressive properties against Th2


B👍

One of the following stains is used for capsule of Cryptococcus neoformans?

A. PAS

B. Mucicarmine

C. Gomori methenamine silver

D. Giemsa

E. Fite – Faraco
B 👍 chapter 0

Human dendritic cells are generated from:

A. Spleen

B. Bone marrow

C. Peripherial blood

D. Regional lymph nodes

E. Thymus
C
Most mutations are generated during: ‫مكرر‬
ٌ ‫جديد غير‬

A. Replication of damage RNA

B. Replication of damage DNA

C. Repair of damage mitochondria

D. Repair of damage golgi apparatus

E. Formation of cell wall


C

Which one of the following stains has strong reaction in tumors with follicular differentiation?
‫جديد غير مكرر‬

A. Carcinoemberyonic antigen

B. Pancytokeratin

C. S-100 protein

D. Vimentin

E. Epithelial membrane antigen


B

The highest incidence of drug reaction is with:

A. Penicillin G

B. Barbiturates

C. Cephalosporins

D. Ampicillin

E. Packed red blood cells


D

Tetracyclines have anti-inflammatory effects which include:

A. Increasing matrix metalloproteinase activity

B. Inhibit leukocyte chemotaxis

C. Increase production of TNF a


D. Increase production of IL 1B

E. Decrease production of IL 1B
B

One of the following viruses is RNA- double stranded?

A. Echovirus

B. Coxsackie A

C. Hepatitis A

D. Rota virus

E. Respiratory syncytial virus


D👍

Which one of the following melanocortin receptors is highly expressed in melanocytes?

G. MC1R

H. MC2R

I. MC3R

J. MC4R

K. MC5R
1

Mutation in loricrin occurs in:

A. Vohwinkel syndrome

B. Ichthyosis vulgaris

C. Erythrokeratoderma variabilis

D. Olmsted syndrome

E. Pachyonych
A

One of the following adhesion proteins is not a component of hemidesmosome anchoring


filament complexe:
A. Laminin 311

B. Plectin

C. BP Ag 1

D. Integrine

E. Tetraspan CD 151
A

One of the following type of lasers has liquid media:

A. Argon

B. Carbone dioxide

C. Alexandrite

D. Ruby

E. Rhodamine dye
E

The plasma half-life 19-35 h is characteristic of the following antihistamine:

A. Hydroxyzine

B. Desloratadine

C. Loratadine

D. Levocetrizine

E. Fexofenadine
B

A 5-years old boy had Impetigo which was complicated by acute glomerulo-nephritis, which one
of the following is nephrotogenic strains of streptococcus? ‫جديدمهمم‬

A. 7

B. M-type 2

C. M-type 9

D. 21
E. 25
B

Elastin is not produced by one of the following:


a. Fibroblasts
b. Smooth muscle cells
c. Macrophages
d. Endothelial cells
e. Condroblasts
C

Which One of the following is not a Tick-borne disease? ‫جديد‬

A. Tularemia

B. Relapsing fever

C. Rickettsial fevers

D. Lyme disease

E. Trichinosis
5

One of the following is not correct about cytotoxic T- cells:

A. Are CD8+

B. Recognize their peptides in association with MHC class II molecules

C. Have at least 3 different pathways of killing.

D. Third pathway is mediated by cytokines IFN-alfa and gamma

E. Naïve CD8+ T-Cells can develop into Tc1 and Tc2 cells.
B

Regarding Passini’s corpuscle:

A. Possesses a characteristic capsule but no lamellar wrapping.

B. Capsule is arranged in 20 layers of cells

C. Slowly adapt to mechano-stimuli

D. Subcapsular zone contains collagen and fibroblasts

E. Capsule is wrapped in lamellar Schwan cells


B

Palmoplantar eccrine sweat glands are fully developed by:

A. End of the 1st trimester

B. During the 1st trimester

C. During the 3d trimester

D. During the 2d trimester

E. End of the 3d trimester


D

Which one of the following granules is seen in Botryomycosis discharge? ‫جديده غير مكرره‬

A. Sulfur granules

B. Red granules

C. Black granules

D. White granules

E. Blue granules
A
Which one of the following is NOT a predisposing factor to colonization and infection with
MRSA? ‫جديدة‬

A. Age (older than 65)

B. Prior antibiotic therapy

C. Recent hospitalization

D. Vaccination

E. Chronic illness.
D

One of the following IS NOT correct about Langerhans cell:

A. Murine Langerhans cells are derived from radio-resistant hematopoietic precursor cells.

B. The Formation of Langerhans cells depends on TGF-B2

C. After UVB irradiation the cells appear to repopulate from the hair follicule.
D. Langerin is not an exclusive marker for epidermal Langerhans cells.

E. Formation of Langerhans cells depends on MC-SF receptor ligands.


B👍

The embryonic epidermis is morphologically similar to adult skin by:

A. The mid third trimester

B. The mid first trimester

C. The second trimester

D. 8 week of EGA

E. 40 week of EGA
A

One of the following is not correct about The- 17 cells:

A. They are rapidly induced in response to infectious agents.


B. Transcription factor involved in Th- 17 cells development have hyper IgE syndrome
C. There is evidence that Th- 17 cells are involved in reheumatoid arthritis and psoriasis
D. IL 23 inhibits IL-17 production
E. Th- 17 cells are Produced by CD4 + T cells
4

One of the following receptor molecules is not expressed by APC:

A. CD8

B. B 7-2

C. CD 58

D. CD 265

E. CD 30
A

The infraorbital foramen is located: ‫مهمه غير مكرره‬

A. 5 cm below the infraorbital rim in the midpupillary line

B. 1 cm below the infraorbital rim in the midpupillary line

C. 2 cm below the infraorbital rim in the midpupillary line


D. 3 cm below the infraorbital rim in the midpupillary line

E. 4 cm below the infraorbital rim in the midpupillary line


B

The mechanism of action in Q- switched lasers is: ‫مهمه جديده غير مكرره‬

A. Tissue evaporation.

B. Vascular rupture.

C. Water boiling.

D. Photothermolysis

E. Melanocyte destruction.
D
One of the following is not a physiologic role of IL - 10:
K. Limiting the inflammatory response
L. Supporting humoral immunity
M. Preventing overwhelming immune response
N. Increasing macrophage Ag presenting
O. Upregulating Th2 immune pathway
4

One of the following stains blue by Giemsa:

A. Amyloid

B. Mast cell granules

C. Nuclei cell granules

D. Acid – fast organisma

E. Sulfared mucopolysaccharides
C

Psoriasis is characterizsd by keratinocyte hyperproliferation maintained by:

A. Genetically aberrant keratinocyte progenitor cells

B. Persistent stimulation of B- cell by immunogen

C. Accumulation of activated CD8+ and CD 4+ T-Cell in skin

D. Differentiation and expansion of type 2 T-Cell


E. Activated macrophage in the skin
C

SLE rate is highest in patient with a deficiency of: ‫مهم ركزي عليه غير مكرر‬

A. C3.

B. C1q

C. C5.

D. C2.

E. C9.
B

Which of the following has the longest plasma half-life?

A. Hydrocortisone.

B. Prednisolone.

C. Betamethasone.

D. Triamcinalone

E. Methylprednisolone.
C

Which nerve fibers within the dermis exhibit the most rapid response to local anesthesia?

A. Unmyelinated C-fibers carrying pain sensation

B. Intermediate fibers carrying sensations of heat and cold

C. A –type fibers that carry pressure sensation

D. Motor neurons that innervate the skeletal musculature

E. Autonomic nerves that innervate smooth muscle


A
Which of the following suture type has the most persistent tensile strength over time?

a. Braided polyester

b. Polygdioxanone
c. Polyglactin 910

d. Poliglecaprone

e. Silk
A
One of the following is not IL-10 familly:
A. IL-19
B. IL-20
C. IL-22
D. IL-23
E. IL-26
D

Which of the following medications have been shown to be efficacious in treatment facial
erythema of rosacea?

A. Topical alpha-antagonists

B. Topical alpha- 1 agonists

C. Topical alpha-2 agonists

D. Topical vit D analogs

E. Oral antimalarials
C
Which of the following is implicated in the pathogenesis of keloid formation:
G. IL – 1
H. IL – 2
I. IL – 6
J. IFN – gamma
K. TGF – beta
5

Tetracyclines have anti-inflammatory effects which include:


A. Increasing matrix metalloproteinase activity
B. Inhibit leukocyte chemotaxis
C. Increase production of TNF a
D. Increase production of IL 1B
E. Decrease production of IL 1B
2 makki

Viral capsid antigen (VCA) IgG is positive in acute infection with:

A. Herpes simplex

B. Condyloma acuminata.
C. Infectoius mononucleosis

D. Reseola infantum.

E. Rocky Mountain spotted fever


C

A consistent histopathologic feature of hydroxyurea dermopathy is:

A. Hydropic degeneration

B. Leukocytoclasis

C. Eosinophilic spongiosis

D. Pigment incontinence

E. Fibrinoid necrosis of vessel walls


A

One of the following is not correct about IL-4:


A. It switches B- cell to produce IgE
B. It is the key cytokine in driving Th-2 responses
C. It increases Th-1 and Th-17 difrerentiation
D. It seems to exert opposing effects on CD4+ cells
E. It can stimulate dendritic cells to produce IL-12
C

A 47-y-old female developed an acneiform facial eruption at the time of elective ambulatory
surgery when she was treated with pre and post-operative antibiotics. Occasionally, the patient
will develop painfull nodules as well. Topical tretinoin cream and doxycycline have not been
effective. Culture of a pustule reveals growth of serratia marcescens. What is the best next step
in the management of the patient?

A. Skin biopsy with tissue culture, as serratia marcescens is normal Skin flora

B. Switch to minocycline

C. Add benzoyl peroxide as it possesses antibacterial properities

D. Switch to metronidazole gel

E. Obtain baseline lab work, including a beta-HCG


E

Which of the following is true regarding Varicella?


A. Iifection of a pregnant woman from 5 days prior to 2 days after delivery can cause congenital
Varicella syndrome

B. An individual lesion will evolve from macule to papule to vesicle to crust over 5 days

C. The highest risk of congenital Varicella syndrome occurs within the first 20 weeks of
pregnancy

D. Varicella vaccine consists of heat killed virus

E. Disseminated herpes zoster is defined as 5 lesions outside an affected dermatome


C

Pentostatin (Nipent), a purine analogue is an ideal drug for:

A. Leprosy

B. Low-grade T-cell malignancies

C. Urticaria pigmentosa

D. Atopic eczema

E. Psoriasis
B

Under dermoscopy, a sebaceous keratosis has Which of the following feature?

A. Hairpin vessels

B. Milia – like cysts

C. Glomerular vessels

D. A and B

E. A, B and C
D

What is the 1st stage of carcinogenesis?

A. Induction

B. Progression

C. Initiation
D. Promotion

E. Conversion
C

Which of the following is a marker in B – cell lymphoma?

A. CD 5

B. CD 20

C. CD 25

D. CD 30

E. Non of the above


B

One of the following statements about p53 protein is false:

A. The wild-type protein has a half-life of 20 to 30 minutes.

B. Accumulation of p53 protein retards the formation of apoptosis.

C. p53 messenger RNA increases 10- to 20-fold late in the G1 phase.

D. The gene for p53 protein is on chromosome 17.

E. Immunohistochemical procedures detect mutated p53 gene product.


B

Pentostatin (Nipent), a purine analogue is an ideal drug for:

A. Leprosy

B. Low-grade T-cell malignancies

C. Urticaria pigmentosa

D. Atopic eczema

E. Psoriasis

Hydroxyurea works by inhibiting which enzyme?

A. Ribonucleotide reductase

B. DNA gyrase
C. Dihydrofolate reductase

D. Inosine monophosphate dehydrogenase

E. Thymidine kinase
A

Which of the following drugs is the safest option for pregnant patient with intense acne?

A. Dapsone gel

B. Sulfacetamide wash

C. Topical tretinoin

D. Erythromycin gel

E. Adapalene cream
D

Which of the following pairs of keratins is found in suprabasal layers of palmar and plantar skin?

A. Keratin 3 and 12

B. Keratin 5 and 14

C. Keratin 1 and 9

D. Keratin 1 and 10

E. Keratin 4 and 13
C

Which of the following is most likely to cause a decreased therapeutic response to


hydroxychloroquine?

A. A deficiency of thiopurine methyltransferase (TPMT)

B. Co-administration of erythromycin

C. Advanced liver disease

D. Smoking

E. Co-administration of antacids or iron tablets


D

Which of the following prompts cyclosporine- A dose adjustment?


A. Ciprofloxacin

B. Ketoconazole

C. Carbamazepine

D. Gemfibrozil

E. Furosemide
B makki

One of the followings is not a biologic effect of retinoids :


F. Inhibition of apoptosis
G. Inhibition of keratinization
H. Inhibition of tumor promotion and malignant cell growth
I. Differentiation and maintenance of epithelial tissue
J. Regular expression and activation of TLRs
1

Which of the following is NOT a diagnostical criteria for NF – 1?

A. Thinning of long bone cotices

B. Inguinal freckling

C. A maternal aunt with NF-1

D. Iris hamartomas

E. Plexiform neurofibromas
C

The drug which has the best coverage for impetigo in children is:
K. Penicillin
L. Erythromycin
M. Azithromycin
N. Clarithromycin
O. dicloxacillin
5

Which of the following proteins is a dimer?

A. Intermediate filaments

B. Collagen IV

C. Collagen I
D. Gap junctions

E. Elastic fibers
A

Nail changes in Pachyonychia congenita are most consistent with Which of the following
descriptions?

A. Congenital malalignment

B. Leukonychia

C. Koilonychias

D. Subungual hyperkeratosis

E. Onychomadesis
D

Which of the following stains is specific for lymphatic malformations:

A. CD34

B. LYVE- 1

C. VEGFR- 1

D. PECAM- 1

E. Collagen Type IV
B

What eye finding is most common in Marfan syndrome?

A. Cataracts

B. Keratoconus

C. Downward lens dislocation

D. Glaucoma

E. Ectopia lentis
E
Which of the following is not side effects of systemic steroids:
K. Hypokalemic alkalosis.
L. Hypocalcemia.
M. Lymphocytosis
N. Pseudotumour cerebri.
O. Exophthalmus.
3

In which of the following antibiotic, the site of action is DNA gyrase :


A. Vancomycin
B. Clindamycin
C. Sulfonamides
D. Quinolones
E. Trimethoprim
4

Which of the following is NOT an example of vascular malformation:


A. Pyogenic granuloma
B. Angiokeratoma
C. Telangiectasia
D. Arteriovenous fistula
E. Port – wine stain
A

Which of the following is NOT true:


A. Gene consists of DNA packed in chromosome
B. Skin fibropblast can be cultured in the laboratory
C. Chromosomes are present in all nucleated cells
D. In AD, usually only one generation is affected
E. In AR, both parents are carriers
4
Phenolic glycolipid-1 is important in identification of:
A. Mycobacterium Kanasasi.
B. Dermatophytes.
C. Moulds.
D. Mycobacterium Leprae
E. Pox virus.
D
Which of the following has the highest risk for gastrointestinal bleeding:
A. Aspirin
B. Ibuprofen
C. Diclofenac
D. Piroxicam
E. Indomethacin
D
Which of the following is NOT true about elliptical incision biopsy:
A. It is indicated when pathology is epidermal
B. The skin should be anesthesized
C. The scalpel blade no. 15 is used
D. It is closed using rule of halves
E. The length of incision is 3 times of the width
A
Dermo – epidermal junction is only stained by:
A. Hematoxiline Eosine
B. Alcian blue
C. Von kossa
D. PAS stain
E. Toluidine blue
D

The adhesion molecules responsible for intercellular adhesion relate to the family of:
A. Cadherins
B. Selectin P
C. Selectin E
D. Integrins
E. Super gamma globulins
A

Which of the following is used to treat pubic lice infestation of eyelashes:


A. Septrin
B. Ivermectin
C. Petrolatum
D. Pyrethin
E. Malathion
C

Nocireceptors repond to:


A. Temperature changes
B. Harmful stimuli
C. Chemical solutions
D. Mechanical forces
E. Light
B

The epidermis begins to develop at:


A. 4th week of intrauterine life
B. 7th week of intrauterine life
C. 14th week of intrauterine life
D. 10th week of intrauterine life
E. 22nd week of intrauterine life
B

Which one of the following antifungal activity is dependant on cytochrome 450 :?


a. ketoconazole
b. terbinafine
c. amphotericine
d. fluconazole
e. griseofulvin
A
The basic molecular unit of elastin is:
F. Oxytatan
G. Eluanin
H. Tropoelastin
I. Desmosines
J. Profilaggrin
3

Which of the following is incorrect?


A. The dermis the main bulk of skin
B. The dermis protects the body from mechanical injury
C. Oxytalan fibers are found primarily in The reticular dermis
D. Collagen forms 75% of the dry weight of The skin
E. Majority of Collagen is type – I
C
In a female patient presenting with acne which of the following indicates to the testing for
hyperandrogenism ?
A. Premenstrual flare
B. Submarine comedones
C. Pitted scars in the checks
D. Location of lesions in the trunk
E. Sudden onset of severe lesions
E
Which of the following is NOT ectoderm derivative:
A. Sebaceous gland
B. Blood vessel
C. Nerves
D. Nails
E. Epidermis
B
Which of the following DERMATOPHYTES is NOT anthropophylic:
A. M. auduinii
B. T. rubrum
C. T. violaceum
D. M. gypseum
E. Epidermophyton floccosum
D
Which of the following is feature of rosacea?
A. Scale.
B. Seborrhea.
C. Spongiosis
D. Carpet tack sign.
E. Peau d’orange skin.
C

In which of the following causes of pruritus there is dysregulation of central opiod peptides:
A. Urticarial
B. Scabies
C. Cholestasis
D. Small fibre neuropathy
E. Delusional parasitosis
C

Combining tetracycline and isotretinoine for the treatment of the severe acne should be avoided
because of Which of the following ?
A. Multiple exuberant bone growth
B. Intracranial hypertension
C. Severe chilostomatitis
D. Nasal bleeding
E. Myalgia and arthralgia
B

The suffix ximab in the nomenclature of biological agents indicates to What of the following ?
A. Blocker of interaction between interleukin- 1 and its receptor
B. Chimeric monoclonal antibody
C. Humanized monoclonal antibody
D. Human monoclonal antibody
E. Receptor protein Blocker
B

For a patient of chancre and allergy to penicillin, What is the alternative therapy ?
A. Metronidazole 2 g single dose
B. Terbinafine 250 mg x 2 daily for 1 month
C. Tetracycline 500 mg x 4 daily for 1 week
D. Erythromycin 500 mg x 4 daily for 2 weeks
E. Clindamycin 150 mg x 4 daily for 3 days
D

Itching migrating in distribution and time and without lesions is most likely secondary to ?
A. Aging
B. Hepatic disease
C. Renal disease
D. Parasitophobia
E. Internal malignancy
D

Newly produced keratinocytes move outside the basal layer in reponse to:
A. Down-regulation of integrins
B. Ca + influx
C. Osmotic pressure
D. Crowdedness of cells
E. Upregulation of epidermal growth factor α
A
Hutchinson nail sign is periungual extension of pigment to the proximal and lateral folds of the
nail and is noted in:
A. LP
B. Traumatic purpura
C. Onychomycosis
D. Subungual melanoma
E. Periungual fibroma
D
The epidermis is a dynamic tissue in which cells are in constant motion. The kinetics of this
motion is characterized by:
A. Direction and speed are synchronous
B. Keratinocytes pass each other but not melanocytes
C. Movement is unidirectional
D. Stability for Direction and flow is provided by the basal membrane complex
E. Lateral motion is slower than forward
D

Keratohyaline granules:
A. Are acidic in nature
B. Composed primarily of keratin
C. Release filagrin
D. Filagrin is degraded to acid metabolites in the granular layer
E. Metabolites of filagrin help filter UV radiation
E

Which of these statement regarding the molecular basis of ichtyosis is correct:


A. X-linked ichtyosis is associated with complete deletion of steroid sulphatase gene
B. In lamellar ichtyosis there is mutations in keratin 1-10 genes
C. In epidermolytic hyperkeratosis mutations occur in multiple loci involving keratin and
transglutamase genes
D. In ichtyosis vulgaris mutations occur in the genes coding for products of L-granules of the
granular layer
E. Congenital ichtyosiform erythroderma is associated with mutations in genes coding for
transglutamase I and 12 R lipooxygenase
E
Which one of the following drugs cause chronic myelogenous leukemia :
a. Imatinib
b. Interferon
c. Hydroxyurea
d. Busulfan
e. Dapsone
D

The arrector pili muscle:


A. Develop from a bulb on the lateral side of hair canal
B. Function as a support for sebaceous gland
C. Participate in thermoregulation
D. Contracts in response to raised temperature
E. Is attached to basement membrane
C

The papillary dermis is characterized by:


A. Large bundles of small diameter collagen fibers
B. Elaunin elastic fibers
C. Low density of fibroblasts
D. Capillaries from superior plexus project in couples into the papilla
E. Exchange cytokines and growth factors with the epidermis
E

Chancroid is a sexually transmitted infection. It manifeste with:


A. Clean superficial erosions
B. Discrete inguinal lymphadenopathy
C. shallow clean ulcer
D. deep dirty ulcer
E. urethral discharge
D

The following fungal infections of the skin are due to dermatophytes except:
A. Favus
B. Kerion
C. Piedra
D. Tinea pedis
E. Tinea corporis
C

Glucagons :
A. Secretion is inhibited by GABA
B. Is the only hyperglycemic hormone
C. Cause lipid anabolism
D. Cause increased contactibility of cardiac muscle
E. Secretion is stimulated by the hormone secretin
A

Griseofulvin reaches te outermost layer of the skin by:


L. Eccrine sweat
M. Sebum
N. Lymphatic system
O. circulatory system
P. diffusion
1

Which of these molecules plays a major role in adhesion of basal cell extracellular matrix:
A. Fibronectin
B. Perlecan
C. Envoplakin
D. Desmoglein
E. Heparin
A
Involucrin:
A. Is a gluten rich protein found in squamous epithelium
B. Its synthesis occurs after the synthesis of cornifinand loricrin
C. Hypothised to play an organizing role in the epidermis
D. Its synthesis is controlled by Ca
E. Expression is affected by retinoic acid
D
Profilagrin:
A. Is a large calcium binding Glycoproteins
B. Its gene has been mapped 1q21
C. Consists of 15 – 20 repeating copies of filagrin
D. Its terminal domain contains calcium binding region used to assembly the molecule
E. Undergoes hydrolysis to release hydrogen
B
Nidogen belongs to the family of:
A. Glycoproteins
B. Glycosoaminoglycans
C. Proteoglycans
D. Collagen
E. Glycolipids
A
Select the incorrect statement about the remoudeling of keratinocytes into corneal layer:
A. Keratinocytes become less permeable to Ca
B. Profilagrin is processed into filagrin
C. Keratohyaline granules are broken down
D. Glutaminazes are activated
E. Filagrin is degraded into amino acids
A

Different epidermal products appear at different stages of embryogenesis:


A. Desmosomal proteins at week 10
B. Basal keratins at week 10
C. Filagrin at week 10
D. Earliest hairs by week 12
E. Nails at week 25
1

Profilagrin is evident in
A. Stratum corneum
B. Stratum granulosum
C. Stratum lucidum
D. Stratum basale
E. Stratum spinosum
B

Chancroid is a sexally transmitted infection. It manifeste with:


A. Clean superficial erosions
B. Discrete inguinal lymphadenopathy
C. shallow clean ulcer
D. deep dirty ulcer
E. urethral discharge
D

The structural base of the UV protection function of strateum corneum is :


A. Corneodesmosome
B. Lamellar bilayers
C. Corneocyte and extracellular matrix
D. Corneocyte Cytosol
E. Cornified Envelope
C

The skin of a 6 weeks embryo is made of:


A. A single layer of glycogen filled cells
B. 2 layer of cells
C. A single layer known as stratum germinatum
D. 3 layers including A middle layer filled with glycogen
E. Few microvilli are present at the outer surface
B
Hair medulla expresses keratin:
A. K32
B. K35
C. K37
D. K82
E. K85
3

Neural crest cells in the skin:


A. Migrate as melanocytes into the epidermis
B. Contain one type of pigment
C. Their pigment is secreted in vesicular structure
D. The shape of the melanocytes is cuboidal
E. The pigment Produced is transferred to adjacent keratinocytes directly by osmosis
C

Staphylococcal scarlet fever differs from Streptococcal scarlet fever in:


A. Exanthem is less tender
B. Absence of pharyngitis
C. Systemic symptom is less frequent
D. Exanthem is more wide spread
E. Regarded as severe form of SSSS
B
Staphylococcus aureus is protected from oxidative killing of neutrophils by:
A. Staphylokinase
B. Aureolysin A
C. Carotenoids
D. Oat A gene
E. Surface plasmin
C
One of the following site is not suitable for colonization of Staph aureus in neonates:
A. Throat
B. Umbilicus
C. Circumcision site
D. Conjunctiva
E. Skin
A
One of the following is a UVA sun blocker:
A. Padimate
B. Meradimate
C. Octiroxate
D. Trolamine salicylate
E. Ensulizole
B
One of the following antihistamine is used in a full dose in patients with chronic renal failure:
A. Acrevastine
B. Misolastine
C. Citrizine
D. Desloratidine
E. Fexofenadine
2
One of the following is induced by UV radiation:
A. IL 1
B. IL 2
C. IL 4
D. IL 6
E. IL 8
A
The only feature that discriminates Langerhans cells from other cells is:
A. CD1
B. CD207
C. HLA DR
D. CD 39
E. MHC-II
B
Retinoids could cause hyperlipidemia by:
A. Prevent the uptake of lipid from chylomicrons into cells
B. Prevent the uptake of lipid from LDL into cells
C. Decrease the production of HDL
D. Increase the expression of apolipoprotein C3
E. Increase the production of Cholestrol
D

One of the following is not correct regarding skin homing of memory T cells:
A. It depends on interaction of CCL 17 with CCR4
B. It depends on interaction of CCL 22 with CCR4
C. It depends on interaction of CCL 27 with CCR8
D. CCL 17 is synthesized by activated keratinocyte
E. CCL22 is produced by macrophage
C

One of the following is not correct about the markers of NK cells:


A. CD 56+
B. CD 16+
C. CD 154+
D. CD 94+
E. CD 161+
C

Natural killer cells usually does not exihibit One of the following:
A. CD+ 16
B. CD+ 56
C. CD+ 86
D. CD+ 94
E. CD+ 161
C

Where androgen receptors are predominantly expressed?


K. Isthmus.
L. Dermal papilla
M. Stratum basale.
N. Papillary dermis.
O. Periglandulare dermis.
2

Complement receptor 1 ( CR1, CD35 ) is NOT expressed on one of the following :


A. Neutrophils
B. Macrophages
C. T cells
D. B cells
E. Follicular dendretic cells
E

One of the following is a less cause of erythroderma in adults:


A. Psoriasis
B. Atopic dermatitis
C. PRP
D. Drug reaction
E. Idiopathic
C
Which ftide is inducible by component of microbes and proinflammatory cytokines:?
A. HBD 1
B. HBD 2
C. HBD 3
D. LL 37
E. Dermcidin
2 Fitzpatrick

Which of the following TLR is activated by triacylated lipoprotein:


A. TLR 2/6
B. TLR 2/1
C. TLR 3
D. TLR 9
E. TLR 8
B
The keratinocyte is protected from attack by complement by:
A. CD 35
B. CD 21
C. CD 88
D. CD 55
E. CD 59
E

In Buschke – Loewenstein tumour one statement is TRUE:


F. Tends to appear in circumcised men.
G. Is said to represent 80% of 90% of all penile cancers.
H. Does not seem to show a preference for either circumcised or uncircumcised men.
I. Is said to represent 5% to 26% of all penile cancers
J. Radiotherapy is the statement of choice.
D

One of the following is not correct about contraindication for acitrein treatment of psoriasis:
A. Severe liver dysfunction
B. Poorly controlled Diabetus Millitus
C. History of pancreatitis
D. Unreliable patients
E. Combined treatment with methotrexate
E
One of the following plants cause urticarial dermatitis:
A. Dumb cane
B. Garlic
C. Celery
D. Stinging nettle
E. Chrysathemums
D
One of the following is an uncommon cause of drug-induced acne:
A. Danazol
B. Lithium
C. Phenytoin
D. Azathioprine
E. Bromides
D
One of the following is polyenes antifungal drugs:
A. Ketoconazole
B. Itraconazole
C. Terbinafine
D. Amphotericine B
E. Tolnaftate
D
Polyenes=ampho nysta
Allylamines=turbinafine naftifine
Anti dna=floucytocine

The cytokine which predominates in chronic atopic dermatitis is :


A. IL- 12
B. IL- 13
C. IL- 4
D. IL- 5
E. Th 1 cytokines
E
One of the following is not direct mucosal infection:
A. Salpingitis
B. Urethritis
C. Cervicitis
D. Proctitis
E. Pharyngitis
A

One of the following is not correct about chronic pain:


A. Peripheral sensation of C Nerve fibers
B. Central sensation are allokness, punctuate hyperknesis
C. Neuromediator is neurotropin 4
D. Chemical mediator is bradykinin
E. CNS areas activated is anterior cingulate cortex
B
One of the following is not immunocompromised host diseases associated with CMV infection:
A. Cutaneous Vasculitis
B. Ulcers
C. Verrucous plaques
D. Nodules and hyperpigmented plaques
E. Erythema nodosum
E

Which of the following is not a complication of anterior urethritis:


A. Gonnococcal Balanitis👍
B. Tysonitis👍 rare
C. Epididymitis 👍
D. Littritis 👍
E. Periurethral abscess👍
B

Apocrine sweat gland is not innervated by:?


A. Nerve fibers near gland
B. Cholinergic muscarinic
C. α 1- adrenergic receptors
D. B2->B3 adrenergic receptors
E. Purinergic receptors
C

One of the following is not correct about cat scratch disease:


A. It is caused by infection with bartonella henselae
B. It is the most common cause of persistent lymphadenopathy in children
C. There is no systemic manifestation of it
D. Skin lesions are red papule or crusted pustule
E. 2 line treatment azithromycin
C
One of the following is not correct about C1 esterase inhibitor:
A. Types I,II,III are Types of hereditary angioedema
B. Deficiency of C1 esterase inhibitor lead to loss of inhibition of Hageman factor
C. Low level of C4 in the serum, between and during attacks
D. Acquired Deficiency of C1 esterase inhibitor result from low level activation of C1q
E. Angiotensin-converting enzyme (ACE) inhibitor-induced urticarial result from activation of
endogenous kininase II
5

Which of the following is false for reticular dermis:


A. Coarse elastic fibers
B. Few fibroblast
C. Few blood vessels
D. Abundant ground substance
E. Thin collagen fibers
E

Which of the following DERMATOPHYTES is NOT anthropophylic:


A. M. auduinii
B. T. rubrum
C. T. violaceum
D. M. gypseum
E. Epidermophyton floccosum
D

One of the following is not correct about IL-4:


A. It switches B- cell to produce IgE
B. It is the key cytokine in driving Th-2 responses
C. It increases Th-1 and Th-17 difrerentiation
D. It seems to exert opposing effects on CD4+ cells
E. It can stimulate dendritic cells to produce IL-1
C

One of the following is NOT side effect of fumerate:


A. Renal impairement
B. Flushing
C. Headache
D. Fatigue
E. Hepatotoxicity
E

One of the following systemic drugs is not effective for treatment of morphea and lichen
sclerosis:
A. Penicillamine
B. Methotrexate
C. Cyclosporine
D. Vit A analogues
E. Corticosteroids
C

One of the following drugs is not direct-acting cholinomimetic agents that can stimulate eccrine
sweeting:
A. Acetylcholine
B. Cerimelline
C. Pilocarpine
D. Methacholine
E. Donepezil
5

One of the following is not correct about properities of transit amplifying cells:
A. Self-renewal, limited
B. Potential for differentiation limited
C. Cycling in normal epidermis slow
D. Proliferative Potential limited
E. Maintenance of tissue homeostasis limited
C

One of the following phenotypic markers is not of human langerin-dermal dendritic cells:
A. MHC class II
B. CD1a
C. CD11b
D. Langerin (CD 207)
E. CCR7
D

Antigen presenting cell (APC) IS NOT associated with induction of:


A. IL-1 B
B. IL-10
C. IL-12
D. IL-6
E. Chemokines
B

The most common occupation relative risk of developing contact urticarial:


A. Bakers
B. Butchers
C. Medical personal
D. Paints
E. Nurses
A

When a genodermatosis is said to have an incomplete degree of penetrance, what does that
mean?
A. That only females can be affected with few exceptions
B. That only males can be affected with few exceptions
C. That not all individuals who carry the disease genotype will manifeste the disease
D. That the disease can show variation in the severity of the findings that develop
E. That the inactivation of one of the X in women is responsible for dose compensation
C

Pimecrolimus functions by Which of the following ?


A. Binds TNF-α
B. Binds High –affinity IL-2 receptor on T cells
C. Binds CTLA-4
D. Forms complex with FK506 binding protein
E. Forms complex with cyclophilin
D

Which of the following is not a major feature of Tuberus Sclerosis ?


A. Gingival fibroma
B. Periungual fibroma
C. Hypomelanotic macules
D. Facial angiofibroma
E. Lymphangioleiomyomatosis
A

One of the following is mesodermal lineage:


A. Keratinocytes
B. Melanocytes
C. Merckel cells
D. Neurons
E. Langerhans cells
E

A 65 year old male presents with new onset skin yellowing and blue-black blotching on his
gums. Which of the following is most likely to cause the symptoms listed?
A. Amiodarone
B. Quinacrine
C. Clofazimine
D. Minocycline
E. Mercury
B

The corneal layer role in protection against UV radiation is fulfilled through:


A. Reflection of light from the surface
B. Light scattering
C. Reflection from the surface and filtring through acid metabolites present in the corneal layer
D. Absorption of light through keratin
E. Refraction
C
Interferon is not indicated in Which of the following disease:
A. Malignant melanoma
B. MF
C. AIDS related Kaposi sarcoma
D. Condyloma accuminata
E. Hepatitis B
E

Interferon is not indicated in Which of the following disease:


A. Malignant melanoma
B. MF
C. AIDS related Kaposi sarcoma
D. Condyloma accuminata
E. Hepatitis B

Hair follicule are arranged in groups at an equal distance of each other. Each group contains a
number of follicles that equals:
A. 3
B. 10
C. 15
D. 25
E. 50
A

One of the following is not apportunistic:


A. Candidiasis
B. Aspergillosis
C. Blastomycosis
D. Cryptoccosis
E. Trichosporum species
C
Table 77-32

In chicken pox, Which of the following is not true:


A. It spread by aerosol route
B. It is highly infectious
C. The incubation period is 14 – 21 days
D. In adults, the disease is less severe than children
E. Self – limiting cerebellar ataxia may rarely occur
D

One of the following is not true about treponema pallidum:


A. Generally shows 8 – 12 spiral convulsions
B. Stained by silver stain
C. Septrin sensitive
D. Protected from phagocytosis by polysaccharide capsule
E. It does not produce exotoxin
D

WHICH immunoglobulin is present on the cell surface of most circulating B cells:


A. IgA
B. IgG
C. IgD
D. IgE
E. C5
C

Concerning mycological examination, which of the following is false:


A. For black piedra the hair is simply cut above the skin level
B. For dermatophytes the hair should be plucked out with intact root
C. In tinea incognito, villus hairs examination may be the easiest method
D. The hyphae at the distal end of the nail are more likely to be viable
E. Stripping can be used in taking scales in partially treated pityriasis versicolor.
D

Which of the following is not a sign of diabetic ketoacidosis:


A. Dehydration
B. Bradycardia
C. Hypotension
D. Smell of acetone
E. Kussmaul breathing
B
Which of the following is false about chromosomes:
A. The karyotype is the individual number of chromosomes
B. Blue eyes is an exemple of phenotype
C. Chromosomal disorders have high risk to relatives
D. Trisomy is an extra chromosome number defect
E. Monosomy fetuses are usually non viable
E

Which of the following is false about delusion of parasitosis:


A. The typical patient is a middle aged or elderly man
B. It is not uncommon for the delusion to be shared by other family members
C. Patient have been showen to respond to the neuroleptic drug pimzide
D. Patient may engage in self mutilation
E. Patient may exhibit the “match-box” sign
A

What is the most commonly mood stabilizer used in hypomania:


A. Lithium
B. Benzodiazepines
C. ECT
D. Antipsychotics
E. Phenothiazines
A

Which of the following about nails is correct:


A. Fingernails grow at about 0.5mm/month
B. Toenail grow faster than fingernails
C. After avulsion toenails return to former length in 6 months
D. Nails grow faster in winter than summer
E. Nails growth is slower in adults than in children
E

What process takes place in the granular layer when it passe into the corneal:
A. Apoptosis
B. Lysis of all organelles.
C. Lysis of organelles except keratin
D. Lysis of all intracellular structure except keratin and fillagrin
E. Deposition of new keratin in the cytoplasm
D
Fitz

Tin- tack sign is observed in :


a. SLE
b. DLE
c. Sarcoidosis
d. Psoriasis
e. Darier
B
Which one is not correct about wet ( rural ) type of cutaneous leishmania :
a. Incubation period ranges from few weeks to 2 months
b. Ulcerates rapidly
c. Self healing take 12 months
d. Appears as red furuncle like nodule
e. Ulcer with raised red margin is observed
C

Chronic HBV infection is marked by:


A. Incubation period
B. HBs Ag
C. Anti –HBc (IgM)
D. Anti- HBs
E. Anti –HBc (IgG)
E

In type -1 diabetes mellitus, One of the following is not correct:


A. The age of onset is <40y
B. Ketonuria is positive
C. Family history is uncommon
D. The patient is usually obese
E. Non of the above
D

Which of the following is false for reticular dermis:


A. Coarse elastic fibers
B. Few fibroblast
C. Few blood vessels
D. Abundant ground substance
E. Thin collagen fibers
E

Histologically, which of the following is not a feature in the palms:


A. Thick cornified layer
B. Absence of hair follicule
C. Flat rete ridges and papillae
D. Many nerve endings
E. Numerous eccrine sweat glands
C

One of the following is not an oxygen-dependant killing mechanism in Neutrophils:


A. Lysozyme
B. Lactoferrin
C. Major basic proteins
D. Cationic proteins
E. Hydrogen peroxidase
C

One of the followings is not a feature of leukocyte adhesion with transmigration:


A. Migration of leukocytes as a result of stasis of blood flow in the microvasculature.
B. Interaction of integrins with immunoglobulin found on endothelial cells.
C. The selectins found only on endothelial cells.
D. Icam-1 and vcam-1 adhesive molecules on the endothelial cells.
E. Neutrophils in the first 6-24 hours of acute inflammation.
C

.Which one of the following cytokines is a macrophage activator:


A. IFN Y.
B. IL-2
C. IL-4.
D. TNF-alpha.
E. IL-12.
A

Newly produced keratinocytes move outside the basal layer in reponse to:
A. Down-regulation of integrins
B. Ca + influx
C. Osmotic pressure
D. Crowdedness of cells
E. Upregulation of epidermal growth factor α
A

“Once a keratinocyte leaves the basal cell layer, the normal transit time to stratum corneum is at
least:
1. 7 days
2. 14 days
3. 21 days
4. 28 days
5. 35 days
2

The epidermis is a dynamic tissue in which cells are in constant motion. The kinetics of this
motion is characterized by:
A. Direction and speed are synchronous
B. Keratinocytes pass each other but not melanocytes
C. Movement is unidirectional
D. Stability for Direction and flow is provided by the basal membrane comples
E. Lateral motion is slower than forward
D

Very common loss of function mutations in the fillagrin gene causes all of the followings
EXCEPT:
A. Genetic disorder of Ichthyosis vuglaris.
B. Risk factor for the development of atopic dermatitis.
C. Atopic asthma.
D. Contact dermatitis
E. Systemic allergies.

Fillagrin is first detectable in:


A. 8 weeks of fetal life
B. 12 week of fetal life
C. 20 weeks of fetal life
D. 15 weeks of fetal life
E. 10 weeks of fetal life
D

Premature babies are not prone to one of the following:


A. Iron deficiency anemia.
B. Neonatal jaundice.
C. Hyaline membrane diseas.
D. Birth trauma.
E. Rickets.
##‫تاكدي منها‬

One of the following is not correct about cytotoxic T- cells:


A. Are CD8+
B. Recognize their peptides in association with MHC class II molecules
C. Have at least 3 different pathways of killing.
D. Third pathway is mediated by cytokines IFN-alfa and gamma
E. Naïve CD8+ T-Cells can develop into Tc1 and Tc2 cells.
B

In laser treatment, the shortest thermal relaxation time is used in:


A. Tattoo
B. Melanosomes.
C. Port-wine stain (PWS) vessels.
D. Terminal hair follicle.
E. Leg veins.
A

Which of the following is false regarding impetigo?


A. Caused by superficial gram-positive organisms.
B. Pediculosis capitis is a risk factor for development of this condition.
C. These infections may be followed are sometimes followed by acute glomerulonephritis.
D. Topical antibiotic monotherapy is appropriate.
E. Topical polymyxin would be an effective therapy.
5

In acute viral hepatitis, one of the following is not true:


A. Negative antibody test exludes acute infection.
B. Variable incubation periods.
C. Can be transmitted by several routes.
D. Sometimes a selflimiting disease.
E. Liver cirrhosis is a complication.
A
Which of the following is true about hepatitis B virus infection:
F. Vertical transmission is rare.
G. Only 10% of infected persons have spontaneous remission.
H. Hepatitis vaccination should be used after acute infection.
I. The presence of hepatitis e antigen indicates high infectivity.
J. It can lead to development of hepatocellular carcinoma.
4

One of the following is not true about lepromatous leprosy:


A. Numerous lesions are present macules, papules, nodules.
B. Asymmetrical distribution.
C. Vague border and difficult to distinguish between normal and affected skin.
D. Sensation is not affected.
E. Many bacilli could be found in skin lesion.
B

One of the following IS NOT characterized of tuberculoid leprosy:


A. Hypohydrotic.
B. Anesthetic.
C. Atrophic
D. Asymmetrical.
E. < 5 lesions.
C

Regarding the cellular mechanisms of vascular leakage, which of the following is INCORRECT:
A. The ‘immediate transient response’ Only Occurs in Venules of 20-6- µm Diameter.
B. Sunburn Can Aause a Delayed and Prolonged Vascular Leakage.
C. The Endothelial Injury Caused By Leukocytes is Due to Free Radical-induced Damage.
D. The Cytoskeletal and Junctional Retraction Mechanism is Mediated by The Complement
System.
E. The Most Common Mechanism For Increased Vascular Permeability is “Endothelial
Contraction” (“immediate transient response”).
E

Which one is not correct about IgG:


a. It is the most abundant Ig.
b. IgG is the major Ig of the innate immune response
c. IgG1 and IgG3 are potent activators of the classical complement pathway
D. IgG2 is less effective.
E. Most of the autoimmune dermatoses which are caused by autoantibodies are mediated by
IgG, most offten IgG4.
2

. which one is not true with respect to IgG antibodies


• they mediate allergic reaction
• they are glycoproteins
• they predominate in the primary response to antigen
• they occur on the surface of lymphocytes
• they fix complement
3

The KERATINOCYTE is protected from attack by complement by:


A. CD 35
B. CD 21
C. CD 88
D. CD 55
E. CD 59
5

One of the following chemical mediators play a role in stimulation of fibroblasts proliferation?
A. Kinins
B. Serotonin
C. Histamine
D. Prostaglandins
E. Complement
2 👍 table 141-1

- One of the following statements is not true:

A. γ chain is a heavy chain of IgG.


B. At the age of one year, the highest immunoglobulin is IgM.
C. IgG4 does not fix the complement.
D. IgA1 is produced by lymphocyte in the respiratory and GIT.
E. IL-4 foster IgE production.
D

IgA2 secretory (gut respirating)


IgA1 serum

One of the following is not a feature of IgG:

A. IT is the major immunoglobulin


B. It has 2 heavy gamma chains
C. Most isotypes fix complement
D. It gives long-lasting immunity
E. IgG4 is decreased in atopic dermatitis
E

which one is true regarding IgM :


a. cross the placenta
b. produced in a secondary immune response
c. found as a diametric form linked by a J chain
d. can activate complement
e. usually found on the surface of mast cells
4

. Which of the following is part of the alternative complement cascade system:

A. C 1
B. C1 INH
C. C3
D. C5b
E. anaphylatoxin
C

EBV :
A. Approximately 75% of young adults with primary infection develop infectious mononucleousis
B. EBV is commonly associated with B-cell lymphoma
C. The virus enters cell by specifically binding to cell surface complement CR4
D. The humoral immune response, although it generates Ab to a variety of EBV Ag , appear to
be protective
E. Splenomegaly is found in >50% of patients and can be accompanied by hepatomegaly.
E

The following is used to detect antibodies:


A. PCR
B. Complement fixation
C. Haemagglutination fixation
D. Coagulase test
E. ELISA
5

.A test which detects the presence of a multiple number of antigens is:


A. Complement Fixation Test.
B. Passive Latex Agglutination.
C. Direct Fluorescent Antibody Test.
D. Indirect fluorescent antibody Test.
E. Immuno-electrophoresis.
5

The classical and alternative pathway meet at complement component:


A. c3
B. c4
C. c4b
D. c5
E. factor d
A

A complement component which is strongly chemotactic for neutrophils is:


A. C3
B. C3b
C. C5a
D. C5b
E. C9
C

“Which complement component is composed of three separate proteins:


A. C1
B. C2
C. C3
D. C4
E. C5
A

Activation of C5a results in:


A. Opsonization
B. Increased inflammation
C. Endotoxin shock
D. Activation of interferon
E. Activation of NK cells
B

The cytotoxic T-Lymphocytes s kill target cells by:


A. Antibody and complement
B. Generation of reactive oxygen species
C. Activation of monocytes
D. Liberation of perforin and Activation of apoptosis
E. TNF- alpha
D

Complement receptor 1 ( CR1, CD35 ) IS NOT EXPRESSED ON ONE OF THE FOLLOWING :


A. NEUTROPHILS
B. MACROPHAGE
C. T cells
D. B cells
E. Follicular dendretic cells
E

The most sensitive marker for melanoma is:


A. S100.
B. HMB45.
C. Tyrosinase.
D. MART 1/ Melanin A.
E. P16.
A

. Which one of the following laboratory tests is not used for diagnosis of primary syphilis :

A. Dark field microscopic examination


B. VDRL
C. TPHA
D. FTA – ABS 19S-IgM
E. FTA – ABS
B

CD207 is a sensitive and specific marker of :


a. Helper T cell
b. Cytotoxic cell
c. Merkel cell
d. Langerhans cell
e. Vascular endothelial cell
D

One of the following inherited photosensitivity disorders has an action spectrum within UVB-UVA
II range:
A. Bloom syndrome.
B. Trichothiodystrophy.
C. Cockayne syndrome.
D. Xeroderma pigmentosa
E. UV-sensitive syndrome
D

One of the following is not true about PCR:


A. PCR is simple and rapid.
B. It is low sensitive in amplifying low amounts of DNA
C. The total number of PCR products after n cycles
D. PCR can be used to label DNA with radioactive
E. PCR can be used for rapid haplotype analysis
2
Extremely sensitive

Krause end bulbs are:


1. Adapting mechanoreceptors found on weight-bearing sites that respond to vibrational stimul
2. Found on the vermillion border of the lips
3. Located in the dermal papillae of digital skin
4. Located in the deep dermis and within the subcutis in weight-bearing sites of the body
5. Found at the orifice of the hair follicle and particularly sensitive to cold
B

One of these structure is not a regular component of the papillary dermis:


A. CAPILLARY
B. Venules
C. Blind ended lymphatics
D. Elastic fine fibres
E. Reticulin fibres
E

The most predominant vessels within the upper dermis are:

A. Superficial arteriolar plexus.


B. Terminal arterioles .
C. Arterial capillaries .
D. Venular capillaries.
E. Post capillary venules
E

The perception of the color of the skin is related to which wavelength region of the following?
A. 200-290 nm.
B. 290-320 nm.
C. 320-400 nm.
D. 400-700 nm.
E. 700-1200 nm.
D

In infectious mononucleosis, One of the following is not correct:


A. It is caused by Epstein Barr virus
B. Nasal secretion is the main source of infection
C. It is not highly contagious
D. Heterophil antibody is detected by monospot test
E. Amoxicillin should be avoided
C

About scabies , Which one of the following is not correct:


a. It is caused by highly host - specific eight- legged mite
b. Mites from animals are not a source of human infestations
c. It is too small to be seen by the naked eye
d. A female mite will lay 6 – 10 eggs....3 only
e. Eggs require 10 days to mature
D

The component of the least percentage in composition of sebum is:


A. Wax esters
B. Cholesterol
C. Cholesterol esters
D. Glycerides
E. Squalene
B

One of the following is not true about anti-infammatory effects of glucocorticoids:


A. Increase eosinophils apoptosis and decrease Neutrophils apoptosis
B. Decrease production of ICAM-1 and E-selectin
C. Decrease production, differentiation and antigen presentation of monocytes/macrophage
D. Increase regulartory T-cell function
E. Redistribute B cells to lymphoid deposits resulting in transient lymphopenia
E

In infectious mononucleosis, One of the following is not correct:


A. It is caused by Epstein Barr virus
B. Nasal secretion is the main source of infection
C. It is not highly contagious
D. Heterophil antibody is detected by monospot test
E. Amoxicillin should be avoided
C

Neutrophil polymorphs:
A. Have multilobed nuclei
B. Are the predominant cell type in chronic inflammation
C. May fuse to form multinucleate giant cells
D. Have phagocytic abilities
E. Have numerous eosinophilic granules in their cytoplasm
A

Antigenic competition:
A. Involves competition between T-cell epitopes for the MHC groove
B. Involves competition for available soluble antibodies
C. Is unrelated to the concept of dominant and subdominant epitopes
D. Can only occur with cryptic epitopes
E. Refers to the differential immunogenicity of the carbohydrate and protein moieties in a
glycoprotein
A

Proliferation of activated T-cells:


A. Is stimulated by a single signal induced by engagement of the T-cells receptor
B. Requires both the signal described in A plus costimulation from B7
C. Requires both the 2 signals described in B plus interaction between LFA-1 and ICAM-1
D. Requires only mutual binding of LFA-3 and CD2 on the antigen-presenting cell and T-cell
respectively
E. Is unaffected by anergy
2

One of the following is characteristic of B-cells:


A. MHC class I
B. CD3
C. Measles virus receptor
D. Polyclonal activation by concanavalin A
E. Surface immunoglobin
5

The CD4:
A. Is essentially an intracellular glycoprotein
B. Is heterodimeric
C. Binds processed peptide in its outer groove
D. Binds to MHC class II on antigen-presenting cells
E. Is highly polymorphic
4

A patient with UP, to determine the current mast cell burden which of the following labs would
you check ?
K. Serum tryptase
L. Urine tryptase
M. CBC with differential.
N. Alk. phos
O. None of the above.
A

The most frequent cause of PUVA-induced phototoxicity is:


A. Therapy on consecutive days
B. The dose of UVA was too high
C. UVB was given in addition to UVA
D. Concomitant medication
E. Concurrent disease
D

The cornefied envelope is:


K. A 15 – 20 nm insoluable layer of protein
L. Attached to the inner side of plasma membrane through corneosomes
M. Overlaid by a 5 nm lipid envelope
N. Its precursors are synthesized in the basal layer
O. Its Construction is carried out in 2 steps
3

Which of the following is not a major interaction between laser light and the skin :
K. absorption.
L. Reflection .
M. Transmission .
N. Radiation
O. Scatter
4

Filagrin:
K. A cationic protein rich in histamin
L. Organizes intermediate filaments into a macromolecular structure
M. Is released at the level of spinous layer
N. Its synthesis starts in the basal layer
O. Has no role in water preservation.
2

Which of these dermal molecule is not a glycoprotein:


K. Laminin
L. Thrombospondin
M. Vitronectin
N. Tenascin
O. Syndican
5

Elastic connective tissue:


L. Extends from papillary dermis to reticular
M. Is absent in hair follicules sheathes
N. mature Elastic fibers contain 75% elastin
O. oxytalan fibers form microfibrils containing soluable Elastin
P. mature Elastic fibers are most abundant in middermis
4
Microfibrillar and amorphous matrix of the dermis :
L. Is constituted wholly of several proteoglycas.
M. Tropoplastin, the precursor of Elastic fibers is poor in hydrophobic amino acids
N. Desmosine is a cross linked 3 lisle residues of tropoplastin
O. Microfibrillar matrix molecules Protect Elastic fibers from complement assisted destructive
reactions
P. Free glycosoaminoglycans constitue the bulk of the matrix
5

The collagen:
L. Collagen molecules are composed of 2 chains according to type
M. Some chains have a helical domain others have not
N. chains are glycosylated before assembly into procollagen
O. Procollagen is cleaved by hydrolytic enzymes.
P. Type IV is aligned into a staggered pattern
3

The SUBCUTIS:
L. Is absent in eyelids and female genitalia
M. The thickness of the layer is independent of race
N. Up to the 14th week is made of mesenchymal lipobalsts
O. Matures into lipolytes by the 20th week
P. Some of the primitive cells mature into macrophage
C

Small proline rich proteins (SPRR):


K. Cross link with involucrin.
L. 5 genes encode for the 3 types known
M. Expression is induced by UV radiation
N. SPRR 3 Is abundantly expressed in vivo
O. SPRR 2 is the least expressed in vivo
3

What anesthetic would you give to a patient with an amide allergy for a punch biopsy?
K. Lidocaine.
L. Benzocaine.
M. Prilocaine.
N. Procaine
O. Mepovacaine.
4

.
The inheritance pattern of chronic granulomatous disease is:
A. AD
B. AR
C. X-linked dominant
D. X-linked semidominant
E. X-linked recessive
5

The inheritance pattern of dermatosis with acantholysis is:


A. AD
B. AR
C. X-linked dominant
D. X-linked semidominant
E. X-linked recessive
A

Comma – shaped corneal opacities is characteristic of:


A. Netherton syndrome
B. Lamellar ichtyosis
C. Bullous congenital ichtyosiform erythroderma
D. X –linked Ichtyosis
E. Ichtyosis hysteri
D

A gene showing codominance:


A. Has both alleles independently expressed in the heterozygote
B. Has one allele dominant to the other
C. Has alleles tightly linked on the same chromosome
D. Has alleles expressed at the same time in development
E. Has alleles that are recessive to each other
A

Mutations which occur in body cells which do not go on to form gametes can be classified as :
A. Auxotophic Mutations
B. Somatic Mutations
C. Morphological Mutations
D. Oncogenes
E. Temperature sensitive Mutations
B

Primary cell in creating granulation tissue is:

A. Neutrophil
B. Macrophage
C. Fibroblast
D. Keratinocyte
E. Myofiboblast
3

Touch, pain, temperature and itch are mediated by???


K. Meissner’s corpuscles
L. Vater - pacini corpuscles
M. Penicillate fibers
N. Kraus end bulbs
O. Papillary nerve endings
3

The drug which has the best coverage for impetigo in children is:
K. Penicillin
L. Erythromycin
M. Azithromycin
N. Clarithromycin
O. dicloxacillin
5

The element necessary for function of matrix metalloproteinase is?


K. Zinc
L. Ca
M. Mg
N. Cu
O. Sulfur
1

The antibody which represent a marker for epidermal proliferation is:


K. Ki -67
L. K 10
M. K 13
N. K 16
O. P 53
1

An aging – related nail change in elderly persons is:


K. Asymmetric lunula
L. Macrolunula
M. Microlunula
N. Triangular lunula
O. Variably shaped lunula
3

One of the following is not a constituent of eccrine sweat:


K. Lactate
L. Phosphate
M. Urea
N. Sodium
O. Potassium

Glucocorticoids preferentially induce fracture in:


L. Cortical bone
M. Trabecular bone
N. Long bone
O. Irregular bone
P. Marrow rich bone
B

The addition of an H2 antagonist to an H1 antagonist for the management of chronic urticarial


might?
K. Help itch but not wheal numbers or severity
L. Reduce wheal numbers and severity but not itch
M. Stabilize mast cells
N. Discourage the development of tolerance to the antagonist
O. Lead to better absorption of the H1 antagonist
2

One of the following is not a physiologic role of IL - 10:


K. Limiting the inflammatory response
L. Supporting humoral immunity
M. Preventing overwhelming immune response
N. Increasing macrophage Ag presenting
O. Upregulating Th2 immune pathway
D
The protein which is up-regulated in the absence of plakoglobin is:
L. Beta – catenin
M. Desmocollin – 1
N. Envoplakin
O. Loricrin
P. Plakophilin
1

Poxviruses replicate in the:


K. Host cell cytoplasm
L. Host cell nucleus
M. Host cell mitochondria
N. Outside the Host cell
O. Inside and outside the Host cell
A

,The most important inflammatory cells to granuloma formation in the skin are:
F. Platelets
G. Neutrophils
H. B-cell lymphocytes
I. T – cell lymphocytes
J. Eosinophils
D

The induction of an immune response directed against one or more specific tumor antigens in
immunotherapy is known as:
K. Passive
L. Active
M. Passive non specific
N. Passive specific
O. Active specific
5

The proliferation of T cells after activation is primarily controlled by:


L. IL-2 receptor
M. IL-12
N. IL-4
O. IL-4 receptor
P. INF gama
2

Which of the following is true about gap junctions:


L. They are composed of 6 connexins
M. They mediate tight adhesion between cells
N. They mediate calcium-induced calcium release
O. They require several heterotypic connexin subunits for proper function
P. All of the above
1

1-Which of the following lasers has gas medium:


A. Alexandrite.
B. Ruby.
C. Diode.
D. Erbium YAG.
E. Krypton
E

2-Which of the following lasers has deepest penetration:


A. CO2.
B. Erbium Yag.
C. Nd: YAG
D. PDL.
E. Argon.
C

3-Which of the following is not a complication of snake bite:


A. Metabolic acidosis
B. Renal failure.
C. Cardiac arrhythmia.
D. Respiratory failure.
E. Disseminated intravascular coagulation.

4-In laser treatment, the shortest thermal relaxation time is used in:
A. Tattoo
B. Melanosomes.
C. Port-wine stain (PWS) vessels.
D. Terminal hair follicle.
E. Leg veins.

5-Fleeting arthritis is a characteristic clinical feature of:


A. Ankylosing arthritis.
B. Gouty arthritis.
C. Rheumatic arthritis
D. Septic arthritis.
E. Rheumatoid arthritis.
C

6-Among the following , which of the following does not cause dementia:
A. Iron deficiency
B. Alcoholism.
C. Alzheimer 's disease.
D. AIDS.
E. Thiamin deficeincy.
A

7-Which of the following is false about depression:


A. Feeling of guilt.
B. Suicidal thinking.
C. Loss of interest.
D. Weight changes.
E. Flight of ideas
5

8-In acute viral hepatitis, which of the following is false:


A. Negative antibody test excludes acute infection
B. Variable incubation periods.
C. Can be transmitted by several routes.
D. Sometimes a self limiting disease.
E. Liver cirrhosis is a complicaion.

9-About infectious diarrhea, which statement is not true:


A. May present with bloody diarrhea.
B. Caused either by bacteria or viruses.
C. Colonoscopy is essential for diagnosis
D. Can be complicated by septicemia.
E. Isolation prevents the spread of infection.

10-Regarding diabetes mellitus, which of the following is not true:


A. Type 2 is caused mainly by insulin resistance.
B. Diabetes ketoacidosis is commonly seen in type 1.
C. Prolonged hypoglycemia can induce brain damage.
D. Microalbuniemia is an indicator of diabetic nephropathy.
E. Sulphonylurea drugs are used to treat type 1 diabetes mellitus

11-Which of the following is not seen in hyperthyroidism:


A. Atrial fibrillation.
B. Weight loss.
C. Hyperlipidemia
D. Heat intolerance.
E. Systolic hypertention.

12-The first therapeutic dose of NB-UVB therapy is:


A. 40% of MED.
B. 70% of MED
C. 20% of MED.
D. 100% of MED.
E. 10% of med

13-Which of the following types of laser is not Ablative:


A. CO2 laser
B. Q switched ND:YAG 532
C. Diode laser
D. Erbium Yag laser
E. Pulse-dye laser

14-Concerning UV light, which statement is not true:


A. Wave lengh of 360 nm is 1000-fold less erythemogenic than 300nm.
B. An immediate erythema reaction is rarely seen i UVB –induced sunburn.
C. DNA is hypothesized to be chromophore for UVB erythema.
D. Delayed tanning is usually as result of exposure to UVA
E. Fair skin usually tans only with UVB doses above the erythema threshold.

15-Which of the following is not true about newborn skin:


A. Stratum corneum thickness is 9-10 µm.
B. Spinous cell glycogen content is abundant
C. Similar number of melanocytes to adult.
D. Elastic fibers are small and immature.
E. The epidermal surface has vernix.

16-Which of the following is not a complication of snake bite:


A. Metabolic acidosis
B. Renal failure.
C. Cardiac arrhythmia.
D. Respiratory failure.
E. Disseminated intravascular coagulation.
Answer/A
17-The majority of patients with dermatitis herptiformis are associated with:
A. HLA-B8
B. HLA-A5
C. HLA-B51
D. HLA-CW
E. HLA-DR4
A/A

18-One of the followings is not correct regarding vibration white finger:


A. Development of white finger or fingers in response to cold
B. Associated permanent loss of sensation
C. Possible permanent finger neuropathy and pain in the affected limb
D. Associated with exposure to vibrations between 30 and 300 Hz
E. Worker at risk include operators of chainsaws and pneumatic tools

19-One of the followings is not correct about non-bullous congenital ichthyosiform erythroderma:
A. Collodion baby
B. Cicatricial alopecia
C. Mutations in genes encoding lipoxygenase 3
D. Mutations in genes encoding K1 and K10
E. Ectropion

20-Which of the following is not a feature of Behcet's disease:


A. Oro-genital ulceration.
B. Pulmonary fibrosis
C. Blindness.
D. Recuurent DVT.
E. Pathergy test.
A/B

21-The followings are lesions usually follow Blaschko’s lines EXCEPT:


A. Hypomelanosis of ITO
B. Focal dermal hypoplasia (Goltz syndrome)
C. Menkes kinky hair disease
D. Pigmentary demarcation lines, type c
E. Epidermal nevus.
A/D

22-The following is not true in Sjogren 's syndrome:


A. It is more common in females than males.
B. There is salivary gland enlargement.
C. Reynaud's phenomenon may be seen.
D. Lymphadenopathy.
E. It is a seronegative arthritis
A/E ‫صح‬
23-One of the followings is correct about Erythema infectiosum:
A. Caused by ds DNA virus
B. Characterized in children by cardiac involvement
C. It is bacterial infection
D. Also called 5th disease
E. Does not affect pregnancy outcome
A/D

24-A 1-year-old boy presents with a seborrheic dermatitise


like rash isolated to the scalp. A skin biopsy specimen is
obtained and reveals Langerhans cell histiocytosis.
1. What is the most likely disease course?
a. Isolated cutaneous disease that spontaneously
remits within a few months
b. Isolated cutaneous disease that is responsive to
topical steroids
c. Isolated cutaneous disease with high recurrence
d. Life-threatening multisystem disease with high
recurrence
e. Multisystem disease with a benign course
----
2. Which of the following features indicates a poor
prognosis?
a. Multifocal rather than solitary cutaneous
involvement
b. Poor response to initial therapy
c. Male sex
d. Langerhans cell histiocytosis proliferation index
[20% seen on the biopsy specimen
e. Fever with mucosal involvement
------
The patient continues to be refractory to both first-line
and salvage therapies. Genetic analysis indicates a BRAF-
600E mutation.
3. Which of the following is NOT true of the BRAF
inhibitor vemurafenib?
a. Usage is only indicated in severe disease because
of the risk of malignancy
b. Cutaneous side effects are frequent and severe
c. Clinical evidence of efficacy is high, especially in
children
d. May cause elevated liver enzymes
e. May cause prolonged QT interval

25-The type of pityriasis rubra pilaris (PRP) associated with HIV infection is:
A. Type 1
B. Type II
C. Type VI
D. Type V
E. Type III
A/c

26-Pustular psoriasis and acrodermatitis continua of Hallopeau are associated with:


A. HLA-B13
B. HLA-B27
C. HLA-BW16
D. HLA-DR7
E. HLA-AW19
A/B

27-One of the followings is not acute complication of toxic epidermal necrolysis (TEN):
A. Renal failure
B. Bacterial infection and septicemia
C. Ectropion
D. Hyper catabolism: insulin resistance
E. Diffuse interstitial pneumonitis

A/C

28-One of the following drugs does not precipitate Erythema multiforme primarily:
A. B-blockers
B. Non-steroidal anti-inflammatory drugs
C. Sulfonamides
D. Antibiotics
E. Anti-epileptics
A

29-All of the followings may be used for treatment of acne fulminans EXCEPT:
A. Oral corticosteroids
B. Oral isotritinoin
C. Oral antibiotics
D. Methotrexate
E. Dapsone
D

30-One of the followings is not common systemic manifestations of sweet’s syndrome:


A. Arthralgias
B. Arthritis
C. Myalgias
D. Fever
E. Abdominal pain

31-All of the following chemotherapeutic agents may cause mucosal hyper- pigmentation
EXCEPT:
A. Busulfan
B. 5-fluorouracil
C. Vinblastine
D. Hydroxyurea
E. Cyclophosphamide
C

32-One of the followings is not subtype of rosacea:


A. Erythematotelangiectatic
B. Comedopustular
C. Papulopustular
D. phymatous
E. Ocular

33-One of the followings is not a cause of hypothalamic hyperhidrosis:


A. Hyperthyroidism
B. Acromegaly
C. Hypoglycemia
D. Acute intermittent porphyria
E. Hypothyroidism
A/E

34-One the followings is not cutaneous manifestations of Behcet’s disease:


A. Sweet’s syndrome-like dermatosis
B. Erythema nodosum
C. Neutrophilic lobular panniculitis
D. Pyoderma gangrenosum
E. Pyoderma vegetans
A/E

35-One of the followings is not correct regarding treatment of Hidradenitis suppurativa:


A. Systemic steroids often leads to dramatic improvement
B. CO2 laser has been proven to be quite effective
C. Isotritinoin has been particularly effective
D. Infliximab may be used
E. Botulium toxin A to reduce sweating

36-The biologic-false- positive results in non-treponemal tests include the followings EXCEPT:
A. Pregnancy
B. Drug abuse
C. Hepatic cirrhosis
D. Endemic treponematosis
E. Idiopathic, familial

37-One of the followings is not a form of sycosis:


A. Tuberculoid
B. Barbea
C. Lupoid
D. Mycotic
E. Herpetic

38-Which of the following is not included in the herpes virus group:

A. Epstein Bar virus.


B. Cytomegalovirus.
C. Varicella zoster virus.
D. Coxackie virus
E. Human herpes 8

39-The incubation period of varicella is:

A. 3 days.
B. 7 days.
C. 14 days
D. 21 days.
E. 50 days.

40-One of the following statements about keratins is false:

A. Keratin filaments are the hallmark of keratinocytes.


B. All keratins are within 30-40 Kd molecular mass.
C. Most acidic keratin genes localize to chromosome 12 q11, q13
D. Basal cells has a large nucleus with prominent nucleolus.
E. Not all basal cells have the potential to divide.

41-Fingernail growth is:

A. 0.1 mm/day
B. 0.2 mm/day.
C. 0.3 mm/day.
D. 0.4 mm/day.
E. 0.5mm/day.

42-The biologic-false- positive results in non-treponemal tests include the followings EXCEPT:
A. Pregnancy
B. Drug abuse
C. Hepatic cirrhosis
D. Endemic treponematosis
E. Idiopathic, familial
A/D

43-One of the followings is not correct regarding treatment of Hidradenitis suppurativa:


A. Systemic steroids often leads to dramatic improvement
B. CO2 laser has been proven to be quite effective
C. Isotritinoin has been particularly effective
D. Infliximab may be used
E. Botulium toxin A to reduce sweating

44-Criteria for HIV seropositivity include the followings EXCEPT:


A. Repeatedly positive ELISA
B. Positive westen blot study
C. Reactivity against viral protein marker p24,gp41
D. Reactivity against viral protein marker gp 120/160
E. Reactivity against viral protein marker p51
A/E

45-Which of the following is not true:

A. New synthesis of K1/K10 occurs in the spinous layer.


B. Profillagrin mRNA is evident in the granular cell layer.
C. Horny cell is the largest cell in the epidermis.
D. Cells of the deeper layers of stratum corneum are prone to desquamation
E. Cells of the mid stratum corneum have higher concentration of amino
A\D

46-Merkel cells are evident in volar epidermis of human fetus at:

A. 4 weeks.
B. 8 weeks.
C. 12 weeks
D. 16 weeks.
E. 20 weeks.
A/C

47-The weakest region in dermal epidermal junction (DEJ) is:

A. Basement membrane.
B. Lamina lucida
C. Lamina densa.
D. Sublamina densa.
E. Hemidesmosomes
A/B

48-Vitronectin is present throughout the dermis in association with:

A. Elastic fibers
B. Collagen fibers.
C. Reticular fibers.
D. Anchoring filaments.
E. Tonofilaments.
A/A

49-Which of the following collagen is polymorphic :

A. Collagen III.
B. Collagen V
C. Collagen VII.
D. Collagen XVII.
E. Collagen IV.

A/B

- [ ] 50-Cigar-shaped yeast, histologically is characteristic for:

- [ ] A. Blastomycosis.
- [ ] B. Sporotrichosis
- [ ] C. Candidiasis.
- [ ] D. Histoplasmosis.
- [ ] E. Actinomycosis.
- [ ] A/B
-[]
-[]
- [ ] 51-Which of the following biological drugs block the interaction between CD2 and LFA-3
receptors:
- [ ] A. Etaneracept.
- [ ] B. Infliximab.
- [ ] C. Alefacept.
- [ ] D. Adalimumab.
- [ ] E. Ustekinumab.
- [ ] A/C

52-Hemophilus ducreyi is the causative organism of:

A. Granuloma inguinale.
B. Hard sore.
C. Lymphogranuloma venerium.
D. Urethritis.
E. Soft sore
A/E

53-Which of the following provides the principal barrier to transepidermal water loss:

A. Involucrin.
B. Profillagrin.
C. Lamellar bodies.
D. Lorocrin.
E. Fillagrin
C

54-Hair perforation test is useful to distinguish Trichophyton rubrum from:

A. Epidermophyton floccosum.
B. Trichophyton schoenleinii.
C. Microsporum gypsum.
D. Trichophyton mentegrophytes
E. Trichophyton verrucosum.
A/D

55-Concerning fungi, which of the following is true:

A. Most fungi are obligate anaerobes.


B. Trichophyton violaceum is a geophilic.
C. No microconidia in epidermophyton floccosum
D. Pectinate hyphae is characterstic for Trichophyton mentegrophytes.
E. Microsporum gypseum causes pig ringworm.
A/C

56-One of the following statements is not true:

A. γ chain is a heavy chain of IgG.


B. At the age of one year, the highest immunoglobulin is IgM.
C. IgG4 does not fix the complement.
D. IgA1 is produced by lymphocyte in the respiratory and GIT
E. IL-4 foster IgE production.
D

57-The most predominant vessels within the upper dermis are:

A. Superficial arteriolar plexus.


B. Terminal arterioles .
C. Arterial capillaries .
D. Venular capillaries.
E. Post capillary venules
E

58-Which of the following is not true about natural killer cells:

A. They compromise 2-15% of circulating lymphocytes.


B. They have no TCR.
C. They have surface Immunoglobulin receptor.
D. They have CD4 and CD8 receptors
E. They can be distinguished by CD16.
D

59-Which of the following is not true about genetic diseases:

A. There is a high risk to relatives in disease due to single gene defect.


B. Every nucleated cell has 23 pairs chromosomes.
C. In sex chromosomes, males are homologous
D. Translocation is an abnormal chromosomal structure.
E. In DNA strand thymine (T) is always pairs with adenine (A).
C

60-Plasma cells do not reside in one of the following:

A. Bone marrow.
B. Blood
C. Respiratory tract.
D. Gastrointestinal tract.
E. Spleen.
A/b

61-Which of the following impairs cartilage formation:

A. Fluoroquinolones
B. Clindamycin.
C. Penicillins.
D. Rifampicin.
E. Dapsone.
A

62-One of the following is not a function of IL-1:

A. Suppression of migration and activation of Langerhans cells


B. T cell proliferation.
C. Increase expressions of adhesion molecules on endothelium.
D. Fever.
E. Induction of chemotaxis and cytotoxicity of monocytes.

63-Adrenaline with local anesthesia should be avoided in lesions on:

A. Forearm.
B. Abdomen.
C. Thigh.
D. Fingers and toes
E. Scrotum.
D
64-With H&E stain normal melanocyte appears as:

A. Clear cell
B. Granular cell.
C. Polyhidral cell.
D. Spindle cell.
E. Rounded cell.
A

65-Which of the following side effect is associated with intravenous biphosphonates:


A. Blurred vision.
B. Sexual dysfunction.
C. Flu-like symptoms
D. Anxiety.
E. Darkening of the skin.
C

66-Expression of which marker is reduced in psoriasis:

Ki- 67 antigen.
Keratin 16.
ρ-3 integrin.
Filagrin
Keratin 6.
D

67-Fluconazole differs from other azoles by:


A. High molecular weight.
B. Low water solubility.
C. Not affected by food or gastric acidity
D. Poor penetration to CSF.
E. Metabolized in the liver.
C

68-The chromophore of pulsed dye laser is:


A. Oxyhemoglobine
B. Melanine.
C. Carboxyhemoglobine.
D. Collagen.
E. DNA.

69-Which of the following specimens is not used for Mycobacterium tuberculosis staining:
A. CSF.
B. Pleural fluid.
C. Sputum.
D. Biopsy material.
E. Urine
E

70-Thalidomide downregulates:

A. Prostaglandin D2.
B. β2- Intregrin and β1 &α4 integrin
C. Calcineurin.
D. CYP3A4.
E. IL-8.
B
71-The major function of the Golgi apparatus is :

A. Endocytosis.
B. Secretion.
C. Diffusion of small molecules.
D. Protein synthesis
E. Energy generation.
B

72-The international standard series patch test consists of:

A. 25 allergens
B. 22 allergens.
C. 23 allergens.
D. 20 allergens
E. 21 allergens.
D

73-Following UV irradiation, the most prevalent DNA photoproduct is:


A. Cyclobutane pyrimidine dimer
B. Adenine dehyrodimer.
C. Thymine-adenine dimer.
D. Cytosine photohydrate.
E. Pyrimidine 6,4 pyrimidone.
A
74-A minimal erythema dose (MED) phototest should be read at:

A. Two hours.
B. 24 hours
C. 48 hours.
D. 12 hours.
E. 96 hours.
B

75-Which of the following provides the principal barrier to transepidermal water loss:

A. Involucrin.
B. Profillagrin.
C. Lamellar bodies.
D. Lorocrin.
E. Fillagrin
A/E

76-Which of the following is not a feature of Malassezia furfur:

A. It is a lipophilic fungi.
B. It is a saprophytic.
C. It is a dimorphic yeast-like.
D. It is a Gram negative
E. Olive oil has to be added to Sabourauds media for its growth.
D

77-Which of the following is not true about clofazimine:

A. It is a phenazine dye.
B. Its half life is 6-12 days
C. It is mainly absorbed from jejunum.
D. It may cause red - brown discoloration of skin.
E. It can be used in pregnancy.
E

78-Occlusive dressing enhance absorption of topical steroids by:

A. 4 folds.
B. 10 folds
C. 100 folds.
D. 2 folds.
E. 8 folds.

79-Tissue contraction begins at:

A. 24 hours of wound healing.


B. 3rd day of wound healing.
C. 2nd week of wound healing
D. First month of wound healing.
E. 3rd month of wound healing.
C

80-Which of the following is false about HIV:

A. Has reverse transcriptase enzyme.


B. Belongs to the genus lentivirus.
C. There are two serotypes of HIV.
D. Has high affinity for CD4.
E. Is a DNA virus
E
RNA

81-PAS stain reveals:

A. Calcium.
B. Amyloid.
C. Reticular fibers.
D. Basement membrane
E. Bacteria.

82-Which of the following facts about histamine is true:


A. Histamine is a potent activator of mast cells.
B. The major source of histamine in the skin are keratinocytes
C. There are currently 6 known histamine receptors
D. The H2 receptors is the receptor mainly responsible for pruritus
E. In addition to H1, H3 and H4 receptors can also modulate pruritus
E

83-Which of the following is the initial cutaneous response to surgical incision:

A. Formation of a fibrin plug


B. Migration of macrophages.
C. Migration of polymorphonuclear leukocytes
D. Mitosis of fibroblasts.
E. Vasoconstriction
E

84-The envelop glycoprotein of HIV virus is:


A. p24.
B. gp 41.
C. p17.
D. gp 120
E. Reverse transcriptase.
D

85-Which of the following is not true regarding human papilloma virus:

A. It belongs to pox virus


B. It is DNA virus.
C. Some strains are oncogenic.
D. Can infect mucous membrane.
E. It can be transmitted sexually.
A

86-Which of the following groups azathioprine belongs to:

A. Purine analogues
B. Nitrogen mustards.
C. Alkylsulfones.
D. Pyrimidine analogues.
E. Folic acid analogues.
A
87-Which medium is used for fixation and transport of biopsy specimen to be used for
immunofluorescence technique:

A. Neutral buffered formalin solution.


B. Michel solution
C. Zenker’s fluid.
D. Formal saline solution.
E. Frozen tissue.
B
88-Which is not true about Sarcoptes scabiei:

A. It multiplies only on human skin.


B. Males are smaller than females.
C. Females live up to 6 weeks.
D. Scybala means packet mite eggs
E. Symptoms usually occurs after 2-6 weeks.
D

89-Which of the following inhibits DNA gyrase enzyme:

A. Tetracycline.
B. Ciprofloxacin
C. Dapsone.
D. Clarithromycin..
E. Cefriaxone.
B
90-In which of the following sites the apocrine sweat gland is not present:

A. Mucous membranes
B. External auditory canal.
C. Eye lids.
D. Axillae.
E. Nipples.
A

91-Which of the following diseases is transmitted by mosquitos:

A. Trypanosomiasis.
B. Bilharziasis.
C. Filariasis
D. Schistosomiasis.
E. Cysticercosis.
C

92-Which of the following should not to be taken after meals?

A. Rifampicin
B. Griseofulvin.
C. Clofazimine.
D. Dapsone.
E. Itraconazole.
A

93-For neutrophils to accumulate at an inflammatory site, they must be first attracted to the site
in a process called:

A. Cytotaxis.
B. Chemotropism.
C. Leukocytosis.
D. Chemotaxis
E. Extravasation.
D

94-Flap valves are features of:

A. Glomus bodies.
B. Arterioles.
C. Skin lymphatic vessels
D. Venules.
E. Motor nerves.
C

95-Soft keratin is present in:

A. Cortex of the hair.


B. Cuticle of the hair.
C. Hair matrix.
D. Nail plate.
E. Medulla of the hair

96-Gene signature is:

A. The relative position of genes along the chromosome


B. The expression level of a gene
C. A pattern of genetic markers which infer a biologic state
D. The effect of one gene’s expression level on other genes
E. A measure of a gene’s relative importance in a cell type
C

97-Fillagrin is first detectable in:

A. 8 weeks of fetal life


B. 12 week of fetal life
C. 20 weeks of fetal life
D. 15 weeks of fetal life
E. 10 weeks of fetal life
D

98-One of the following mucocutaneous features occur more often in girls with Turner syndrome
than in normal girls:

A. Halo nevi
B. Scrotal tongue
C. Vitiligo
D. Psoriasis
E. Macrocheilitis
B

99-One of the following is NOT effective for the treatment of pyoderma gangrenosum:

A. Systemic corticosteroids
B. Cyclosporine
C. Infliximab
D. Tacrolimus ointment
E. Granulocytes – colony stimulating factor
E

100-The most serious complication of SJS/TEN is related to:

A. Pancreas
B. Kidney
C. Liver
D. Spleen
E. Eye
E

101-The most common granuloma which develops in papillary dermis is:

A. Sarcoidosis
B. Lichen nitidus
C. Tuberculoid leprosy
D. T. B. granuloma
E. Ruptured hair follicle granuloma
B

102-Inflammatory paronychia can be caused by deficiency of:


A. Zinc
B. Pyridoxine
C. Cupper
D. Vitamin A
E. Vitamin C
A

103-One of the following nail anomalies increases significantly after renal transplantation:

A. Absence of lunula
B. Splinter hemorrhage
C. Half-and-half nails
D. Leukonychia
E. Onychomycosi
D JAAD
104-One of the following statements about brittle nails is true:

A. A portion of the nail plate is formed by the nail bed


B. Calcium and sulfur content of the normal nail are high and contribute to the hardness of the
nail plate
C. Gluing fractures of the nail plate and applying nail hardeners are not useful therapeutic
options for onychoschizia
D. Onychorrhexis is characterized by lamellar splitting of the free edge and distal nail plate
E. Onychomadesis refers to nails that have thickened and developed a deep curve
A👍 👍 👍

105-Tuberous sclerosis is associated with:

A. Diabetes mellitus
B. Polycystic kidney
C. Leukemia
D. Juvenile rheumatoid arthritis
E. Muscular dystrophy
B

106-The following feature is a consistent histologic finding for mycosis fungoides:

A. Pautrier's microabscess
B. Epidermotropism of T lymphocytes
C. Spongiosis
D. Presence of a TCR rearrangement
E. Wedge shaped infiltrate
B

107-Histologic examination of nevus anemicus shows:

A. Presence of melanocytes around blood vessels


B. Absence of melanocytes
C. Smooth muscle hamartoma
D. Collections of mast cells
E. Normal skin
E

108-Fox-fordyce disease is a result of obstruction and rupture of:

A. Intraepidermal portion of apocrine sweat duct


B. Dermoepidermal portion of apocrine sweat duct
C. Intraepidermal portion of eccrine sweat duct
D. Dermoepidermal portion of eccrine sweat duct
E. Dermal portion of both eccrine and apocrine sweat ducts
A
109-Dermal dendrocyte hamartoma shows positive immunohistochemical staining of which of
the following?

A. HLA-DR
B. CD1a
C. CD34
D. S100 protein
E. Factor IX
C

110-Periodic acid–Schiff–positive granules in the outer layer of eccrine ductal cells can be
observed in:

A. Niemann-Pick disease
B. Lafora's disease
C. Fabry's disease
D. Hurler's syndrome
E. Kanzaki syndrome
B

111-Vaginal discharge smear test is not useful to screen:111-

A. Gonococcal infection.
B. Trichomoniasis.
C. Candidiasis
D. Syphilis
E. Bacterial vaginosis
D

112-Which of the following is not a cutaneous manifestations of Epstein-Barr virus infection:

A. Oral hairy leukoplakia.


B. Gianotti-crosti syndrome.
C. Genital ulcers.
D. The papular purpuric gloves and socks syndrome.
E. Erythema multiforme
D

113-Which of the following clinical associations with parvovirus B19 is false:

A. Erythema infectiosum.
B. Papular-purpuric gloves and socks syndrome.
C. Generalized petechial eruption.
D. Erythema multiforme
E. Hydrops fetalis.
D
114-Which of the following formaldehyde related allergens is associated with textile dermatitis?

A. Diazolidinyl urea
B. DMDM hydantoin
C. Imidazolidinyl urea
D. Quaternium 15
E. Melamine formaldehyde
E

115-One of the following is caused by super Antigen:

A. Kawasaki disease
B. Lichen planus
C. Lichen sclerosus et atrophicus
D. Alopecia mucinosa
E. Chronic plaque psoriasis
A

116-Which one of the following has the best photoprotective effect?

A. Clouds
B. Densely leafed trees
C. Fog
D. Pollutants
E. Water
B

117-The most frequent cause of PUVA-induced phototoxicity is:

A. Therapy on consecutive days


B. The dose of UVA was too high
C. UVB was given in addition to UVA
D. Concomitant medication
E. Concurrent disease
B

118-Which specific B cell depleting agent has been associated with the development of Kaposi
sarcoma?

A. Prednisone
B. Cyclosporine
C. Methotrexate
D. Cyclophosphamide
E. Rituximab
E
119-One of the following inherited photosensitivity disorders has an action spectrum within UVB-
UVA II range:

A. Bloom syndrome.
B. Trichothiodystrophy.
C. Cockayne syndrome.
D. Xeroderma pigmentosa
E. UV-sensitive syndrome
D

120-The usual course of erythema toxicum neonatorum is:

A. Resolution within first 5 days


B. Resolution within 1-2 months
C. Resolution within first 48 hours
D. Resolution within 1-2 weeks
E. Resolution after 1 year of age
D

121-Merkel cell carcinoma is associated with:

A. Frequent local relapses


B. Low metastatic potential
C. Excellent response to wide local excision in all clinical stages
D. Distant organ spread preceding regional lymph node spread
E. Lack of in vitro sensitivity to cisplatin
A

122-T4 endonuclease V is an enzyme that repairs damaged DNA in:

A. Bacteria
B. Fungi.
C. Viruses.
D. Yeast.
E. Prions
A

123-The classical localization of segmental vitiligo in children is :

A. Perineal and perianal


B. Face.
C. Back.
D. Abdomen.
E. Upper limbs
2
face is the most common site
Fig 66-7

124-A 4 year old child presents with a linear clustering of verrucous brown papules on his
posterior leg that have present since birth. The most likely diagnosis is:

A. Nevus sebaceous of Jadassohn


B. Lichen striatus
C. Linear epidermal nevus
D. Flat warts
E. Incontinentia pigmenti
C

125-Hemorrhagic bullae occurring after the consumption of raw seafood is most likely caused
by :

A. Mycobacterium marinum
B. Vibrio vulnificus
C. Erysipelothrix rhusiopathiae
D. Sporothrix schenkii
E. Photobacterium phosphoreum
B

126-Which tattoo color produces the most photosensitivity reactions?

A. Red
B. Blue
C. Green
D. Yellow
E. White
D

127-Which of the HPV human papilloma viruses proteins is involved in carcinogenesis by


degrading p53?

A. E2
B. E4
C. E5
D. E6
E. E7
D

128-The chromophore of the Er:YAG laser is

A. Collagen
B. Elastic fibers
C. Water
D. Melanin
E. Hemoglobin
C

129-Which of the following medication should not be discontinued preoperatively?

A. Aspirin
B. NSAIDs
C. Clopidogrel
D. Ticlopidine
E. Dipyridamole
E

130-The common presentation of non-gonococcal urethritis is:

A. Multiple genital ulcers


B. Hematuria
C. Urethral discharge
D. Dysuria
E. Inflammatory adenitis
C

131-The average daily requirement of vitamin C is:

A. 5 mg
B. 10 mg
C. 20 mg
D. 40 mg
E. 50 mg
E

132-Damage to the temporal branch of the facial nerve which supplies the frontalis muscle
results in:

A. Difficulty raising the upper eyelid.


B. Difficulty raising the eyebrow
C. Difficulties in forehead furrows
D. It has no effect in the forehead furrows
E. Difficulty in the movement of lower eyelid
B

133-What is the major function of urocanic acid?

A. Bacteriocidal acid produced by stratum corneum


B. Primarily a UVB filter
C. Primarily a UVA filter
D. Helps degrade free fatty acids
E. Aids in protecting the skin from dermatophytes
C

134-Which of the following statements about transforming growth factor β (TGF β) is false?

A. TGFβ 1 was originally found in platelets.


B. TGFβ 2 was found in bovine bone.
C. TGFβ 3 was found in human rhabdomyosarcoma.
D. TGFβ stimulates the growth of keratinocytes
E. TGFβ stimulates the growth of fibroblast.
D👍
T 141-2 TGF a —> keratinocytes proliferate

135-One of the following is not correct about stem cells and location:
A. Interfollicular epidermis
B. Palmoplantar epidermis
C. Sebaceos gland
D. Melanocytes(bulge)
E. Hair follicul (bulge)
D

136-One of the following is NOT TRUE about microscopic anatomy of nail matrix:

A. Nail matrix contains melanocytes in the highest three cell layers


B. Produces the nail plate
C. The cells differentiate the expression of trichocytokeratin
D. Nail matrix contains Langerhan’s cells
E. During the process of nail matrix growing there is retained nuclei
D

137-The target antigen in dermatitis herptiformis is :

A. Desmoglein 1
B. Desmoglein 3
C. BPAG 1, BPAG2
D. Transglutaminase -3
E. Collagen 7
D

138-One of the followings is not correct about Langerhan’s cell:

A. They can be identified by enhanced green fluorescent protein, associated fluorescence


B. They can release IL-1
C. They are more likely to induce Th1 rerponses than the Th2.
D. They are indispensable for contact hypersensitivity
E. They are possessing Birbesck granules

139-The earliest development of the hair rudiment occurs in the eyebrow, upper lip and chin at
about:

A. 10 weeks of gestations
B. 12 weeks of gestations
C. 9 weeks of gestations
D. 13 weeks of gestations
E. 15 weeks of gestations
C

140-Pulmonary fibrosis is an adverse effect of:

A. Dapsone
B. Methotnexate
C. Cyclophosphamide
D. Cyclosporine
E. Azathioprine

141-Photosensitivity is a prominent feature of:

A. Zinc deficiency
B. Biotin deficiency
C. Niacin deficiency
D. Thiamine deficiency
E. Ascorbic acid deficiency

142-Urticaria is a prominent feature of which of the following?

A. Primary amyloidosis
B. Muckle wells syndrome
C. Lichen amyloidosis
D. Nodular amyloidosis
E. Melkerson Rosenthal syndrome

143-Which paraprotein is found most commonly in patients with pyoderma gongrenosum?

A. IgG
B. IgA
C. IgM
D. IgE
E. IgD
144-226Which cell type is most susceptible to cold injury during cutaneous cryosurgery:

A. Perineural
B. Endothelial
C. Fibroblast
D. Melanocyte
E. Keratinocyte

145-Treponema carateum causes :

A. Syphilis
B. Bejel
C. Yaws
D. Pinta
E. Periodontal disease

146-One of the following lasers has wavelength 532 nm

A. Erbium : YAG
B. Excimer
C. Argon
D. Ruby
E. KTP
E

147-One of the following types of cutaneous lupus is least associated with SLE:

A. Acute cutaneous L.E.


B. Rowell’s syndrome
C. L.E. tumidus
D. Chelblain lupus
E. Subacute cutaneous L.E
E

148-One of the followings is not correct about causes of genital ulcer diseases

A. Genital herpes, HSV2>HSV1


B. Primary syphilis, Treponema pallidum
C. Chancroid, haemophilus ducreyi
D. LGV, Chlamydia trachomatis type D-K
E. Donovanosis, calymmatobacterium granulomatis.
D

149-One of the followings is ester type of local anesthetics

A. Procaine
B. Lidocaine
C. Mepivacaine
D. Articaine
E. Bupivacaine hydrochloride
A

150-The location of obstruction in crystallina type of miliaria is:

A. Stratum granulosum
B. Stratum corneum
C. Stratum lucidum
D. Stratum basale
E. Dermal- epidermal junction
B

151-One of the followings is not needed for pre-acitretin screeming

A. Liver function test


B. Glucose
C. Urinalysis
D. Serum creatinine
E. Spinal X-ray
B

152-The cytokine which is reduced in psoriasis is :

IFN-α
IL-10
IL-2
IL-12
IL-23
2

153-Stem cells have high activity in one of the following conditions:

A. During fetal development and wound healing


B. Cycling in normal epidermis
C. Growth in culture
D. Potential for differentiation
E. Self- renewal
A

154-Desmocollin 1 antigen is observed in the following type of pemphigus

A. Pemphigus vulgaris
B. Pemphigus folliaceous
C. Paraneoplastic pemphigus
D. Drug-induced pemphigus
E. IgA pemphigus
B
155-One of the following is not complications of granuloma inguinale:

A. Permanent scarring
B. Inguinal adenitis
C. Squamous cell carcinoma
D. Depigmentation
E. Genitals destruction
B

156-The target antigen in dermatitis herptiformis is :

A. Desmoglein 1
B. Desmoglein 3
C. BPAG 1, BPAG2
D. Transglutaminase -3
E. Collagen 7
D

157-One of the following is not correct aout hair cycle :


a. Anagen phase lasts about 3 years
b. Catagen phase lasts about 3 months
c. Telogen phase lasts about 3 months
d. 84% of scalp hairs are in anagen phase
e. 2% of scalp hairs are in catagen phase
B

158-Hair follicule start to appear at :


a. 10 weaks of gestation.
b. 12 weaks of gestation.
c. 14 weaks of gestation.
d. 9 weaks of gestation
e. 16 weaks weaks of gestation.
D

159-Elastin is not produced by one of the following:


a. Fibroblasts
b. Smooth muscle cells
c. Macrophages
d. Endothelial cells
e. Condroblasts
C

160-Apocrine sweat glands are not found on one of the following sites :
a. Axiliae
b. Areolae
c. External auditory cana
d. Forehead
e. Eyelids
D

161-Sample epithelia are characterized by the keratin pair :


a. K5/k15
b. K5/k14
c. K8/k18
d. K1/k10
e. K7/k17
C

162-The amount of topical medication required for twice daily application to the entire face for
one week is approximately :

A. 30 gm
B. 60 gm
C. 90 gm
D. 120 gm
E. 180 gm
A

162-Regarding cutaneous drug eruption the commonest form is :

A. Urticaria and angioedema


B. Fixed drug eruption.
C. Steven-Johnson syndrome.
D. Anaphylaxis.
E. Exanthema maculopapular /morbiliform
E

164-Forschneimer’s sign is seen in :

A. Measles
B. Rubella
C. Scarlet fever
D. Herpangina
E. Infectious mononucleosis
B

165-Which type of interleukins is a mediator of the systemic acute phase response :

A. IL1
B. IL6
C. IL7
D. IL10
E. IL12
A

166-Transverse grooves on nail following systemic disease are called:

A. Mee’s line.
B. Coenen’s line.
C. Trachyonychia.
D. Beau’s line
E. Grove sign.
D

167-Pulmonary fibrosis is an adverse effect of:

A. Dapsone
B. Methotnexate
C. Cyclophosphamide
D. Cyclosporine
E. Azathioprine
B

168-226Which cell type is most susceptible to cold injury during cutaneous cryosurgery:

A. Perineural
B. Endothelial
C. Fibroblast
D. Melanocyte
E. Keratinocyte
D

169-Photosensitivity is a prominent feature of:

A. Zinc deficiency
B. Biotin deficiency
C. Niacin deficiency
D. Thiamine deficiency
E. Ascorbic acid deficiency
C

170-Urticaria is a prominent feature of which of the following?

A. Primary amyloidosis
B. Muckle wells syndrome
C. Lichen amyloidosis
D. Nodular amyloidosis
E. Melkerson Rosenthal syndrome
B

171-Which paraprotein is found most commonly in patients with pyoderma gongrenosum?

A. IgG
B. IgA
C. IgM
D. IgE
E. IgD
B

172-Which one is the major protein of tight junction :


a. Integrin
b. Laminin
c. Connexin
d. Claudin
e. Nidogen
D

173-The envelop of glycoprotein HIV virus is :


a. P24
b. Gp41
c. P17
d. Gp120
e. Reverse transcriptase
D

174-Which one is not correct about wet ( rural ) type of cutaneous leishmania :
a. Incubation period ranges from few weeks to 2 months
b. Ulcerates rapidly
c. Self healing take 12 months
d. Appears as red furuncle like nodule
e. Ulcer with raised red margin is observed
C

175-Tin- tack sign is observed in :


a. SLE
b. DLE
c. Sarcoidosis
d. Psoriasis
e. Darier
B

176-Phage 2 type 71 staphylococcus aureus is not found in patient with :


a. Lichen plan
b. Psoriatic scales
c. Atopic dermatitis
d. Dermatomysitis
e. Hemodialysis
B

177-One of the following is alkylamine class of antihistamine:


a. Triprolidine HCL
b. Diphenhydramine HCL
c. Hydroxyzine HCL
d. Azatadine maleate
e. Promethazine HCL
A

178-One of the following is not RNA containing:


a. Picorna v.
b. Retrovirus
c. Rhabdo v.
d. Paramyxo v.
e. Parvo v.
E

179-In which of the following disease Kaposi’s varicelliform eruption does not occur :
a. DLE
b. Severe seborrheic dermatitis
c. Scabies
d. Darier disease
e. Burns
A

180-The venereal disease with shortest incubation period is:


a. Syphilis
b. Chlamydia urethritis
c. Pinta
d. Chancroid
e. Mycoplasmal infection
D

181-One of the following is not correct about the side effect of amphotericin B :
a. Hypersensitivity reaction including anaphylaxis
b. Febrile reaction
c. Microchromic anemia
d. Decreased potassium and magnesium serum level
e. Renal dysfonction
C

182-Non inflammatory types of tinea capitis is caused by :?


a. MicrosporumGypseum
b. TrichophytonViolaceum
c. TrichophytonMentagraphytes
d. MicrosporumCanis
e. Trichophytonrubrum
B

183-Witch One of the following is negative in Wood’s light :?


a. Microsporumauduinii
b. MicrosporumGypseum
c. Trichophytonschoenleinii
d. Porphyreas
e. Trichophytontonsurans
E

184-One of the following tetracyclins is not excreted through glomerular filtration :


a. Minocycline
b. Demeclocycline
c. Clomocycline
d. Doxycycline
e. Methacycline
D

185-One of the following is not correct about cyclosporin :


a. It binds to cyclophilin , that inhibits cytoplasmic calcineurin
b. Dephosphorylation prevents the normal translocation of NF-AT ( NUCLEAR FACTOR OF
ACTIVATED T- CELLS ) from cytoplasm to nucleus
c. Activate production of IL-2
d. NK cells and B- cells are affected by cyclosporine
e. The effects of Tacrolimas are similar to those of cyclosporin
C

186-One of the following cytokines is not produced by keratinocytes :


a. Il-1
b. TNF -beta
c. Il- 6
d. TNF ALFA
e. TGF –beta
B

187-One of the following is not correct about staining:


a. Masson trichrome stains elastic fibers
b. Alcian blue PH2.5 stains acid MPS
c. VERHEOFF Van gieson stains elastic fibers
d. Scarlet red stains lipid
e. Oil red O stains lipid
A
MT : collagen, msc
188-S-100 is positive in one of following tumors :
a. SCC
b. Atypical fibroxanthoma
c. Melanoma
d. Leiomyosarcoma
e. Angiosarcoma
C

189-Complement receptor 1 ( CR1, CD35 ) IS NOT EXPRESSED ON ONE OF THE


FOLLOWING :
A. NEUTROPHILS
B. MACROPHAGE
C. T cells
D. B cells
E. Follicular dendretic cells
E

190-ONE OF THE FOLLOWING IS NOT EXPRESSED BY BASOPHILS :


A. Leukotriens
B. Il-4
C. Il-3
D. IgE
E. Prostaglandin
C

191-One of the following is not correct q nail plate and its layers and nail bed :
a. Calcium is significantly contributed to the hardness of the nail
b. Contain significant amount of phospholipid
c. The keratin of nail bed is k5/ k14
d. It has thin dorsal lamina
e. It has thicker intermediate lamina
A

192-Sample epithelia are characterized by the keratin pair :


a. K5/k15
b. K5/k14
c. K8/k18
d. K1/k10
e. K7/k17
C

193-A skin biobsy from a 35 year old patient shows suprabasal acantholytic cleft and negative
immunofluorescence. Which of following clinical features is most likely present in the patient ?
a. Tense bullae in the abdomen
b. Crusted erosions in axillae
c. Greasy papular lesions in chest
d. Non blanchable erythematous macules
e. Dermatitis reaction in the face due to aftershave lotion
B hailyhaily

194-Which of following cysts contains lobules of sebaceous glands:


a. Milium
b. Pilomatricoma
c. Epidermal cyst
d. Tricholemmal cyst
e. Steatocystoma
E

195-CD207 is a sensitive and specific marker of :


a. Helper T cell
b. Cytotoxic cell
c. Merkel cell
d. Langerhans cell
e. Vascular endothelial cell
D(langrine)

196-Which one is accurately matched ?


a. Karyorrhexis – fragmentation of cllagen fibers
b. Polymorphism – variability in the apperance of nuclei of the same cell type.
c. Kamino bodies – eosinophilic rounded body resulting from degeneration and death of
keratinocytes
d. Theque – collections of naevus cells at and in region of the dermoepidermal junction
e. Munro microabscesses – collection of neutrophil polymorphs in thesratumcorneum
E

197-Which one is a clinical presentation of spongiosis:


a. Erythema
b. Wheal
c. Oozing (weeping)
d. Lichenification
e. Lichenoid papul
B

198-At the surface of Earth, the sense of wormth results from WHICH of the following :
A. 100-290 nm
B. 290-400 nm
C. 400-760 nm
D. 760-1440 nm
E. 1440 nm – 1 mm
E

199-Prolyl hydroxylation reaction is the initial step in the biosynthesis in WHICH of the
following :
a. Collagen
b. Elastin
c. Keratin sulphate
d. Hyaloronic acid
e. Keratin
A

200-Leptin is involved in:


a. Keratinisation
b. Corneodesmolysis
c. Energy homeostasis
d. Induction of hair telogen phase
e. Repair of dermoepidermaljonction
C

201-Which cell is a major source of angiotensin converting enzyme?


a. Pericyte
b. Veil cell
c. Merkel,s cell
d. Endothelial cell
e. Smooth muscle cell
D

202-Large clear cells, small dark cells and myo-epithelial cells make WHICH of the following :
a. Peripherial cells of sebaceous gland
b. Main duct of sebaceous gland
c. Secretory coil of the eccrine sweat gland
d. Secretory coil of the apoccrine sweat gland
e. Duct of the eccrine sweat gland
C

203-Phage 2 type 71 staphylococcus aureus is not found in patient with :


a. Lichen plan
b. Psoriatic scales
c. Atopic dermatitis
d. Dermatomysitis
e. Hemodialysis
B

204-Which one is not correct about wet ( rural ) type of cutaneous leishmania :
a. Incubation period ranges from few weeks to 2 months
b. Ulcerates rapidly
c. Self healing take 12 months
d. Appears as red furuncle like nodule
e. Ulcer with raised red margin is observed
C

205-One of the following is alkylamine class of antihistamine:


a. Triprolidine HCL
b. Diphenhydramine HCL
c. Hydroxyzine HCL
d. Azatadine maleate
e. Promethazine HCL
A

206-One of the following is not RNA containing:


a. Picorna v.
b. Retrovirus
c. Rhabdo v.
d. Paramyxo v.
e. Parvo v.
E

207-In which of the following disease Kaposi’s varicelliform eruption does not occur :
a. DLE
b. Severe seborrheic dermatitis
c. Scabies
d. Darier disease
e. Burns
A

208-The venereal disease with shortest incubation period is:


a. Syphilis
b. Chlamydia urethritis
c. Pinta
d. Chancroid
e. Mycoplasmal infection
D

209-One of the following is not correct about the side effect of amphotericin B :
a. Hypersensitivity reaction including anaphylaxis
b. Febrile reaction
c. Microchromic anemia
d. Decreased potassium and magnesium serum level
e. Renal dysfonction
C

210-Non inflammatory types of tinea capitis is caused by :?


a. MicrosporumGypseum
b. TrichophytonViolaceum
c. TrichophytonMentagraphytes
d. MicrosporumCanis
e. Trichophytonrubrum
B

211-A biopsy from a lesion on the face of an adult patient shows dermal palissades of cells and
stroma. Which one is most likely to be present in the lesion ?
a. Positive diascopy
b. Ichtyosiform scale
c. Foamy histocytes
d. Positive congo red stains
e. Telangiectasia
A

212-Which disorder is transmitted through healthy female carriers?


a. Phenylketonuria
b. Xeroderma pigmentosum
c. Acrodermatitis enteropathica
d. Hereditary haemorrhagic telangiectasia
e. Fabry’s diseaes
E

213-Which one is a feature of Netherton syndrome?


a. Distinctive face
b. Premature ageing
c. Trauma – induced blistering
d. Congenital erythroderma
e. Conical teeth
D

214-Multiple epidermoid cysts, fibrous tissue tumors, osteomas and polyposis of the colon are
features of ?
a. Cowden disease
b. Gardner’s syndrome
c. Papillon – Lefevre syndrome
d. Cronkhite- Canada syndrome
e. Goltz syndrome
B

215-Multiple epidermoid cysts, fibrous tissue tumors, osteomas and polyposis of the colon are
features of ?
a. Cowden disease
b. Gardner’s syndrome
c. Papillon – Lefevre syndrome
d. Cronkhite- Canada syndrome
e. Goltz syndrome
B

216-A 50 year patient, with multiple nodular lesion on the face, trunc and proximal limbs , of long
duration,. Biopsy shows aggregates of epitheliod cells, mulinucleate cells and few surronding
lymphocytes. What is the next management step?
a. X-ray chest
b. Treat by potent topical steroid
c. CBC
d. Treat by topical nitrogen mustard
e. Confirm the diagnostic by detailed exam of urogenital tract.
A

217-A 4y. old child presents with a 3 cm red nodular lesion on the back. Biopsy shows spindle
shaped granulated cells staining with toluidine blue. Which one is the most expected course?
a. Dissemination to internal organs
b. Breaking down into an ulcero- vegetative lesion
c. Spreading to lymph nodes
d. Spontaneous resolution over childhood period
e. Diffuse cutaneous involvement
D

218-On which sites the location of bruises suggest non- accidental trauma?
a. Elbows
b. Knee
c. Medial side of thigh
d. Front of lower legs
e. Forehead
C

219-A 10 y. child with yellowish plaque with velvety surface on scalp. Biopsy shows Mature
sebaceous glads, papillomatous hyperplasia of overlying epidermis, inconspicous hair follicles
and buds of undifferentiated epithelial cells. What is the appropriate management?
a. Ct scan brain
b. Removal of the lesion
c. Potent topical steroid
d. Topical nitrogen mustard
e. Reassurance
B

220-A 20 y. patient with an inherited susceptibility to human papilloma virus infection presents
clinically as :
a. Perforating keratotic disorder
b. Punctuate keratoderma
c. Epidermodysplasia verruciformis
d. Multiple minute digitate hyperkeratosis
e. Acrokeratosis verruciformis
C

221-Which of following is a feature of dermatophytide( ide) reaction?


a. Lack of dermatophytesin lesions
b. Close location to the original infection
c. Persistance of lesions after clearance ofdermatophytosis
d. Restricted to microsporumcanis infection
e. Immunosuppressed patient
A
222-A napped claires is a well recognized feature of :
a. Parapsoriasis
b. PRP
c. Darier disease
d. Lymphomatoid papulosis
e. Reiter syndrome
B

223-The presence of a mixed population of cells bearing different genetic characteristic leading
to phenotypic diversity is referred to as:
A. Balanced translocation
B. Mosaicim
C. Reciprocal translocation
D. Heteroplasty
E. Uniparental meroisodisomy
B

224-Which one is a well recognized Th 1- promoting factor :


a. IL-17
b. IL-23
c. IL-2
d. IL-12
e. IL-4
C

225-Th 17 is distinguished by production of :


a. TNF- alfa
b. IL- 17
c. IL – 4
d. IL- 23
e. Interferone – gamma
B

226-A central parakeratosis without hypergranulation is a characteristic histopathologic feature


of :
a. Lichen nitidus
b. Pityriasis lichenoide
c. GVHD
d. PRP
e. Parapsoriasis
B

227-Tetracyclines have anti-inflammatory effects which include:


A. Increasing matrix metalloproteinase activity
B. Inhibit leukocyte chemotaxis
C. Increase production of TNF a
D. Increase production of IL 1B
E. Decrease production of IL 1B
B

228-When interpreting a biochemistry of a loricrin , you will find :


a. A cysteine- rich protein
b. A lysine- rich protein
c. An arginine- rich protein
d. A histidine- rich protein
e. A prolin- rich protein
A

229-Which mutatio is responsible for AR lamellar ichtyosis?


a. TTDN1
b. BLM
c. TGM1
d. ATP2C1
e. DHCR7
C

230-Which one is a well recognized cause of Darier disease?


a. Mutation in SERCA1
b. Mutation in ATP2C1
c. Mutation in ENTACTIN
d. Mutation in PECAM-1
e. Mutation in TGM1
B

231-Which one is a common component of keratohyalin granules ?


a. Profillaggrin
b. Fibrillin
c. Plakophilin
d. Laminin
e. Connexin
A

232-hich one is restricted to Merkel cell ?


a. K8
b. K18
c. K19
d. K20
e. K25
D

233-The basal layer is attached to basement membrane zone through :


a. K1 – K9
b. K1 – K10
c. K4 – K13
d. K8 – K18
e. K5 – K14
E

234-Which one is resulting from disruption of desmoglein – 4:


a. naxos disease
b. autosomal recessive hypotrichosis
c. bullous impetigo
d. lethal acantholyticepidermolysisbullosa
e. striate palmoplantarkeratoderma
B

235-Lamellar granules are developed in :


a. Spinous layer
b. Granular layer
c. Stratum corneum
d. horny layer
e. Basal layer
B

236-Which one has more expression of desmoglein –1:


a. Granular layer
b. Dermoepidermal junction
c. Hair follicles
d. Blood capillaries
e. Basal layer
A

237-Which one is significanty increased in DLE:


A. HLA- B7
B. HLA- B8
C. HLA- A2
D. HLA- Dr3
E. HLA- Dr4
E

238-which virus is single stranded RNA :


a. Measles
b. HSV
c. Adenovirus
d. Parvovirus
e. HPV
A

239-which one is true regarding IgM :


a. Cross the placenta
b. Produced in a secondary immune response
c. Found as a diametric form linked by a J chain
d. Can activate complement
e. Usually found on the surface of mast cells
D

240-SKIN DAMAGE DURING IMMUNE- COPLEXE REACTION is mediated by :


a. Histamine
b. Neutrophil activation
c. Macrophage activation
d. T cell activation
e. NK cell activation
D

241-WHICH CYTOKINE IS NOT SECRETED BY T HELPER lymphocytes:


A. IL12
B. IL4
C. IL10
D. IL2
E. IFN- GAMMA
E

242-WHICH CELLS SECRET CYTOKINES THAT STIMULATE IGE PRODUCTION BY B


CELLS, RECRUIT AND ACTIVATE MAST CELLS DURING INDUCTION PHASE OF Type 1
hypersensitivity reaction :
A. MELANOCYTES
B. NK CELLS
C. MACROPHAGES
D. NEUTROPHILS
E. CD4+ LYMPHOCYTES
E

243-A 5Y. BOY known to have sickle cell disease presented with an acue febrile rash and
transient aplastic crisis, which virus is well recognized to cause this presentation ?
a. Herpes simplex
b. Varicella- zoster
c. Parvovirus
d. Rubella
e. Rubeola
C

244-A neonate presented with disseminated infection. Which CSF test is more sensitive to
detect herpes simplex meningitis ?
A. CSF Protein Analysis
b. HSV PCR
C. HSV Culture
D. Tzanck smear
e. HSV IgG antibody
B
245-A 38 year man , HIV + , He asked about the chance of him progressing to symptomatic
AIDS . whitch test would be most usefull?
a. HIV antibody test
b. HIV RT PCR
c. Neopterin
d. HIV
e. CD4 Lymphocyte count
E

246-Failure of phagolysosome formation in neutrophils is well documented in :


a. Skin fragility syndrome
b. POEMS syndrome
c. COSTELLO syndrome
d. Leopard syndrome
e. Chediak- higashi syndrome
E

247-Lactoferrin is well recognized component in :


a. Keratinocyte
b. Melanocyte
c. Neutrophil
d. Fibroblast
e. T- cell lymphocyte
C

248-A central parakeratosis without hypergranulation is a characteristic histopathologic feature


of :
a. Lichen nitidus
b. Pityriasis lichenoide
c. GVHD
d. PRP
e. Parapsoriasis
B

249-Th 17 is distinguished by production of :


a. TNF- alfa
b. IL- 17
c. IL – 4
d. IL- 23
e. Interferone – gamma
B

250-Which one is a well recognized Th 1- promoting factor :


f. IL-17
g. IL-23
h. IL-2
i. IL-12
j. IL-4
IL2

251-A 10 year child with yellowish plaque with velvety surface on scalp. Biopsy shows Mature
sebaceous glads, papillomatous hyperplasia of overlying epidermis, inconspicous hair follicles
and buds of undifferentiated epithelial cells. What is the appropriate management?
f. Ct scan brain
g. Removal of the lesion
h. Potent topical steroid
i. Topical nitrogen mustard
j. Reassurance
Removal

252-On which sites the location of bruises suggest non- accidental trauma?
A. Elbows
B. Knee
C. Medial side of thigh
D. Front of lower legs
E. Forehead
C

253-A 4 year old child presents with a 3 cm red nodular lesion on the back. Biopsy shows
spindle shaped granulated cells staining with toluidine blue. Which one is the most expected
course?
A. Dissemination to internal organs
B. Breaking down into an ulcero- vegetative lesion
C. Spreading to lymph nodes
D. Spontaneous resolution over childhood period
E. Diffuse cutaneous involvement
D

254-One of the following is not true about lepromatous leprosy:


A. Numerous lesions are present macules, papules, nodules.
B. Asymmetrical distribution.
C. Vague border and difficult to distinguish between normal and affected skin.
D. Sensation is not affected.
E. Many bacilli could be found in skin lesion.
B

255-One of the following is not correct about special stains in Dermatology ?

A. Alcian blue (PH 0.05) stains sulfated mucopolysaccharide


B. Masson”s trichrome stains smooth muscle green
C. Periodic acid Schiff stains fungal walls red
D. Pagoda stains amyloid orang
E. Myeloperoxidase stains immature myeloid cell orang
B

256-The commonest clinical feature of infant with rubella is:


a. Cataract
b. Heart defects
c. Deafness
d. Purpura
e. Mental retardation
C

257-Which one of the following is not a feature of major depressive disorder:


a. Disturbance of appetite
b. Depressed mood
c. Psychomotor agitation
d. Thoughts of suicide
e. Panic attacks
E

258-Which one of the following is usually c- ANCA positive :


a. Polymyositis
b. Wegeners granulomatosis
c. Polyarteritis nodosa
d. Mucocutaneous lymph node syndrome
e. Behcet syndrome

259-A 20 y. patient, with genetic disease, with greyish small flaky scals, most distinct on the
extensor surfaces of the arms and lower legs, sparing the flexural creases. The lesion
developed early in childhood. Biopsy showes hyperkeratosis and absent granular layer. Which
one is defected ?
a. Connexins
b. Keratin1 & 10
c. Cholesterol sulphate
d. transglutaminase
e. Filaggrin
E

260-Sample epithelia are characterized by the keratin pair :


A. K5/k15
B. K5/k14
C. K8/k18
D. K1/k10
E. K7/k17
C

261-Concerning stratum corneum lipids, Which one of the following is not true:
a. Human typically comprises about 20 corneocyte cell layers
b. Ceramides accounts 20% of lipids
c. Each corneocyte is surrounded by lipid – enriched ectracellar matrix
d. Free fatty acids constitute 10 – 15 % of stratum corneum lipids
e. Cholestrol is the 2 most abundant lipid by weight in stratum corneum
B

262-For monitoring cyclosporine, Which one of the following is not correct:


a. At least 2 baseline blood pressure reading
b. BUN and creatinie every 2 weeks for 1 – 2 months then monthly
c. Urine analysis monthly
d. Fasting lipid panel every 2-4 weeks for 1 – 2 months then monthly
e. CBC and platelets count every week
E

263-Which one of the following is not correct about hypertrophic scar :


a. The onset is delayed
b. Usually is proceeded by trauma
c. It is confined to wound margin
d. Increased mast cell histologically
e. Contains myofibroblasts
A

264-Which one of the following is not a feature of Cushing syndrome:


a. Sweating
b. Depression
c. Weiht gain
d. Hypertension
e. Myopathy
A

265-About scabies , Which one of the following is not correct:


a. It is caused by highly host - specific eight- legged mite
b. Mites from animals are not a source of human infestations
c. It is too small to be seen by the naked eye
d. A female mite will lay 6 – 10 eggs
e. Eggs require 10 days to mature

266-Regarding wound healing, which of the following is not true:


A. Proliferation phase is the first phase.
B. Platelets, the first cells appear in healing process
C. Depth of the wound determines the degree of contraction.
D. Macrophages are important cells in healing process.
E. The dermal matrix is produced by fibroblast.
B

267-Which one of the following is autosomal recessive condition :


a. Aneuploidy
b. Oculocutaneous albinism
c. Hypohidrotic ectodermal dysplasia
d. Incontinencia pigmenti
e. Darier disease
B

268-The incubation period of tinea nigra is :


a. 3 – 5 days
b. 1 week
c. 10 – 15 days
d. 21 days
e. 60 – 90 days
D

269-Which one of the following mycobacteria is scotochromogen:


a. M. Kansasii
b. M. intercellulare
c. M. tuberculosis
d. M. marinum
e. M. scrofulaceum
E

270-Which of the following is not a feature of eccrine gland:


A. Highest density on plams and soles.
B. It has long thin duct opens into the skin surface.
C. It is associated with terminal hair follicle
D. Present at birth.
E. It has large secretory clear cells.
C

271-KTP laser wavelength is :


a. 308 nm
b. 694
c. 755
d. 532
e. 2940
D

272-Currently, the standard laser used for vascular lesions is:


a. Alexandrite laser
b. Flash- lamp PDL
c. Excimer laser
d. Nd YAG laser
e. Q- switched ruby laser
B

273-Which one of the following about genetics is not true.


a. Nuclear DNA is packed into chromosomes
b. Somaitic and germline cells contain approximately 4 m of DNA
c. The maternal and paternal gamates are haploid cells
d. X and Y chromosomes share only two small regions (pseudoautosomal regions)
e. Karyotype describes the constitution of an individual
E

274-Which of the following concerning basic genetic is false:


A. Meiosis is the process of somatic cell division of proliferating cells leading to genetic material
duplication
B. The location of particular sequence of DNA on chromsome is known as locus.
C. Telomere plays the key role in maintenance of chromsomal integrity.
D. Each individual will have two alleles at any given autosomal locus.
E. Alleles constitute the genotype of an individual.
A

275-Concerning cryotherapy Which one of the following is not true:


a. Liquid nitrogen is the coldest cryogenic agent
b. Slow cooling produces extracellar ice formation
c. Thawing should proceed rapidlyx
d. It can be used by cotton tiped applicator
e. It can be used in the treatment of cutaneous malignancy
C

276-Which one of the following antifungal activity is dependant on cytochrome 450 :?


a. ketoconazole
b. terbinafine
c. amphotericine
d. fluconazole
e. griseofulvin
A

277-The dorsal nail plate is produced by :


a. Proximal matrix
b. Nail bed
c. Nail fol
d. Hyponychium
e. Distal matrix
A

278-Which of the following cells contain neuroendocrine peptides:


A. Keratinocytes.
B. Langerhans cells.
C. Merkel cells
D. Melanoctes.
E. Fibroblast.
C

279-Which one of the following blocks DNA synthesis:


a. Polyamines
b. Imidazoles
c. Flucytosine
d. Triazoles
e. Allylamines
C

280-Staining of the dermatophytes with PAS will reveal branched hyphae colored as:
A. Brown-black.
B. Red-purple
C. Blue.
D. Yellow-orange.
E. Green-blue.
B

281-Which one of the following is CONCERNED with heat regulation:


a. Arteries
b. Veins
c. Lymphatics
d. Venous plexus
e. Capillaries
D

282-Melanocyte develops at:


A. 4 weeks of EGA.
B. 12 weeks of EGA
C. 16 weeks of EGA.
D. 20 weeks of EGA.
E. 24 weeks of EGA.
C

283-Which one of the following returns leukocytes and interstitial fluids :


a. Arterioles
b. Lymphatics
c. Subcutaneous venous plexus
d. Capillaries
e. Arteriovenous anastomosis
B

284-Which one of the following is the most commun cause of erythema nodosum in chidren:
a. Dapsone
b. Streptococcal infection
c. IBD
d. Sarcoidosis
e. Amyloidosis
B

285-Which one of the following proteins has antibodies in Relapsing polychondritis:


a. Collagen type I
b. Collagen type II
c. Collagen type III
d. Collagen type IV
e. Collagen type XI
B

286-A 14 month old boy has right congenital glaucoma , seizures and port-wine stain over right
facial side. The most likely diagnosis is:
a. Nodding syndrome
b. Sturge - Weber syndrome
c. Maffucci syndrome
d. Parkes – Weber syndrome
e. Proteus syndrome
B

287-Which one of the following binding protein is essential in the mechanism of action of
Tacrolimus :
a. Fibronectin- binding protein
b. Immunophilin- binding protein
c. Lipopolysaccharide- binding protein
d. Retinol - binding protein
e. Albumin
B

288. A 15 year old boy presented with dermatitis involving the whole face. His dermatologist
gave him betamethasone valerate cream twice daily for 1 week. How much grams he should
apply for each application:
a. 1 gram
b. 3 grams
c. 5 grams
d. 7 grams
e. 10 grams
B

289-Which one of the following does not cause immunologic granuloma:


a. Beryllium
b. Silica
c. Zirconium
d. Cadmium
e. Mercury
E

290-A gardener presented with non-immunologic contact urticaria. Which one of the following
does not cause non-immunologic contact urticaria :
a. Benzoic acid
b. Cinnamic aldehyde
c. Nicotinic acid esters
d. Natural rubber latex
e. Sorbic acid
E

291-Which one of the following is not true regarding Doxepin :


a. It is a tricyclic antidepressant
b. It is a potent H1 receptor - blocker
c. It is a potent H2 receptor - blocker
d. It is a mast cell stabilizer
e. It is a inhibits he reuptake of serotonine
D

292-Which one of the following does not cause scleroderma – like disorder :
a. Bleomycine
b. Vinyl chloride
c. Silicosis
d. Tolerization
e. Fosinopril
D

293-A 34 year old female patient presented with extensive LP and liver disease, Which one of
the following drug is recommended to treat this patient :
a. Prednisolone
b. Prednisone
c. Acitretine
d. Methotrexate
e.PUVA
A

294-Which one of the following drugs cause chronic myelogenous leukemia :


a. Imatinib
b. Interferon
c. Hydroxyurea
d. Busulfan
e. Dapsone
D

295-Which one of the following monoclonal antibodies is directed against CD20:


a. Epratuzumab
b. Rituximab
c. Ustekinomab
d. Golimumab
e. Infleximab
B

296-Which one of the following side effects is not caused by thalidomide:


a. Somnolence
b. Constipation
c. Motor neuropathy
d. Thromboembolism
e. Irregular heartbeat
C

297-In localized acute cutaneous LE, WHICH of following is characteristically spared :


a. Forehead
b. Nasolabial fold
c. Chin
d. V- area of the neck
e. Ears
B
298-Which one of following investigations is necessary before starting quinacrine therapy :
a. Calcium profile
b. G6PD
c. Lipid profile
d. Chest X- ray
e. Urine analysis
B

299-The carpet tack sign is a prominent feature in one of following skin disease :
A. Sarcoidosis
B. DLE
C. Follicular mucinosis
D. Nodular amyloidosis
E. Ashy dermatosis
B

300-WHICH hormone is increased in female patients of SLE :


a. Testosterone
b. Dihydrotestosterone
c. High potency estrogen
d. DHEA
e. Prolactin
C

301-Lactoferrin is well recognized component in :


a. Keratinocyte
b. Melanocyte
c. Neutrophil
d. Fibroblast
e. T- cell lymphocyte
C

302-Which of the following TLR is involved in recognizing lipopolysaccharides:


F. TLR 2
G. TLR 3
H. TLR 4
I. TLR 5
J. TLR 7
H

303-Which of the following is part of the alternative complement cascade system: ?


F. C 1
G. C1 INH
H. C3
I. C5b
J. anaphylatoxin
C3

304-Which of the following bind to cell membrane associated proteins in adherens junctions:
F. Plectin
G. Actin
H. Alpha – catenin
I. E – cadherin
J. Myosin
4

305Which of the following is able to cross the placenta:


F. IgA
G. IgD
H. IgE
I. IgG
J. IgM

306-What is the spectrum of UV A2 light:


F. 400 – 460
G. 340 – 400
H. 315 – 340
I. 290 – 315
J. 200 - 290

307-5 . alpha – reductase converts testosterone to Which of the following:


F. Andostenedione
G. DHT
H. DHEA
I. 17- alpha hydroxyprogestrone
J. 17- alpha hydroxypregnenoline
2

308-One of the following is needed by tyrosinase to function:


F. Zn
G. Fe
H. Selenium
I. Mg
J. Cu
5

309-One of the following cytokines is involved in stimulating adaptive immunity:?


F. IL- 1
G. IL- 2
H. IL- 10
I. IL- 12
J. IFN- alpha
IL10
2
2 ‫نختار‬
Adaptive: 22-2-23-4-17-12-13
310-Which of the following is the most potent anaphylatoxin?
G. C1
H. C3a
I. C3b
J. C5a
K. C5b
C5a

Most abundant C3a

311-At which earliest gestational age is epidermal development most likely to be complete?
F. 12 weeks.
G. 15 weeks.
H. 20 weeks.
I. 22 weeks
J. 28 weeks.
22wk

312-Langerhans cells do not express Which of the following markers :


F. FcRI
G. CD1a
H. MHCII
I. CD54
J. CD80/86
4

313-Which of the following is FALSE about melanin synthesis:


F. Pheomelanin has oval – shaped structure
G. Melanin precursors are dependant on copper-requiring enzyme
H. Eumelanin is brown or black in color
I. Tyrosinase is the rate-limiting step in melanin synthesis
J. Chronic sun exposure causes larger melanosomes to be created
1

314-Pain is detected by Which of the following:


F. Pacini corpuscles
G. Meissner corpuscles
H. Ruffini corpuscles
I. C – type fibers
J. Merkel cells
4

315-One of the following Antigene – presenting cell surface molecules interacts with the T-cell
surface molecule LFA-1:
F. ICAM-1
G. CD40
H. MHC 2
I. CD80/86
J. LFA-3
1

316-Langerhans cells originate from:


A. Ectoderm
B. Neural crest
C. Mesoderm
D. Bone marrow
E. Endoderm
D

317-Granulation tissue is initially composed of which of the following:


F. Collagen I
G. Collagen II
H. Collagen III
I. Collagen IV
J. Collagen VI
3

318-The protein product of the gene PTCH1 is which of the folowing:


F. A TLR
G. A membrane- bound receptor
H. Receptor kinase
I. A G- protein- bound receptor
J. A ligand
2

319-Which of the following is implicated in the pathogenesis of keloid formation:


G. IL – 1
H. IL – 2
I. IL – 6
J. IFN – gamma
K. TGF – beta
5

320-which one of the following is most closely associated with psoriasis:


A. HLA – B8
B. HLA – DR2
C. HLA – DQ8
D. HLA – DR6
E. HLA – Cw6
E

321-What cells are part of the innate immune system:


F. NK cells
G. langerhans cells
H. Th1 cells
I. Th2 cells
J. B cells
1

322-What makes up the majority of the protein envelope of epidermal keratinocytes:


F. Loricrin
G. Involucrin
H. Envoplakin
I. Fillagrin
J. Transglutaminase
Loricrin

323-At how many weeks does the fetal basement membrane begin to develop:
F. 3 weeks
G. 7 weeks
H. 9 weeks
I. 12 weeks
J. 16 weeks
7 wk

324-What is the most abundant amino acid in collagen?


F. Proline
G. hydroxy proline
H. lysine
I. hydroxylysine
J. glycine
Glycine

325-Which of the following is associated with more severe psoriatic disease:


F. Beta-defensin 1
G. Beta-defensin 2
H. Beta-defensin 4
I. Beta-defensin 5
J. Beta-defensin 6
2

326-Which one of the following are markers on dendritic cells:


F. CD20
G. CD 11a
H. CD 86
I. CD 8
J. CD 1a
5

327-One of the following is MOST closely related to melanoma:


F. p16
G. p53
H. PTCH1
I. p24
J. Lewis Y antigene
1

328-Eosinophils are activated by which interleukin :


F. IL -1
G. IL –2
H. IL –3
I. IL –4
J. IL -5
5

329-One of the following is not a pre-formed mediator of mast cells:


F. Tryptase
G. Leukotriene C 4
H. Heparin
I. Histamine
J. Cathepsine
2

330-To which protein do intermediate filaments bind in hemidesmosomes:


F. Laminin 5
G. Integrin alpha 6 beta 4
H. Plakoglobin
I. Beta catenin
J. Plectin
5

331-What is the main fat in the cornified layer:


A. Cholestrol
F. Free fatty acids
G. Loricrin
H. Ceramides
I. Involucrin
4

332-Which CYTOKINE is NOT secreted by T helper lymphocyte:


A. IL12
B. IL4
C. IL10
D. IL2
E. IFN- gamma
1

333-Which one is true regarding IgM :


a. Cross the placenta
b. Produced in a secondary immune response
c. Found as a diametric form linked by a J chain
d. Can activate complement
e. Usually found on the surface of mast cells
D

334-Of the following the most common manifestation of Tuberous Sclerosis is:
F. Cardiac arrhythmias
G. Mental retardation
H. Seizures
I. Enamel pitting
J. Visual loss caused by retinal astrocytoma
4

335-Livedo reticularis in patients with Systemic Lupus Erythematous and anticardiolipin


antibodies has been associated with :
F. Arthritis
G. Serositis
H. Cerebrovascular disease
I. Oral ulcers
J. Photosensitivity
3

336-Serum theophylline level is affected by the following drugs EXCEPT:


F. Erythromycin.
G. Cimetidine.
H. Ketoconazole.
I. Pentoxyphilline
J. Rifampicin.
4

337-Pyogenic granuloma: which of the following statements is FALSE?


F. Commonly occurs in the gingival, lips or fingers.
G. Usually develops at the site of preexisting injury.
H. May appear within a preexisting nevus flammeus.
I. With multiple lesions has a predilection for the interscapular region.
J. Drug-indeced pyogenic granuloma tend to persist after withdrawal of the causative drug
5

338-In Buschke – Loewenstein tumour one statement is TRUE:


F. Tends to appear in circumcised men.
G. Is said to represent 80% of 90% of all penile cancers.
H. Does not seem to show a preference for either circumcised or uncircumcised men.
I. Is said to represent 5% to 26% of all penile cancers
J. Radiotherapy is the statement of choice.
4

339-In distinguishing alopecia areata from trichotillomania under microscope, One of the
following favors alopecia areata ?
F. Multiple catagen hairs
G. Pigment casts
H. Granulomatous inflammation
I. Eosinophils
J. Follicular plugging
4

340-The most common early symptom in melanoma is:


F. Tenderness
G. Pruritus
H. Bleeding
I. Ulceration
J. Elevation
5

341-Renal involvement in Henoch-Schonlein purpura:


F. Is second, after skin, in frequency of appearance.
G. Tends to occur early in the course.
H. Never progresses to renal failure if the only presenting sign is microscopic hematuria.
I. Is unlikely to develop later if renal manifestations are absent in the first episode of HSP
J. Is always associated with linear deposite of IgA in kidney biopsy specimens.
4

342-Each of the following is a variant of Pyoderma Gangrenosum except?


F. Bullous
G. Pustular
H. Psoriasiform
I. Vegetative
J. Ulcerative
3

343-Keratosis follicularis (Darier’s disease) is associated with each of the following nail changes
EXCEPT:
F. “V-shaped” notch at the nail plate’s distal free edge.
G. Splinter hemorrohages.
H. Proximal nail fold keratotic papules.
I. “V-shaped” lunula
J. Longitudinal dyschromic lunular bands.
4

344-Verrucous carcinoma is probably best considered as:


F. A typical SCC that happens to display a warty clinical morphology
G. A melanoma with a warty clinical morphology
H. A distinct clinicopathologic variant of low-grade SCC
I. A verrucous form of malignant acanthosis nigricans
J. A distinct variant of high -grade SCC
3

345-Clinically , The initial stage of involvement of skin with systemic sclerosis characterized by?
F. Induration
G. Erythema
H. Edema
I. Sclerosis
J. calcification
3

346-A severe eczematoid dermatitis that often resemble atopic dermatitis associated with
thrombocytopenia is characteristic finding in:
F. Ataxia Telangiectasia
G. Wiskott-aldrich syndrome
H. Severe combined immunodeficiency
I. Common variable immunodeficiency
J. X – linked agammaglobulinemia
2

347-Each of the following is true of the occurrence of malignancy in patients with Pyoderma
Gangrenosum except:
F. Occurs in 7% of patient
G. Leukemia is the most common reported malignancy
H. The prognosis of Pyoderma Gangrenosum is good
I. IgA myeloma has been associated with Pyoderma Gangrenosum
J. Bullous Pyoderma Gangrenosum is the most commonly associated
3

348-After the skin, the 2d organ mostly involved by mast cell disease is :
F. Bone
G. Gastro-intestinal tract
H. Brain
I. Liver
J. Kidney
1

349-The bacterial cell wall is not the site of action of:


F. Penicillin
G. Monobactams
H. Sulfonamides
I. Vancomycin
J. Cabapenems
3

350-Epidermal melanin unit refers to:


F. 36 melanocytes+ 1 keratinocyte
G. 1 melanocytes+ 36 keratinocyte
H. 1 melanocytes+ 36 langerhans cell
I. 36 melanocytes+ 1 langerhans cell
J. 36 melanocytes+ 1 merkel cell
2
351-Which of the following disease presents most often with palpable purpura:
F. Systemic Lupus Erythematous
G. Dermatomyositis
H. Wegener granulomatosis
I. Rheumatoid Arthritis
J. Churg- straus syndrome
3

352-One of the following does not bind to C1q:


F. IgG 1
G. IgG 2
H. IgG 3
I. IgG 4
J. All of the above bind to C1q
4

353-The basic molecular unit of elastin is:


F. Oxytatan
G. Eluanin
H. Tropoelastin
I. Desmosines
J. Profilaggrin
3

354-The elastic fibers are highest in number in:


F. Skin
G. Lung
H. Aorta
I. Achilles tendon
J. Liver
1

355-Major sites of stem cells:


F. Cortex
G. Cuticle
H. Huxeley’s layer
I. Henle’s layer
J. Bulge of ORS
5

356-One of the following layers is not present in vellus hair:


F. Medulla
G. Cortex
H. Cuticle
I. Huxeley’s layer
J. Henle’s layer
1
357-One of the following histamine H1 antagonists has an ulcerogenic effect:
F. Diazoline
G. Loratadine
H. Suprastine
I. Dimedrol
J. Acrivastine
1

358-What next test should be performed on the patient with neurofibromatosis type I :
F. Blood pressure
G. Urinalysis
H. EKG
I. Spirometry
J. Pulse oximetry
1”

359-Biopsy of suspected alopecia areata would show inflammation around which portion of the
hair follicle.:
A. Hair bulb
B. Infundibulum
C. Isthmus
D. Bulge
E. Matrix
1

Plasma cells are not conspicuous in the infiltrate of:


A. Syringocytadenoma papilliferum
B. Early syphilis
C. Granuloma inguinale
D. Rhinoscleroma
E. Lichen planus
E

360-How do the histopathologic features of the cutaneous lesions of Lichen differ from the
mucous membranes lesions of the same entity?
F. Hyperkeratosis is greater in oral lesions.
G. Epidermal atrophy is more frequent in cutaneous lesions.
H. Parakeratosis is present in oral lesions
I. There are greater numbers of eosinophils in oral lesions.
J. Plasma cells are not commonly seen in oral lesion.
3

361-In a patient diagnosed to have ecthyma gangrenosum, what is the pathophysiology of the
formation of skin lesion?
F. Infection of the epidermis and superficial dermis
G. Embolic event
H. Perivascular invasion by organism with release of endotoxin
I. Invasion of bacteria to the vessel lumen
J. Destruction of dermis and subcutaneous tissue by bacterial endotoxin
3

362-Biopsy of suspected DLE would show inflammation around which portion of the hair follicle.:
F. Hair bulb
G. Infundibulum
H. Isthmus
I. Bulge
J. Matrix
3

Which one is significanty increased in DLE:


A. HLA- B7
B. HLA- B8
C. HLA- A2
D. HLA- Dr3
E. HLA- Dr4
5

363-One of the following cytokines is involved in stimulating adaptive immunity:?


F. IL- 1
G. IL- 2
H. IL- 10
I. IL- 12
J. IFN- alpha
3

364-Which HPV is associated with verrucous carcinoma?


F. HPV 6.
G. HPV 16
H. HPV 32.
I. HPV 1.
J. HPV 3.
1

365-Which HPV strain is most closely associated with periungual squamous cell cacrinomas?
A. HPV 6
B. HPV 16.
C. HPV 32.
D. HPV 1.
E. HPV 3.
2

366-The most common associated malignancy in Dermatitis Herpetiforms is?


F. GI lymphoma
G. Gastric adenocarcinoma.
H. Colon adenocarcinoma.
I. Breast adenocarcinoma.
J. Castelman’s.
1

367-In Which of the following the autoantibodies to type XVII collagen is not Associated with.?
F. Bullous pemphigoid.
G. Cicatricial pemphigoid.
H. Pemphigoid gestationis.
I. Linear IgA disease.
J. None. All are associate with type XVII collagen
5

368-The following is not pregnancy category C:


F. Trimethoprim - sulfamethoxazole
G. Cyclosporine
H. Efalizumab
I. Cephalosporins
J. Benzoyl peroxide
4

269-Which retinoid is not excreted in the urine:


F. Tretinoin
G. Isotretinoin
H. Acitretin
I. Bexarotene
J. Etretinate
4

370-Which of the following vehicules provides the greatest hydration for the skin:
F. Solution
G. Gel
H. Lotion
I. Ointment
J. Cream
4

371-One of the following is not true about HS:


A. Primary lesion is vesicle
B. Neuralgia is frequent
C. Junctional mucosa is affected
D. Eczema herpeticum occurs
A. Recurrances are common
2

372-Which of the following does not cause photo-onycholysis:


F. Tetracyclines.
G. Amiodarone
H. Photodynamic therapy.
I. Ciprofloxacin.
J. Psoralens.
2

373-Which of the following is the longest acting systemic corticosteroid:


G. Cortisone
H. Prednisone
I. Methyl prednisone
J. Dexamethasone
K. Triamcinolone
5

374-Which of the following treatment of head lice infestation has demonstrated no resistance:
F. Permethrin cream 1%
G. Permethrin cream 5%
H. Lindane shampoo 1%
I. Pyrethrin
J. Ivermectin
5

375-Which of the following is not characteristic of telogen effluvium?


F. Associated with a telogen count > 20%.
G. May be precipitated by infections, stress, hypothyroidism, anticonvulsants and
antirthypertensice medications.
H. Hair loss usually begins immediately after a precipitating event
I. Eventual spontaneous hair regrowth is expected.
J. All of the above are true.
3

376-Which of the following characterize telogen hair?


F. 60% of hairs in this cycle at any time.
G. Hair Growth Phase.
H. Duration of cycle is 3 months
I. Inner Root Sheath is Lost.
J. Bulb regression.
3

377-Which of the following statements regarding lice is NOT true?


F. Eggs are laid close to the scalp.
G. Eggs easily slide off the hair shaft
H. Infestation can be asymptomatic.
I. When full grown these insects can survive for 36 hours away from their host.
J. Transmission can occur through linens.
2

378-Which of the following regarding hair growth during fetal and neonatal development is
false:

F. Human hair follicule morphogenesis takes place only once


G. Terminal hairs initially grow in a synchronous and wave – like pattern anteriorly to posteriorly
H. Pigmented lanugo hair is shed in an anterior to posterior wave at 7 – 8 months of gestation
I. Shorter unpigmented lanugo hair is shed 3 – 4 months after birth
J. Pigmented hair on the scalp are shed in an anterior to posterior wave postnatallyl
2 👍 👍 bolognia p1146

379-Epithelial stem cells of hair follicle arise from where?


F. Rapidly proliferating keratinocytes from the hair matrix.
G. Lowermost portion of the hair follicle.
H. Segment of the outer root sheath located near the insertion of the arrector pili muscle
I. Follicular dermal papilla.
J. Mesenchymal connective tissue follicular sheath that is attached to the hair follicle basement
membrane.
3

380-Which of the following is the most accurate description of the exogen phase of the hair
cycle:
F. The shedding of the telogen hair from the hair follicule
G. Growth phase of the hair follicule
H. Keratinocyte apoptosis of lower 2/3 rds of the hair follicule
I. Hair follicule resting phase
J. Empty telogen hair follicule phase
1

381-Which of the following is false: ?


F. Dark hair has a predominance of eumelanin
G. Red hair has a predominance of pheomelanin
H. Hair fiber strength is largely due to disulfide bonding
I. Estrogens increase hair growth rate
K. All of the above are true
4

382-The topical therapy that can achieve the longest period of remission in psoriasis is ?
F. Calcipotriol
G. Tar
H. Mild steroids
I. Potent steroids
J. Anthraline
1

383-Melanoma most frequently develops:


F. In a nevus spilus
G. In a congenital nevus
H. In a dysplastic nevus
I. In a blue nevus
J. De novo
3
384-Indicators reflecting a poor prognosis regarding Dermatomyositis include all of the following
except?
F. Asthenia
G. Myositis associated with connective tissue disease
H. Fever
I. Dysphagia
J. Leukocytosis
2

385-Scrotal tongue has been associated with all of the following except?
A. Down ‘s syndrome
B. Melkersson – Rosenthal syndrome
C. Mongoloids
D. Geographic tongue
E. Lepromatous leprosy
3

386-The most common complication of erythropoitic protoporphyria is:


F. Renal failure
G. Cholelithiasis
H. Mental status changes
I. Hypertrichosis
J. Cardiomyopathy
2

387-The following mucosal manifestation are known to occur in sarcoidosis except?


A. Xerostomia
B. Salivary gland swelling
C. Gingival swelling
D. Leukoplakia
E. Swelling of the lachrymal and parotid glands and facial palsy
4

388-Gingival hyperplasia is caused by the following except?


F. Phenytoin
G. Cyclosporine
H. Nifedipine
I. Diltiazem
J. Lithium
5

389-The following statements are true regarding phototoxic eruptions except?:


F. Blister and desquamation may occur in severe cases
G. Clinically, it resembles a severe sunburn reaction
H. The eruptions may be followed by persistent hyperpigmentation
I. Dacarbazine, fluorouracil and vinblastine are common causative agents
J. It is sharply demarcated from uninvolved areas
3👍 👍
Not persistent
Fitz

390-Which of the following topical therapy may be effective for keratosis lichenoid chronica?
F. Calcipotriol
G. Triamcinolone
H. Tretinoin
I. Permethrin
J. Metronidazole
1 acc to CME

391-Hair loss may be caused by all of the following except?


A. Contraceptive Pills
B. Heparin
C. Phenytoin
D. Retinoids
E. Cyclophosphamide
C

392-Among the following lesions, which of the most likely to turn malignant?
F. Nevus comedonicus.
G. Verrucous epidermal haevus.
H. Nevus sebaceous
I. Sebaceous adenoma.
J. Steatocytoma multiple
3

393-Sister Joseph’s nodule of the umbilicus represents a metastasis from primary cancer in the
following organs. EXCEPT:
F. The stomach.
G. The ovary.
H. The intestines.
I. Pancreas.
J. Kidney
5

394-Regarding breast cancer in males, mark the WRONG statement:


F. The incidence approaches that in women in cases of klinefelter’s syndrome.
G. It is usually related to wearing braces (suspenders)
H. It can be associated with previous exposure to radiation.
I. It is likely to produce nipple inversion.
J. As in women the essential features is a breast mass
2

395-In SCC of lower lip, mark the wrong ?


F. It is rare in negroes
G. Very rarely metastatises
H. Undue exposure to the sun in the major cause
I. Clay – pipe smoking can prove it
J. Poor dentition and syphilis can be important predisposing causes
2

396-In chronic renal failure by renal dialysis What is the commonest complication?
F. Gynecomastia
G. Calciphylaxis
H. Sensorimotor neuropathy
I. Perforating skin disorders
J. Cutaneous necrosis
2

397-Dendritic epidermal cells express which of the followings markers:


F. CD 5
G. CD 4
H. CD 45
I. CD 8
J. MHC class II molecules
5

398-One of the following is not correct about NK cells:


G. The major task of NK cells is to eliminate infected or malignant cells
H. They expresse Fc receptors that bind IgM
I. They recognized involves killer- activating and killer-inhibitory receptors
J. NK cells Carry receptors that recognize MHC class 1 molecules
K. Tumor cells and viruses often down regulate MHC class 1 molecules
2

399-Toll-like receptor (TLR) 6 is:


F. Diacylated lipoprotein
G. Flagellin.
H. Lipopolysaccharide.
I. Ds RNA.
J. Triacylated lipoprotein.
1

400-One of the following is NOT correct about melanocytes:


F. In premature there is high number of cells.
G. In newborn similar number of cells to young adults.
H. In adult their number decrease with age.
I. In premature high number of mature malenosomes
J. In adult, melanin production dependent on skintype and body area.
4

401Basic organogenesis is complete by the end of:


F. 12 weeks EGA.
G. 8 weeks EGA
H. 6 weeks EGA.
I. 14 weeks EGA.
J. 16 weeks EGA.
2

402-Which of the following has been shown to be the most efficacious for Lichen spinolosus ?
K. Topical corticosteroid.
L. Topical retinoid
M. Calcipotriene
N. 12% lactic acid
O. Bactroban ointment
4

403-One of the following is not correct about Th- 17 cells:


K. They are rapidly induced in response to infectious agents.
L. Transcription factor involved in Th- 17 cells development have hyper IgE syndrome
M. There is evidence that Th- 17 cells are involved in reheumatoid arthritis and psoriasis
N. IL 23 inhibits IL-17 production
O. Th- 17 cells are Produced by CD4 + T cells
4

404-The cellular source of antibacterial peptide Dermcidin is:


K. Keratinocytes .
L. Airway epithelia
M. Granulocytes
N. Intestinal tract
O. Sweat glands
5

405-Early innate immune reponses are not dominated by one of the following cytokines:
K. IL-1
L. IL-6
M. IL-8
N. TNF -BETA
O. INF- alpha
4

406-Early cornification can be observed within the hair canal at approximately:


K. 12 weeks estimated gestational age
L. 18 weeks estimated gestational age
M. 15 weeks estimated gestational age
N. 22 weeks estimated gestational age
O. 8 weeks estimated gestational age
2

407-Regarding cryotherapy which of the following statement is NOT TRUE:


F. The freezing time is shorter for benign lesions than malignant.
G. 10 seconds with open spray technique will cure 80% of actinic keratosis.
H. The freeze time is the same for both cryoprob or spray technique
I. 2-3 nm lateral spread is sufficient for successful treatment of warts.
J. Keloids can be treated with open spray technique alone or in combination with intralesional
steroids.
3

408-Which of the following LASER media is NOT a gas:


F. Argon.
G. Krypton.
H. Xenon chloride.
I. Holmium – doped
J. Copper vapor.
4

409-High-frequency electrosurgery method does not include:


A. Electrolysis
B. Electrodesiccation.
C. Electrofulguration.
D. Electrosection.
E. Electrocoagulation.
1

410-Eccrine and apocrine epithelium are stained by One of the following immunohistochemical
stains:
K. CEA
L. Bcl-2
M. Epithelial membrane antigen EMA
N. P53
O. P63
1

411-One of the following is not correct about special stains in dermatology:


J. Crystal violet stains amyloid metachromatically purple with blue background
K. Fite- faraco stains mycobacterium leprae blue
L. Orcien stains muscle and nerves yellow
M. Sudan black stains lipids black
N. Colloidal iron stains acid mucopolysaccharides blue
2 red

412-One of the following plants does not cause Contact Dermatitis:


F. Garlic
G. Peruvian lily
H. Poison ivy
I. Poison oak
J. Celery
1
- [ ] ‫اذا ذكر اليرجك نختار گارلك الن يسوي اررتنت كونتاكت د‬
413-In electron microscopy, the cell that demonstrates cytoplasmic projections and secretory
granules is:
F. Langerhans cell.
G. Keratinocyte.
H. Mast cell
I. Melanocyte.
J. Macrophage.
3

414-Keratohyalin granules:
F. Stain intensely with eosin.
G. Contain glycosphingolipids, phospholipids and cermaides that are released into the
extracellular spaces between the stratum granulosum and stratum corneum.
H. Mark the last living layer of the epidermis
I. Contain unmodified profilaggrin that is easily visible in the stratum corneum.
J. Have s100 as a surface marker.
3

415-Which of the following is a vasoconstrictor in the absence of epinephrine?


F. Lidocaine.
G. Procaine.
H. Cocaine
I. Bupivacaine.
J. Prilocaine.
3

416-One of the followings is not a biologic effect of retinoids :


F. Inhibition of apoptosis
G. Inhibition of keratinization
H. Inhibition of tumor promotion and malignant cell growth
I. Differentiation and maintenance of epithelial tissue
J. Regular expression and activation of TLRs
1

417-The major difference between darker skin and lighter skin is:??
F. The relative concentration of melanocytes per unit area of skin surface.
G. The relative activity of cutaneousmelanocytes in creation of melanosomes
H. The relative action of cutaneousmelanocytes in transferring melanosomes to keratinocytes
I. The fate of the melanosomes once they are within the keratinocyte.
J. Number of melanosomes is higher in lighter skin.
2

418-One of the following occupations may not be infected by anthrax:


F. Farmer
G. Wool sorter
H. Veterinarian
I. Dock worker
J. Butcher
3
419-Which of the following is not the most frequently subjective side effects of methotrexate ?
F. Nausea
G. Abdominal pain
H. Headache
I. Loss of libido
J. Fatigue
4

420-Which of the following is not atypical form of Pityriasis Rose:


F. Psoriatic like
G. Urticarial
H. Vesicular
I. Pustular
J. Purpuric
1

421-The mediator that has no activity on pain is:


A. Histamine
B. Prostaglandin E
C. Substance P
D. Tryptase
E. Opiod peptide
5

422-The type of laser which has semiconductor solid media is:


K. Argon
L. Diode
M. Co2
N. Copper vaour
O. Krypton
2

423-The molecules involved in the tight junction are:


K. Desmogleins
L. Desocollins
M. Claudins and daudins
N. Plakins
O. Actins and catanins
3

424-Anchoring fibrils:
K. Are noncollagenous in nature
L. Connect the basal cells to the lamina lucida.
M. Insert in special plaque in upper dermis
N. Fail to insert in lamina densa
O. Are comparable in size to Anchoring filaments
3
425-One of the following light sources has monochromatic spectral output:
K. Arc lamp
L. Fluorescent lamp
M. Light emitting diodes ( LED )
N. Lasers
O. IPL
4

426-One of the following is not an indication for electrofulguration/electrodessication (superficial


skin ablation ):???
K. Actinic keratosis
L. Epidermal nevus
M. Angiofibroma
N. Lentigo
O. Seborrheic keratosis
3

427-Concerning mycological examination, which of the following is false:

A. For black piedra the hair is simply cut above the skin level
B. For dermatophytes the hair should be plucked out with intact root
C. In tinea incognito, villus hairs examination may be the easiest method
D. The hyphae at the distal end of the nail are more likely to be viable
E. Stripping can be used in taking scales in partially treated pityriasis versicolor.

.428-The smallest genetic unit of genetic material which produces a phenotypic effect upon
mutation is:
• Allele
• Nucleic acid
• Recon
• Gene
• Muton
5

429-. One of the followings is notcommon systemic manifestations of sweet’s syndrome:


A. Arthralgias
B. Arthritis
C. Myalgias
D. Fever
E. Abdominal pain
5

430-One of the following type of HPV does not associate with common palmar , planter
myrmecial and mosaic warts:
L. HPV 1
M. HPV 2
N. HPV 3
O. HPV 27
P. HPV 57
3

431-Which of the following does not cause nodules on hair shafts:


L. Black piedra
M. Nit
N. Trichomycosis
O. Trichophyton mentagrophytes
P. Hair casts
4

432-The main target organism for valganciclovir is:


K. HSV 1
L. HSV 2
M. VZV
N. Resistant HSV
O. CMV
5

433-One of the following antimicrobial drugs is bacteriostatic:


F. Tetracyclines
G. Penicillins
H. Aminoglycosides
I. Quinolones
J. Vancomycin
1

434-Which of the following is not clinical manifestation of disseminated gonococcal infection:


L. Arthritis
M. Tenosynovitis
N. Scalp abscesses
O. Proctitis
P. Meningitis
4

435-A biopsy from a lesion on the face of an adult patient shows dermal palissades of cells and
stroma. Which one is most likely to be present in the lesion ?
a. Positive diascopy
b. Ichtyosiform scale
c. Foamy histocytes
d. Positive congo red stains
e. Telangiectasia
1

436-Silver preferentially deposits in :


• Eccrine glands and apocrine glands
• Fat
• Hair follicules
• Apocrine glands
• Eccrine glands
1

437-Which one of the following is 3d line treatment of physical urticaria:


L. Chlorpheneramine
M. Desloratidine
N. IV. IG. infusion
O. Prednisone
P. Thyroxine
3

438-The organism which cause cellulitis in fisherman is :

L. Staphylococcus aureus
M. Group A streptococci
N. Bacillus anthracis
O. parapoxvirus
P. vibro vulnificus
5

439-Which one of the following is not immunologic etiology of urticarial lesions :


L. Angiotensin- converting enzyme inhibitors
M. Autoimmune
N. IgE - dependant
O. Vasculitis
P. C1 esterase inhibitor deficiency
1

440-One of the following adhesion proteins is not observed in lamina densa:


K. Perlecan
L. Lamin 311
M. Lamin 332
N. Elastin
O. Type IV collagen
4

441-Keratin 2 is gene defect in the following type of ichtyosis:


K. Ichtyosis vulgaris
L. X- linked recessive ichtyosis
M. Lamellar ichtyosis.
N. Ichtyosis Bullosa of Siemens
O. Epidermolytic ichtyosis
4

442-Which of the following is not a feature of anxeity neurosis:


A. Confusion.
B. Initial insomnia.
C. Panic attacks.
D. Tremor.
E. Frequency of micturation.
3

443-In chronic renal failure by renal dialysis What is the commonest complication?

A. Gynecomastia
B. Calciphylaxis
C. Sensorimotor neuropathy
D. Perforating skin disorders
E. Cutaneous necrosis
3

444-One of the following statement is not true about stratum corneum:

A. Basket weave orthokeratosis is the normal pattern


B. Basket weave pattern is not seen in frozen section
C. Caused by increased lipid concentration in keratinocyte cytoplasm
D. Nuclei are normally extruded before keratinocytes reach stratum granulosum
E. Nuclei are retained in conditions of dysmaturation
4

445-In an infant who has atopic dermatitis, Which of the following is incorrect:???

K. Skin barrier defects causes higher transepidermal water loss


L. Infant with severe, refractory disease may have an overlying food allergy
M. Acute and chronic phase are Th2 predominant
N. Nearly 80% of patients with infantile AD develop allergic rhinitis or asthma later in life
O. None of the above. All are true statements
3

446-Movement of basal cells in basal layer is facilitated by :


A. Crowding of cells resulting from mitosis
B. Made possible by degradation of Anchoring filaments
C. Associated with downgrading of integrin alpha 6 beta 4
D. Mediated through cytokinens arriving from basement membrane
E. Controlled by growth factors

447-Cell cycle of basal cells:


K. G1 phase has the greatest variability in duration
L. Duration of G1 phase is not governed by the physiological state
M. S phase lasts from 2 – 4 hours in most human cells
N. G2 phase lasts from 6 – 16 hours
O. G0 phase is a must for all cells before going into G1
1

448-Neonatal desquamation normally begins how long following birth:


K. Immediately after birth
L. 0 – 12 hours
M. 12 - 24 hours
N. 24 - 36 hours
O. 3 weeks after birth
4

449-What process takes place in the granular layer when it passe into the corneal:
A. Apoptosis
B. Lysis of all organelles.
C. Lysis of organelles except keratin
D. Lysis of all intracellular structure except keratin and fillagrin
E. Deposition of new keratin in the cytoplasm

450-Which one is a well recognized Th 1- promoting factor :


a. IL-17
b. IL-23
c. IL-2
d. IL-12
e. IL-4
D

451-One of the following is not a constituent of eccrine sweat:


K. Lactate
L. Phosphate
M. Urea
N. Sodium
O. Potassium
2

452-An aging – related nail change in elderly persons is:


K. Asymmetric lunula
L. Macrolunula
M. Microlunula
N. Triangular lunula
O. Variably shaped lunula

453- Antibody to C1q is associated with:


K. Hereditary angioedema
L. Hypocomplementemic urticarial vasculitis syndrome
M. Melkersson- Rosenthal synd
N. Acquired CI inhibitor deficiency
O. Drug – induced angioedema
2

454-Glucocorticoids preferentially induce fracture in:


L. Cortical bone
M. Trabecular bone
N. Long bone
O. Irregular bone
P. Marrow rich bone
2
455-The antibody which represent a marker for epidermal proliferation is:
K. Ki -67
L. K 10
M. K 13
N. K 16
O. P 53
1

456-Which of the following antimicrobial peptides is not expressed in sebaceous gland:


A. Antileukoprotase
B. Cathelicidin
C. Psoriasis
D. β- defensing 1
E. β- defensing 2
A

457-Human dendritic cells are generated from:


A. Spleen
B. Bone marrow
C. Peripherial blood
D. Regional lymph nodes
E. Thymus
C

458-An elderly patient presents with recurrent attack of painful unilateral papulovesicular lesions
on the trunk with a dermatomal distribution. What is the next appropriate step?
A. Swabs for bacterial culture.
B. Neurological clinical evaluation.
C. Topical acyclovir and rest at home.
D. Investigation for underlying malignancy.
E. Punch biopsy.

459-Elastic connective tissue:


A. Extends from papillary dermis to reticular
B. Is absent in hair follicules sheathes
C. mature Elastic fibers contain 75% elastin
D. oxytalan fibers form microfibrils containing soluable Elastin
E. mature Elastic fibers are most abundant in middermis
A

460-Neonatal desquamation normally begins how long following birth:

A. Immediately after birth


B. 0 – 12 hours
C. 12 - 24 hours
D. 24 - 36 hours
E. 3 weeks after birth
461-Higher risk for toxicity from glucocorticoid therapy does not occur in Which of the
following ?
A. Female patients
B. Patients with rheumatoid arthritis
C. Patients with dermatomysitis
D. Patients with hyperalbuminemia
E. Patients who smoke

462-Hair follicle development in the human embryo begins during:


1. 1st trimester
2. 2nd trimester
3. 3rd trimester
4. At the blastocoele stage
5. Within 2 weeks of fertiization
1

463-Retinoid topical preparation pass into te keratinocytes through the process of :


A. Attachment to special cell wall receptors
B. Phagocytosis
C. Liquefaction of the cell membrane
D. Osmosis
E. Diffusion gradient
E

464-Neutrophil polymorphs:

A. Have multilobed nuclei


B. Are the predominant cell type in chronic inflammation
C. May fuse to form multinucleate giant cells
D. Have phagocytic abilities
E. Have numerous eosinophilic granules in their cytoplasm
A

465-One of the following histopathologic findings is characteristic for dg of BCCarcinoma:


A. Solid lobules of basaloid cells with jigsaw-puzzle appearance and thickened basement
membrane
B. Masses of basaloid cells with shadow keratinization and foreign body reaction
C. Solid lobules of basaloid cells composed of individual nodules of 2 cell type.
D. Basaloid lesion with mature and immature hair follicle formation.
E. Solid lobules of basaloid cells with sebaceous differentiation
3

466-One of the following is not correct about eccrine apparatus anatomy:


A. They are absent on nail beds
B. Secretary layer has large vesicular cells and small columnar cells
C. Myoepithelial layer is absent in eccrine ducts
D. They empty directly into epidermis
E. Dermal duct is situated within reticular dermis
B
467%Side effect of Bleomycin includes:
A. Raynaud’s phenomenon.
B. Sedation.
C. Discoloration.
D. Xerosis.
E. Ocular irritation.
A

468-Which of the following is not true about systemic lupus erythromatosus:


A. It is more common in women.
B. Remissions is commonly occur in pregnancy.
C. Anti-DNA antibodies are absent in drug induced lupus.
D. Antimalarial can reduce frequency of exacerbations.
E. Renal involvement carries the worest prognosis.
B

469-Which of the following is not a component of the epidermal differentiation complex:


A. Loricrin
B. Involucrin
C. S – 100 calcium-binding proteins
D. Small proline- rich peptides
E. Transglutaminase
5

470-Tinea manuum :
A. Dermatophyte infections on the dorsal aspect of the hand
B. Dermatophyte infection of the palm and interdigital spaces have a clinical presentation similar
to Tinea corporis
C. The reason for the two different clinical pictures is thought to be related to the lack of
sebaceous glands on the palms
D. Interdigital type Tinea pedis is often present in patients with Tinea manuum
E. Tinea manuum is usually non-inflammatory and often bilateral
4

471-Which of the following is not true about newborn skin:


A. Stratum corneum thickness is 9-10 µm.
B. Spinous cell glycogen content is abundant.
C. Similar number of melanocytes to adult.
D. Elastic fibers are small and immature.
E. The epidermal surface has vernix.
2

472-Melanocytes first appear in:


A. Upper extremities.
B. Trunk.
C. Head.
D. Genitalia.
E. Axilla.
3

473-Cyanosis:
A. Is yellowish discoloration of the sclera.
B. Is produced by high pCO2.
C. Can easily occur in Anemia.
D. Occurs more with polycythemia.
F. Is due to high oxygenated.
C

474Antigen presenting cells (APC) activation is not associated with induction of the following
cytokine:
A. IL-1&.
B. IL-6.
C. IL-12.
D. Chemokine’s.
E. MHC class I&II.

475A 42 y old housekeeper has a chronic hand dermatitis and is complaining of severe pruritus.
One of the following is not class I corticosteroid?
A. Halobetasol o.o5% cream
B. Fluocinonide o.1% cream
C. Clobetasol o.o5% lotion
D. Diflorasone diacetate o.o5% oint
E. Mometasone o.1% cream
C

476Which of the following facts about histamine is true:

A. Histamine is a potent activator of mast cells.


B. The major source of histamine in the skin are keratinocytes
C. There are currently 6 known histamine receptors
D. The H2 receptors is the receptor mainly responsible for pruritus
E. In addition to H1, H3 and H4 receptors can also modulate pruritus
5

477-Thalidomide downregulates:

A. Prostaglandin D2.
B. β2- Intregrin and β1 &α4 integrin.
C. Calcineurin.
D. CYP3A4.
E. IL-8.

The chemical mediators that plays a role in cross-linking of wound healing?


M. Kinin
N. Serotonin
O. Histamine
P. Prostaglandins
Q. Complement
2

MHC class I-restricted T-cell-mediated cytolosis appears to be involved in the followings


EXCEPT:
A. Lichen planus
B. Fixed drug eruption
C. Cutaneous GVHD.
D. Psoriasis
E. Herpes simplex virus infections.
5

Which of the following provides the principal barrier to transepidermal water loss:
A. Involucrin.
B. Profillagrin.
C. Lamellar bodies.
D. Lorocrin.
E. Fillagrin
5

The short (< 4 weeks) latency period for drug-induced psoriasis is characteristic for the following
drug:
A. Antimalarials
B. ACE inhibitors
C. Lithium
D. β – blockers
E. Terbinafine
E👍 👍

Which one is not a manifestation of Epstein – Bar virus :


a. Gianoti – Crosti syndrome
b. Genital ulcers
c. The papularpurpuric gloves and socks syndrome
d. EM
e. Oral hairy leukoplakia
2

The epithelial cells are derived from


A. Monocytes
B. Lymphocytes
C. Astrocytes
D. Eosinocytes
E. Basocytes
5

Corneal layer cells are:


A. Firmly attached to each other by desmosomes
B. Metabolically active
C. Surrounded by lamellated lipids
D. Have an outer protein envelope
E. Cytoplasm is filled with fibrillar keratin and reticulin
4

A sunscreen most reliably performs up to its sun protection factor when applied at a thickness
of:
K. 0.05 mg / cm2
L. 0.5 mg / cm2
M. 1 mg / cm2
N. 2 mg / cm2
O. 4 mg / cm2
4 bolognia p.2315

The addition of an H2 antagonist to an H1 antagonist for the management of chronic urticarial


might?
K. Help itch but not wheal numbers or severity
L. Reduce wheal numbers and severity but not itch
M. Stabilize mast cells
N. Discourage the development of tolerance to the antagonist
O. Lead to better absorption of the H1 antagonist
2

The protein which is up-regulated in the absence of plakoglobin is:


L. Beta – catenin
M. Desmocollin – 1
N. Envoplakin
O. Loricrin
P. Plakophilin
1

One of the following is not a physiologic role of IL - 10:


K. Limiting the inflammatory response
L. Supporting humoral immunity
M. Preventing overwhelming immune response
N. Increasing macrophage Ag presenting
O. Upregulating Th2 immune pathway
4

C-T CC-TT DNA mutations are characteristic for:


F. Oxidative damage
G. UVA absorption
H. P 53
I. XP
J. UV photodamage
5
Which of the following mycobacteria is rapid grower in culture?
A. Mycobacterium leprae
B. Mycobacterium kansasi
C. Mycobacterium smegmatis
D. Mycobacterium scofulaceum
E. Mycobacterium tuberculosis
C

One of the following has an inhibitory effect on hair growth:


A. IL- 1
B. INSULIN- like growth factor
C. Hepatocyte growth factor
D. Keratinocyte growth factor
E. Vascular endothelial growth factor
1

Regarding breast cancer in males, mark the WRONG statement:


A. The incidence approaches that in women in cases of klinefelter’s syndrome.
B. It is usually related to wearing braces (suspenders)
C. It can be associated with previous exposure to radiation.
D. It is likely to produce nipple inversion.
E. As in women the essential features is a breast mass
2

Each of the following is true of the occurrence of malignancy in patients with Pyoderma
Gangrenosum except:
A. Occurs in 7% of patient
B. Leukemia is the most common reported malignancy
C. The prognosis of Pyoderma Gangrenosum is good
D. IgA myeloma has been associated with Pyoderma Gangrenosum
E. Bullous Pyoderma Gangrenosum is the most commonly associated
C

Which of the following is true about gap junctions:


L. They are composed of 6 connexins
M. They mediate tight adhesion between cells
N. They mediate calcium-induced calcium release
O. They require several heterotypic connexin subunits for proper function
P. All of the above
1

The induction of an immune response directed against one or more specific tumor antigens in
immunotherapy is known as:
K. Passive
L. Active
M. Passive non specific
N. Passive specific
O. Active specific
5
The most important inflammatory cells to granuloma formation in the skin are:
F. Platelets
G. Neutrophils
H. B-cell lymphocytes
I. T – cell lymphocytes
J. Eosinophils
4

Poxviruses replicate in the:


K. Host cell cytoplasm
L. Host cell nucleus
M. Host cell mitochondria
N. Outside the Host cell
O. Inside and outside the Host cell
1

Touch, pain, temperature and itch are mediated by???


K. Meissner’s corpuscles
L. Vater - pacini corpuscles
M. Penicillate fibers
N. Kraus end bulbs
O. Papillary nerve endings
3
The drug which has the best coverage for impetigo in children is:
K. Penicillin
L. Erythromycin
M. Azithromycin
N. Clarithromycin
O. dicloxacillin
5

One of the following is not correct about eccrine apparatus anatomy:


L. They are absent on nail beds
M. Secretary layer has large vesicular cells and small columnar cells
N. Myoepithelial layer is absent in eccrine ducts
O. They empty directly into epidermis
P. Dermal duct is situated within reticular dermis

Desmocollin 1 antigen is observed in the following type of pemphigus

A. Pemphigus vulgaris
B. Pemphigus folliaceous
C. Paraneoplastic pemphigus
D. Drug-induced pemphigus
E. IgA pemphigus
2
The element necessary for function of matrix metalloproteinase is?
K. Zinc
L. Ca
M. Mg
N. Cu
O. Sulfur

The most accurate serologic method to distinguish HSV-1 from HSV- 2 is :


K. Fluorescent antibody testing
L. Enzyme - linked immunosorbent assay
M. Measurement of neutralizing antibodies
N. Western blot
O. Hemagglutination inhibtion
4

Which of the following is a substrate for matrix metalloproteinases?


K. Corneosome.
L. Keratin.
M. Collagen
N. Cornifin.
O. Filaggrin.
3

One of the following is INCORRECT about Odland bodies:


K. They are discharged from the spinous cells into the intercellular space.
L. They help establish a barrier to water loss.
M. They mediate stratum corneum adhesion.
N. They measure 300 to 500 nm in diameter
O. Both A and B.
1

Trichohyaline granules:
K. Stain basophilic.
L. Could be stained with von kossa stain.
M. Represent a hard keratin.
N. Serve the same function of keratohyaline granules of the epidermis
O. Present in the hair cortex.
4

Mothers with Herpes gestationis (gestational pemphigoid or pemphigoid gestationis) are most at
risk for?
L. Hypertension.
M. Diabetes.
N. Thyroid disease
O. Stroke.
P. Myocardial infarction
3

The proliferation of T cells after activation is primarily controlled by:


A. IL-2 receptor
B. IL-12
C. IL-4
D. IL-4 receptor
E. INF gama
2

Which of the following is the predominant component of the stratum corneum intercellular
space?
K. Histidine.
L. Squalene.
M. Sphungolipid.
N. Phospholipid
O. Glucuronic acid.
3

- What process takes place in the granular layer when it passe into the corneal:
A. Apoptosis
B. Lysis of all organelles.
C. Lysis of organelles except keratin
D. Lysis of all intracellular structure except keratin and fillagrin
E. Deposition of new keratin in the cytoplasm
3

One of the following is not correct about eccrine apparatus anatomy:


L. They are absent on nail beds
M. Secretary layer has large vesicular cells and small columnar cells
N. Myoepithelial layer is absent in eccrine ducts
O. They empty directly into epidermis
P. Dermal duct is situated within reticular dermis
2

The most accurate serologic method to distinguish HSV-1 from HSV- 2 is :


K. Fluorescent antibody testing
L. Enzyme - linked immunosorbent assay
M. Measurement of neutralizing antibodies
N. Western blot
O. Hemagglutination inhibtion
4

The element necessary for function of matrix metalloproteinase is?


K. Zinc
L. Ca
M. Mg
N. Cu
O. Sulfur
1

Touch, pain, temperature and itch are mediated by???


K. Meissner’s corpuscles
L. Vater - pacini corpuscles
M. Penicillate fibers
N. Kraus end bulbs
O. Papillary nerve endings
3

Human sebum is distinguished from lipids of internal organs by the presence of:
1. Cholestrol
2. Cholestrol esters
3. Squalene
4. Wax esters
5. Glycerides
4

Which of the following medications is concentrated in the eccrine glands?


1. Cyclophosphamide
2. Cytarabine
3. Ciprofloxacin
4. Cephalexin
5. All of the answers are correct
5

- The proliferation of T cells after activation is primarily controlled by:

A. IL-2 receptor
B. IL-12
C. IL-4
D. IL-4 receptor
E. INF gama
2

Which of the following is not a component of the epidermal differentiation complex:


A. Loricrin
B. Involucrin
C. S – 100 calcium-binding proteins
D. Small proline- rich peptides
E. Transglutaminase
5

DIF is most likely to show antibodies to One of the followings:


A. BPAg 1
B. BPAg 2
C. Desmoglein 1
D. Desmoglein 3
E. Collagen VII
2

Which of the following statements about the direct immunofluorescence pattern in lichen planus
is correct?
1. The DIF is negative in the vast majority of cases
2. Deposition of IgG is within cytoid bodies in the superficial dermis
3. The DEJ deposition is granular
4. Deposition of fibrinogen is within cytoid bodies in the deep dermis
5. There is prominent deposition of IgM within the spinous layer of the epidermis.

The short (< 4 weeks) latency period for drug-induced psoriasis is characteristic for the following
drug:
A. Antimalarials
B. ACE inhibitors
C. Lithium
D. β – blockers
E. Terbinafine
1

Complement component C3 is cleaved by:


A. C3b
B. C3bBb
C. Factor B
D. Factor D
E. Factor H

Antibody to C1q is associated with:


K. Hereditary angioedema
L. Hypocomplementemic urticarial vasculitis syndrome
M. Melkersson- Rosenthal synd
N. Acquired CI inhibitor deficiency
O. Drug – induced angioedema
2

One of the following is LEAST likely to cause contact sensitization ?


K. Neomycin
L. Silver sulphadiazine
M. Diphenhydramine
N. DNCB
O. Diphencyprone
3

One of the following is a broad spectrum blocker of UVB, UVA and visible light ?
K. Salicylate
L. Cinnamate
M. Ferrous oxide
N. Anthranilates
O. PABA
3

Which of the following drugs causes gingival hypertrophy?


L. Ciclosporin
M. Tetracycline.
N. Hydroyxurea.
O. Methotrexate.
P. Cholropheniramine.
1

Which of the following mycobacteria is a rapid grower in culture?


L. M. lepra.
M. M. fortuitum
N. M. kanasaii.
O. M. ulcerans.
P. M. tubruculosis.
2

Where androgen receptors are predominantly expressed?


K. Isthmus.
L. Dermal papilla
M. Stratum basale.
N. Papillary dermis.
O. Periglandulare dermis.
2

An 18-year old male presents to the emergency room with palpable purpura on the legs, arthritis
and abdominal pain following upper respiratory tract infection. Which of the following is most
likely to be found in this patient?
A. Eosinophilia.
B. Aphthous ulcer.
C. Proteinuria.
D. Tophi.
E. Alopecia totalis.
3

What cause thining of the nail plate:


L. Old age
M. Acitretin
N. Tetracycline
O. Darrier’s disease
P. Onychomatrichoma
2

What causes botryomycosis?


L. Aspergillus niger
M. Onchocerca volvulus
N. Staphylococcus aureus
O. Trombiculaa kamushi
P. Malassezia yamotoensis
3

In vagabond’s disease, There is a heavy infection with what:


L. Mite
M. Lice
N. Tick
O. Parapoxvirus
P. Trichophyton violaceum
2

Spores resulting from fragmentation of hyphae into separate cells are what:
K. Microconidia
L. Macroconidia
M. Blastospores
N. Arthroconidia
O. Sporangiospores
4 ‫من فيتزباتريك‬

What is a collodian preparation:


L. Semi-solid
M. Oil – in – water
N. Water – in – oil
O. Hydroalcoholic liquid
P. Cellulose nitrate in organic solvent
5

One of the following is not part of a pillosebaceous unit:


• Sebaceous gland
• Hair follicule
• Arrector pilli
• Apocrine sweat gland
• Eccrine sweat gland
5

One of the following is not true about PLE:


L. IT IS the least common photodermatosis
M. It occurs within hours of sun exposure
N. It is most severe in spring
O. Action spectra : UVB, UVA
P. Antimalarials can be used in treatment

What type of virus is the causative agent of rubela:


L. Poxvirus
M. Parvo virus
N. Toga virus
O. Orthomyxo virus
P. Picorna virus

A pregnant woman has varicella 2 days before delivery, concerning the effects on the neonate ,
which of the following recommendation is most appropriate ???
K. No further recommendation at this time
L. The mother should be started on IV acyclovir as soon as possible
M. The neoborn should receive varicella Immunoglobolines as soon as possible
N. The neoborn should receive the varicella vaccin as soon as possible
O. The neoborn should receive IV acyclovir as soon as possible

A 10 year old male with a history of mild to moderate eczema has numerous molluscum
contagiosum lesions, which of the followings is true:
L. This entity usually present as a large tender nodule
M. Is malignant tumor with 30 – 40 % rate of metastasis
N. The eosinophilic granules in the nucleus consist of HPV virions
O. The 1st line treatment is excision
P. May be treated by topical retinoids

One of the following laboratory abnormalities would best aid diagnosis of Parvovirus B - 19:
M. Elevated hemoglobine
N. Elevated reticulocyte count
O. Decreased Leukocyte count
P. Elevated liver transaminases
Q. Seropositive ANA
3

The pathogenesis of bullae formation in impetigo is best described as:


A. Systemic circulation of exfoliating toxins that targets Desmoglein 1
B. Local production of an exfoliating toxins that targets Desmoglein 1
C. Auto – immune attack of Desmoglein 1 by circulating antibodies
D. Auto – immune attack of Desmoglein 3 by circulating antibodies
E. Inherited mutations in intercellular adhesion proteins
B

SSSS is ASSOCIATED with :


K. Desmoglein 1
L. Desmoglein 3
M. Desmocollin 1
N. Desmocollin 3
P. None of the above
1

Identify the causative genetic mutation in EBS :


K. K5/14
L. KI/10GF
M. Collagen VII
N. Laminin V
O. None of the above
1

DIF is most likely to show antibodies to One of the followings:


K. BPAg 1
L. BPAg 2
M. Desmoglein 1
N. Desmoglein 3
O. Collagen VII
2

WHAT does the dyskerin gene encode?:


K. Protein tyrosine phosphatase
L. Tyrosine kinase receptor
M. Telomerase complex
N. DNA Helicase
O. Nuclear envelope Protein
3

What is the associated virus for the Sixth Disease, Roseola Infantum?
K. HHV-7
L. HHV-8.
M. Hep B.
N. Hep C.
O. EBV.
1

What anesthetic would you give to a patient with an amide allergy for a punch biopsy?
K. Lidocaine.
L. Benzocaine.
M. Prilocaine.
N. Procaine
O. Mepovacaine.
4

Imiquimod increases production of which cytokine?


K. IL-1
L. IL-3.
M. IL-5.
N. TGF-beta.
O. IFN-alpha
5

Which of the following toll-like receptor is involved in recognizing lipopolysaccharide?


K. TLR2.
L. TLR3.
M. TLR4
N. TLR5.
O. TLR7.
3
Which of the following lasers is in the infrared spectrum?
F. Erbium.
G. Excimer.
H. Q-switched Ruby.
I. Frequency Doubled Nd:Yag.
J. Variable Pulsed KTP.
1

Fluconazole differs from other azoles by:

A. High molecular weight.


B. Low water solubility.
C. Not affected by food or gastric acidity.
D. Poor penetration to CSF.
E. Metabolized in the liver.
3

Migration into wound by (leapfrogging) is a feature of :


A. Neutrophils
B. Macrophages
C. Fibroblasts
D. Keratinocytes
E. Myofiboblasts
D

One of the following statement is not true about stratum granulosum:


A. Histidine –rich keratohyaline granules characterizes this layer
B. Appropriate clumping of keratin is a function of keratohyaline
C. Involucrin appears as a substrate for transglutaminase in cross linking of cellular envelope
D. Absence of this layer could be a feature of diseases such as psoriasis and ichtyosis
E. Keratinocytes are anuclear
E

The average thickness of epidermis on acral sites is :


A. 1 mm
B. 2 mm
C. 3 mm
D. 4 mm
E. 5 mm
A

One of the following statement is not true about stratum corneum:


A. Basket weave orthokeratosis is the normal pattern
B. Basket weave pattern is not seen in frozen section
C. Caused by increased lipid concentration in keratinocyte cytoplasm
D. Nuclei are normally extruded before keratinocytes reach stratum granulosum
E. Nuclei are retained in conditions of dysmaturation
D

One of the following is not a feature of villous hair?


A. Non pigmented
B. Medullated
C. Absent erector pili muscle
D. Has extremely large sebaceous gland
E. Has full hair cycle
B

Which of the following is a prerequisite for diagnosis of Bronze baby syndrome?


A. Brain tumor
B. Renal disease
C. Hepatic disease
D. Atopic dermatitis
E. Congenital heart disease
C

Which of the following diseases of the neonatal and infantile period involve the scalp
predominantly?
A. Transient pustular melanosis
B. Eosinophilic pustulosis
C. Anetoderma of prematurity
D. Infantile Psoriasis
E. Toxic erythema of the newborn
B

Which one of the following is not correct about hypertrophic scar :


a. The onset is delayed
b. Usually is proceeded by trauma
c. It is confined to wound margin
d. Increased mast cell histologically
e. Contains myofibroblasts
A

One of the followings about hypertrophic scar is not correct :

A. Always preceded by injury


B. There is prominent erythema
C. It does not contain myofibroblasts
D. No spontaneous healing
E. Good response to treatment
3

What is the prognosis of erythema neonatorum?


A. Progression into ulcers
B. Persistence till infantile period
C. Progression into scally papules
D. Spontaneous disappearance
E. Postinflammatory hypopigmentation
D

Heart block occurs in neonates born to mother suffering of What ?


A. Pemphigus vegetanis
B. Pemphigoid gestationis
C. SLE
D. Subcorneal pustular drematosis
E. Polymorphic eruption of pregnancy
C

Primary cell in creating granulation tissue is:


A. Neutrophil
B. Macrophage
C. Fibroblast
D. Keratinocyte
E. Myofiboblast
C

Which of the following is an emulsifier?


A. Urea
B. Lanolin
C. Glycerin
D. Propylene glycol
E. Pyrrolidone carboxylic acid
2

Propranolol 40mg twice daily is used in the treatment of a clinical feature of Which of the
following diseases?
A. Rosacea
B. Acne fulminant
C. Gutate psoriasis
D. LP Pemphigoides
E. Pityriasis Lichenoid Chronica
1

Infection by Which of the following species responds to combination therapy of dapson,


streptomycin and cotrimaxazole?
A. Madurella mycetomatis
B. Acremonium spp
C. Fusarium spp
D. Madurella Grisea
E. Actinomadura Madurae
5

Depletion of neuropeptides of an afferent nerve terminals and binding to vanilloid receptor 1 is


the mode of action of What?
A. Menthol
B. Doxepin
C. Capsaicin
D. Retapamulin
E. Mechlorethamine
C

Which of the following infection is a bartonellosis?


A. Pitted keratolysis
B. Trichomycosis axillaris
C. Majochi granuloma
D. Cat scratch fever
E. Blistering distal dactylitis
D

Which of the following IL is an anti-infammatory?


A. 6
B. 7
C. 10
D. 12
E. 17
C

Involvement of the nasociliary branch of ophthalmic nerve by herpes zoster leads to the
development of lesions on What ?
A. Ear
B. Forehead
C. Upper lip
D. Lower lip
E. Tip of the nose
5

Which of the following is a compenent of a syndrome that causes triangular lunulae of the
thumb nails ?
A. Alopecia
B. Civatte bodies
C. Glomerulonephritis
D. Cutis verticis gyrate
E. Coronoid lamella
C

An adult patient presented with recurrent attacks of pruritic skin lesions after eating fish. The
lesions lasted for less than 24 hours. Which of the following is a histopathological or a clinical
feature of the disease he suffers from?
A. Excoriations marks
B. Lichenifications
C. Spongiosis
D. Dermal oedema
E. Hyperkeratosis
D

Which of the following cells contain Weibel-Palade inclusions?


A. Fibrocyte
B. Endothelial
C. Nail matrix
D. Apocrine sweat coil
E. Hair IRS
B

Which of the following cells produce leptin ?


A. Corneocyte
B. Lipocyte
C. Fibroblast
D. Sebocyte
E. Basal keratinocyte
B

One of the following is not correct about Dysmorphic disorder:


A. May affect as many as 10 – 40% of dermatology patients
B. Preoccupation with a non-existent or slight defect in appearance
C. Often begins in late adulthood
D. Body site of concern are the nose, mouth, hair, brest and genitalia
E. Typically associated with compulsive or ritualistic behaviors.
C

Transglutaminase enzymes are involved in the formation of Which of the following ?


A. Sebum
B. Keratin 1&10
C. Oxytalan fibers
D. Cornified cell envelop
E. Intercellular lipid in the stratum corneum
D

One of the following is not correct about lichen nitidus:


A. Characterized by numerous discrete, skin-colored, pinhead-sizsd papules
B. Oral involvement is rare
C. Globular deposits of IgG, IgA, IgM or C3 in lower levels of the epidermis
D. Pruritus sometimes
E. Wickham”s striaa absent
C

In Which of the following sites the collagen fibers are oriented vertically in the dermis ?
A. Nail bed
B. Palmar skin
C. Scrotal skin
D. Buccal mucosa
E. Dermal papilla
1

One of the following is not correct about Apocrine sweat glands:


A. They are located at eyelid margins
B. CD 15 staining of Apocrine but not eccrine sweat glands
C. Associated with terminal hair follicule
D. The secretory rate can be as less as 10 times that of the eccrine glands
E. Apocrine chromohidrosis is a reflection of the rich lipofuscin continent of Apocrine sweat.
D

One of the following infection may cause extreme prematurity:


A. Aspergillus
B. Streptococcus A
C. HSV
D. Haemophilus influenza
E. Streptococcus B
C

Café–au-lait macules only is description for the following type of NF :


A. NF 1
B. NF 2
C. NF 6
D. NF 5
E. NF 4
C

One of the following is not Hemi desmosome-anchoring filament complexes:


A. Plectin
B. BP antigen 1
C. Integrin
D. Nidogen
E. Laminin 332 (formerly lamininis )
D

One of the following is not correct about EBV :


A. Oral hairy leukoplakia is an associated mucosal condition that represents an early sign of HIV
infection
B. Lesions are usually painful
C. They appear as corrugated white plaque with hair – like projections
D. There is no association with malignant degeneration
E. Topical or antiviral agents, podophyllin resin or gentian violet can be effective
B

The dose of hydroxychloroquine which does not cause eye toxicity/kg ideal body weight/day:
A. Not exceed 7.5 mg
B. Not exceed 8.5 mg
C. Not exceed 10 mg
D. Not exceed 12 mg
E. Not exceed 6.5 mg
E

Psoriasis patients who have treated by 200 PUVA treatment, are at increased risk for the
development of: ???
A. BCC
B. Melanoma
C. Leukemia
D. T-cell lymphoma
E. SCC
5

One of the following drug is rarely associated with erythroderma:


A. Allopurinol
B. Aspirin
C. Dapsone
D. Vancomycin
E. Sulfonamides
B

The less common drug associated with erythroderma:


A. Allopurinol
B. Phenytoin
C. Sulfonamides
D. Lithium
E. Beta lactam antibiotics
D

One of the following is possible delayed type, cell mediated drug reaction:
A. Angioedema
B. Stevens – Johnson syndrome
C. Petechiae secondary to drug-induced thrombocytopenia
D. Vasculitis
E. Serum sickness
B

BCC is selective contraindication in the following biologic agent used for the treatment of
psoriasis :
A. Ustekinumab
B. Alefacept
C. Etanercept
D. Infliximab
E. Adalimumab
A

One of the following is not IL-10 familly:


A. IL-19
B. IL-20
C. IL-22
D. IL-23
E. IL-26
D

One of the following does not affect proximal matrix:


A. Beau’ lines
B. Pitting
C. Longitudinal fissuring
D. Apparent leukonychia
E. Trachonychia
D

One of the following is not components of membrane attack complex (MAC):


A. C55
B. C5a
C. C6
D. C7
E. C9

C55 ‫ خطأ‬C5B
B

Eccrine sweat glands:


K. Start in 5th month of gestation
L. The glandular part contains 2 separate layers one has clear and the other has dark cells
M. the dark cells contain water , glycogen and mucin
N. the epidermal part of the duct is the acrosyringium
O. Eccrine sweat is devoid of enzyme
4

Neural crest cells in the skin:


A. Migrate as melanocytes into the epidermis
B. Contain one type of pigment
C. Their pigment is secreted in vesicular structure
D. The shape of the melanocytes is cuboidal
E. The pigment Produced is transferred to adjacent keratinocytes directly by osmosis

Which one is a well recognized Th 1- promoting factor :


a. IL-17
b. IL-23
c. IL-2
d. IL-12
e. IL-4
D

An antileprosy drug Which also possesses an anti-inflammatory effects is:


K. Dapson
L. Clofazimine
M. Rifampicin
N. Prothionamide
O. corticosteroid
A

Regarding telogen:
L. Telogen hairs are radiosensitive.
M. Telogen duration is relatively fixed for a particular body site
N. The percentage of hairs in telogen is independent of body site
O. In general, body hair has a lower percentage of hairs in telogen than scalp hair
P. A and B
5

Which of the following is not a component of the epidermal differentiation complex:


K. Loricrin
L. Involucrin
M. S – 100 calcium-binding proteins
N. Small proline- rich peptides
O. Transglutaminase
5

Nonspecific urethritis is mostly caued by :


K. Mycoplasma species
L. Staph aureus
M. candida species
N. trichomonas
O. Chlamydia
5

Griseofulvin reaches te outermost layer of the skin by:


L. Eccrine sweat
M. Sebum
N. Lymphatic system
O. circulatory system
P. diffusion
A

Which of the following topical therapy may be effective for keratosis lichenoid chronica?
F. Calcipotriol
G. Triamcinolone
H. Tretinoin
I. Permethrin
J. Metronidazole

One of the following phenotypic markers is not observed in migrating Langerhans cells:
A. Birbeck granules
B. Langerin
C. MHC class II
D. CD1a
E. CCR6
5

One of the following is cyclooxygenase inhibitor:


A. Cyclosporine
B. Aspirin
C. NASDs
D. Methotrexate
E. Sulfsilamides
C

The most helpful investigative tool in the diagnosis of lymphatic malformations is


K. Ultrasonography
L. Biopsy
M. CT scan
N. Lymphography
O. Arteriography
1

The location of epidermal stem cells in human palmoplanter skin are found at:
A. The base of the rete ridges
B. The basale cell layer
C. The granular cell layer
D. Stratum lucideum
E. Stratum corneum
A

The density of epidermal melanocytes at 80 – 90 days EGA is:


A. 1000 cells / mm2
B. 800 - 1500 cells / mm2
C. -3000 cells / mm2
D. -4000 cells / mm2
E. -5000 cells / mm2
C

About atopic dermatitis, which of the following is NOT TRUE?


F. In 90% of patients, the onset is before 5 years.
G. Parental history if atopy considered to the strongest risk factor.
H. Staphylococcus aureus colonize ove 90% of AD skin lesions.
I. Epidermal hyperplasia is the predominant finding of chronically lichenified plaque.
J. The barrier function of the stratum corneum is usually intact
5

Which of the following is not a feature of major depression?


F. Anhedonia.
G. Irritability.
H. Insomnia or hypersomnia.
I. Psychomotor agitation or retardation
J. Suicidal ideation.
4

UVB light is most effective in the treatment of pruritus associated with:


F. Hypothyroidism.
G. Pharmacologic pruritus.
H. Chronic renal failure
I. Diabetes mellitus.
J. Lymphoma.
3

Which of the following antifyngal is FDA category D in pregnancy:


G. Terbinafine.
H. Variconazole.
I. Amphotricin B.
J. Flucytocin
K. Caspofungin acetate.
2

Which of the following is NOT TRUE about pruritus ani?


F. Underlying dermatosis constitute up to 80% of its etiology
G. Diatary factors such as excessive coffee intake is considered.
H. The onset of the condition is insidious and chronic.
I. Men being more common than females.
J. Appropriate psychiatric screening is required.
1

Which of the following lasers, the target chromphore is vascular?


K. Argon.
L. Erbium YAG.
M. Pulsed dye
N. Alexandrite.
O. Carbon dioxide.
3

Which of the following drugs causes cholestasis?


F. Morphine.
G. Retinoids.
H. Erythromycin estolate
I. Chlorquine.
J. Isoniazide.
3

Which of the following is not side effects of systemic steroids:


K. Hypokalemic alkalosis.
L. Hypocalcemia.
M. Lymphocytosis
N. Pseudotumour cerebri.
O. Exophthalmus.
3

One of the followings about hypertrophic scar is not correct??


K. Always preceded by injury
L. There is prominent erythema
M. It does not contain myofibroblasts
N. No spontaneous healing
O. Good response to treatment
3

Which of the following is not a risk factor for BCC:


K. Cumulative sun exposure.
L. Human papilloma virus
M. Fair skin.
N. Freckling.
O. Chronic non healing wound.
5

PR – like eruption may be caused by:


L. Lithium
M. Cu
N. Gold
O. Zn
P. Iron salts
3

One of the following is true about venous leg ulcer


M. It occurs at perrure site
N. It is a punched out ulcer
O. Leg and ankle odema may be seen
P. Shiny atrophic skin
Q. Peripheral neuropathy
3

The pathogenesis of bullae formation in impetigo is best described as:


A. Systemic circulation of exfoliating toxins that targets Desmoglein 1
B. Local production of an exfoliating toxins that targets Desmoglein 1
C. Auto – immune attack of Desmoglein 1 by circulating antibodies
D. Auto – immune attack of Desmoglein 3 by circulating antibodies
E. Inherited mutations in intercellular adhesion proteins
2

One of the following is not true about PLE:


L. IT IS the least common photodermatosis
M. It occurs within hours of sun exposure
N. It is most severe in spring
O. Action spectra : UVB, UVA
P. Antimalarials can be used in treatment
1
A 10 year old male with a history of mild to moderate eczema has numerous molluscum
contagiosum lesions, which of the followings is true:
L. This entity usually present as a large tender nodule
M. Is malignant tumor with 30 – 40 % rate of metastasis
N. The eosinophilic granules in the nucleus consist of HPV virions
O. The 1st line treatment is excision
P. May be treated by topical retinoids
5

Eccrine sweat glands:


K. Start in 5th month of gestation
L. The glandular part contains 2 separate layers one has clear and the other has dark cells
M. the dark cells contain water , glycogen and mucin
N. the epidermal part of the duct is the acrosyringium
O. Eccrine sweat is devoid of enzymes
D

The sebaceous glands:


L. Are found all over the body
M. Secretion is released by decapitation of cells
N. Ducts of free sebaceous glands open in hair follicule canal
O. Sebum production falls significantly in women after 50y of age
P. Administration of estrogen reduces size and production in males but not in females
4

“Acute gonococcal urethritis is best treated with :


J. Crystalline penicillin
K. Procaine penicillin
L. Cotrimazole
M. Tetracycline
N. Ciprofloxacine
5

Key feature of delusions of parasites include Which of the following:


K. Fixed belief that one is infected with parasites.
L. Sensation of biting or crawling Sensation on the skin
M. Oftentimes presents with history of amphetamine or cocaine use
N. A&B
O. All of the above
4

Chlamydia is best cultured on:

L. McCoy cells
M. Cyclohexamide treated McCoy cells
N. Yolk sac of chicken embryo
O. Blood agar
P. Chocolate agar Micro
2

Molecules involved in desmosomal junction include:

A. Desmogleins
B. Desmocollins
C. Desmogleins and Desmocollins
D. Desmogleins, Desmocollins and plakins
E. Desmogleins, and armadillo plakins
4

Brown fat differ from white fat by being:


A. unilocular
B. metabolically less active
C. its cells contain less mitochondria
D. most prominent retroperitoneally
E. responds to oral noradrenaline
D

A child present with 2 circular lesions that show scales only with partial loss of scalp hair. What
is the most likely diagnosis?
A. Alopecia Areata
B. Tinea capitis
C. Folliculitis
D. Favus
E. Kerion
B

Antimicrobial substances on the surface of the skin include:


A. Special peptides
B. Phospholipids
C. Pyrilladone
D. Alphacarboxylic acids
E. Glycolipids
A

Which subtype of melanoma is most commonly diagnosed on facial skin of elderly (> 70 y old )
A. Nodular
B. SSM
C. Acral lentiginous
D. Lentigo maligna
E. Amelanotic
4

Aquagenic pruritus is frequently associated with a pruritic condition in many cases. Which
condition is that?
A. LSC
B. Cholestasis
C. Hodgkin”s disease
D. Polycythemia vera
E. HIV infection
D

A mutation in the gene on chromosome 17 that encodes the protein neurofibromin increases the
risk of Which malignancy ?
A. Prolactinoma
B. Mucosal neuroma
C. Acute lymphocytic leukemia
D. Juvenile myelomonocytic leukemia
E. Progressive multifocal leukoencephalopathy
D

The sensation of pruritus is transmitted to the central nervous system through which type of
nerve fibers?
A. A- alpha fibers
B. A - beta fibers
C. A - gamma fibers
D. B - fibers
E. C - fibers
E

Which is the most important mediator released by mast cells and causing pruritus in urticarial?
F. Histamine
G. Heparin
H. Tryptase
I. Prostaglandins-D2
J. Platelet-activatingv factor
1

Which histologic subtype of BCC is considered most likely to recur?


A. Pigmented
B. Superficial
C. Nodular
D. Adenoid
E. Sclerosing
5

One of the following is not sexually transmitted:


A. scabies
B. moll. contagiosum
C. condylomata accuminata
D. eczema herpeticum
E. herpes progenitalis
D

In cutaneous Mastocytosis, many of the symptoms are caused by mast cell mediators. Some of
them are preformed and others are newly formed. Out of the listed mediators below, Which
mediator is not preformed (newly formed)?
A. Heparin
B. Histamine
C. Chymase
D. Leukotriene
E. Tryptase
D

One of the following is not true about Herpes Simplex:


A. Primary lesion is vesicle
B. Neuralgia is frequent
C. Junctional mucosa is affected
D. Eczema herpeticum occurs
A. Recurrances are common
B

A 2 day old full term neonate develops blotchy erythematous macules with small central
pustules over the upper trunk and extremities. A germ stain reveals predominantly eosinophils.
What is the most likely dg?
A. Incontinentia pigmenti
B. Transient neonatal pustular melanosis
C. Urticaria pigmentosa
D. Erythema toxicum neonatorum
E. Miliaria
D

A 10 month old infant presents with diarrhea and an eczematous erutption on the hands, feet
and perianal area for 4 weeks shortly after being weaned from breastfeeding. The patient has
mild alopecia and is somewhat irritable. Which vitamin deficiency is suspected?
A. Iron
B. Zinc
C. Copper
D. Selenium
E. Vitamin A
B

Which of the following should be checked periodically in patient with Henoch-Schonlein


purpura ?
A. Chest X- ray
B. EKG
C. Urinalysis
D. LFTs
E. All of the above
C

In children, What is the most common cause of Erythema nodosum?

A. Sarcoid
B. Tuberculosis
C. Staphylococcus
D. Streptococcus
E. Hepatitis B
D

Regarding infant skin, Which of the following is not different from that of adult skin?
A. Weaker attachment between the epidermis and dermis
B. The skin is thinner
C. The body surface area-to-weight ratio of an infant is up to 5 times that of an adult
D. There is a decreased association of infection
E. There is an increased risk of toxicity to topical medications
D

Makki 15-16-17-18
Which of the following is not correct about mechanism of action of azathioprine:

A. Reduced NK cell function


B. Reduction In CD8+ T cell
C. INHIBITION of lymphocyte proliferation
D. Activation of antibody responses
E. Impaired neutrophil chemotaxis
E

One of the following is a broad spectrum blocker of UVB, UVA and visible light ?

A. Salicylate
B. Cinnamate
C. Ferrous oxide
D. Anthranilates
E. PABA

One of the following is not a marker for lymphatics:


A. VEGFR-3
B. Podoplanin
C. PAL-E
D. SLC/CCL21
E. Prox1
C 👍 table 102-1

BV: cd34,palE(pv1),VWF
Lymphatics: podoplanin,lyv,prox,fgfr3

One of the following occlusive ingredients is present in topical moisturizers:


A. Squalene
B. Ceramides
C. Cholesterol
D. Fatty acids
E. Glycerin
E

Regarding keratins distribution, K80 is usually in:


A. Sweat gland ducts
B. Tongue
C. Skin
D. Inner root sheath (hair follicles)
E. Oral mucosa
2🤷 👍

Which of the following has cytotoxic action:


A. Cyclosporine.
B. Cyclophosphamide
C. Azathioprine.
D. Mycophenolate mofetil
E. Methotrexate.
B

Psoriasis is now considered as a systemic condition. The system mostly affected together with
the skin is:
A. The articular
B. Respiratory
C. Renal
D. Gastrointestinal
E. Nervous
A

One of the following fungal infection of the skin is not due to dermatophytes:
A. FAVUS
B. Kerion
C. Piedra
D. Tinea pedis
E. Tinea corporis
C

The cellular source of antibacterial peptide Dermcidin is:


A. Keratinocytes .
B. Airway epithelia
C. Granulocytes
D. Intestinal tract
E. Sweat glands
5

The treatment of Atopic Dermatitis in children often requires the use of topical corticosteroids.
Which of the following choices represents an appropriate choice for use in the face and
intertriginous areas of a child affected by Atopic Dermatitis in order to reduce the possibility of
adverse effects?
A. Clobetasol propionate
B. Amcinonide
C. Mometasone furoate
D. Fluocinonide
E. Desonide
E
Table 125-5

Infection by Which of the following species responds to combination therapy of dapson,


streptomycin and cotimaxazole?

A. Madurella mycetomatis
B. Acremonium spp
C. Fusarium spp
D. Madurella Grisea
E. Actinomadura Madurae
E

What is the primary target of androgen action in the hair follicule?


A. Bulge area
B. Hair matrix
C. Dermal papilla
D. IRS
E. ORS
C

One of the following does not affect proximal matrix:

A. Beau’ lines
B. Pitting
C. Longitudinal fissuring
D. Apparent leukonychia
E. Trachonychia
D

A patient of herpes zoster developed granuloma annulare at the same site. What is this
phenomenon/sign?
A. Pathergy.
B. Koebner.
C. Reverse Koebner.
D. Autosensitization.
E. Isotopic.
E
Forschneimer’s sign is seen in :
A. Measles
B. Rubella
C. Scarlet fever
D. Herpangina
E. Infectious mononucleosis
B
Interferons are not FDA approved indication for:
A. Malignant Melanoma
B. Mycosis Fongoid
C. AIDS related Kaposi sarcoma
D. Condyloma accuminata
E. Hepatitis B
E

Postauricular adenopathy is commonly associated with :


a. Rubella
b. Mumps
c. Erythema infectiosum
D. Scarlet fever
E. Measles
A

A patient treated for rheumatoid arthritis develops worsening of her rheumatoid nodules, which
drug may be responsible for this:
F. Methotrexate
G. Adalimumab
H. Corticosteroids
I. Hydroxychloroquine
J. Gold
1

Which of the following listed facts is incorrect during patient education of Herpes Zoster?
A. Most people recover over several weeks.
B. Workup for immunodeficiency is unnecessary.
C. All adults age 50 and older should get a vaccine.
D. The vaccine is live attenuated virus.
E. This is not sexually transmitted.
B

A 5Y. BOY known to have sickle cell disease presented with an acue febrile rash and transient
aplastic crisis, which virus is well recognized to cause this presentation ?
a. Herpes simplex
b. Varicella- zoster
c. Parvovirus
d. Rubella
e. Rubeola
C
Which of the following is not included in the Herpes Virus group :
A. Epstein Bar virus
B. Cytomegalo virus
C. Varicella zoster virus
D. coxackie virus
E. HHV 8
D

An elderly patient presents with recurrent attack of painful unilateral papulovesicular lesions on
the trunk with a dermatomal distribution. What is the next appropriate step?
A. Swabs for bacterial culture.
B. Neurological clinical evaluation.
C. Topical acyclovir and rest at home.
D. Investigation for underlying malignancy.
E. Punch biopsy.
D

One of the following is not correct about EBV :

A. Oral hairy leukoplakia is an associated mucosal condition that represents an early sign of HIV
infection
B. Lesions are usually painful
C. They appear as corrugated white plaque with hair – like projections
D. There is no association with malignant degeneration
E. Topical or antiviral agents, podophyllin resin or gentian violet can be effective
B

EBV :
A. Approximately 75% of young adults with primary infection develop infectious mononucleousis
B. EBV is commonly associated with B-cell lymphoma
C. The virus enters cell by specifically binding to cell surface complement CR4
D. The humoral immune response, although it generates Ab to a variety of EBV Ag , appear to
be protective
E. Splenomegaly is found in >50% of patients and can be accompanied by hepatomegaly.
E

In infectious mononucleosis, One of the following is not correct:

A. It is caused by Epstein Barr virus


B. Nasal secretion is the main source of infection
C. It is not highly contagious
D. Heterophil antibody is detected by monospot test
E. Amoxicillin should be avoided
B

Which one is not a manifestation of Epstein – Bar virus :


a. Gianoti – Crosti syndrome
b. Genital ulcers
c. The papularpurpuric gloves and socks syndrome
d. EM
e. Oral hairy leukoplakia
C
Which of the following is true regarding oral contraceptives in acne patients:
F. Check free testosterone levels prior to initiation
G. Some oral contraceptives do not carry an increased risk of venous thrombosis
H. oral contraceptives decrease free testosterone levels by competitively binding to testosterone
receptors
I. oral contraceptives increase hormone binding globulins in serum
J. oral contraceptives directly inhibit sebaceous gland activity
4

Which of the following is not a cutaneous manifestations of Epstein-Barr virus infection:


A. Oral hairy leukoplakia.
B. Gianotti-crosti syndrome.
C. Genital ulcers.
D. The papular purpuric gloves and socks syndrome
E. Erythema multiforme
D

Heck’s disease is caused by:


A. HPV 3.
B. HPV 13.
C. HPV 10.
D. HPV 16.
E. HPV 11.
2

Oral papillomas associated with which type of HPV?


A. HPV 28.
B. HPV 32
C. HPV 39.
D. HPV 52, 53.
E. HPV 42, 43.
2

Which of the HPV human papilloma viruses proteins is involved in carcinogenesis by degrading
p53?

A. E2
B. E4
C. E5
D. E6
E. E7
4

Which of the following is not true regarding human papilloma virus:


A. It is double standed DNA virus.
B. It is approximatelly 55nm in diameter.
C. It is enveloped virus
D. L1 represent 95% of virion protein.
E. HPV 16 and 18 are ptentially oncogenic.
C

Which HPV is associated with verrucous carcinoma?


A. HPV 6
B. HPV 16
C. HPV 32.
D. HPV 1.
E. HPV 3.

Main inflammatory mediator in inflammatory phase of wound healing is:


A. IL-6.
B. IL-12.
C. TGF-alpha.
D. PDGF.
E. IL-4.
A

One of the following cytokines has beneficial effect on wound healing:


F. IL-1.
G. TGF-B.
H. IL-8.
I. IL-6.
J. TNF.
2

Which of the following cells are required for wound healing?


1. Neutrophil
2. Macrophage
3. Eosinophil
4. Langerhans cell
5. Lymphocyte
2

Which Interleukin is responsible for neutrophil chemotaxis:


F. IL- 1
G. IL- 5
H. IL- 8
I. IFN- alpha
J. TGF-beta
3

One of the following chemical mediators play a role in stimulation of fibroblast proliferation?
A. Kinins
B. Serotonin
C. Histamine
D. Prostaglandins
E. Complement
B
The chemical mediators that plays a role in cross-linking of wound healing?
M. Kinin
N. Serotonin
O. Histamine
P. Prostaglandins
Q. Complement
2

Regarding wound healing, which of the following is not true:


A. Proliferation phase is the first phase.
B. Platelets, the first cells appear in healing process
C. Depth of the wound determines the degree of contraction.
D. Macrophages are important cells in healing process.
E. The dermal matrix is produced by fibroblast.
A

Natural killer cells usually does not exihibit One of the following:
A. CD+ 16
B. CD+ 56
C. CD+ 86
D. CD+ 94
E. CD+ 161
C
Natural killer cells can adhere and kill target cells that are coated by ?

A. IgE
B. IgM
C. IgA
D. C5
E. IgG
E

Corneal layer cells are:


A. Firmly attached to each other by desmosomes
B. Metabolically active
C. Surrounded by lamellated lipids
D. Have an outer protein envelope
E. Cytoplasm is filled with fibrillar keratin and reticulin
D

The papillary dermis in contrast to the reticular dermis has:


A. Thicker collagen fibres
B. Blood vessels with 2 layers of smooth muscles
C. Tiny elastic fibres that run horizantlly
D. Eccrine sweat glands
E. Villous hair follicles
B
Regarding viral vectors for skin gene therapy, One of the following viruses has transient (no
integration) duration of expression:
A. Retrovirus
B. Poxvirus
C. Adenovirus
D. HPV
E. Parvovirus
C

One of the following is not true about lepromatous leprosy:


A. Numerous lesions are present macules, papules, nodules.
B. Asymmetrical distribution.
C. Vague border and difficult to distinguish between normal and affected skin.
D. Sensation is not affected.
E. Many bacilli could be found in skin lesion.
B
. One of the following IS NOT characterized of tuberculoid leprosy:

A. Hypohydrotic.
B. Anesthetic.
C. Atrophic.
D. Asymmetrical.
E. < 5 lesions.
C

Which of the following specimens is not used for Mycobacterium tuberculosis staining:
A. CSF.
B. Pleural fluid.
C. Sputum.
D. Biopsy material.
E. Urine
E
Which one of the following mycobacteria is scotochromogen:
a. M. Kansasii
b. M. intercellulare
c. M. tuberculosis
d. M. marinum
e. M. scrofulaceum
E

Which of the following is rapidly growing(3-5 days) mycobacterium:


A. Mycobacterium leprae.
B. Mycobacterium tuberculosis.
C. Mycobacterium kanasasi.
D. Mycobacterium smegmatis
E. Mycobacterial intercellulare.
D

Which of the following mycobacteria is a rapid grower in culture?


A. M. lepra.
B. M. fortuitum
C. M. kanasaii.
D. M. ulcerans.
E. M. tubruculosis.
B

Phenolic glycolipid-1 is important in identification of:


A. Mycobacterium Kanasasi.
B. Dermatophytes.
C. Moulds.
D. Mycobacterium Leprae
E. Box virus.
D

- Which of the following should not to be taken after meals?

A. Rifampicin.
B. Griseofulvin.
C. Clofazimine.
D. Dapsone.
E. Itraconazole.
A

One of the following is not correct about special stains in dermatology:


A. Crystal violet stains amyloid metachromatically purple with blue background
B. Fite- faraco stains mycobacterium leprae blue
C. Orcien stains muscle and nerves yellow
D. Sudan black stains lipids black
E. Colloidal iron stains acid mucopolysaccharides blue
B bright red

Side effect of Bleomycin includes:


A. Raynaud’s phenomenon.
B. Sedation.
C. Discoloration.
D. Xerosis.
E. Ocular irritation.
A

Soft keratin is present in:


A. Cortex of the hair.
B. Cuticle of the hair.
C. Hair matrix.
D. Nail plate.
E. Medulla of the hair
E
Which of the following is not feature of niesseria gonorrhoea:
A. Is a Gram negative bacteria.
B. Grows in hemoglobin – containing medium.
C. It needs 3-5% CO2.
D. It has polysaccharide capsule
E. It canot tolerate dryness and low temperature.
D

Which of the following is not characteristic in treponema pallidum:


A. 6-20 µm in length.
B. Regular tight spirals.
C. Limited capacity for DNA repair.
D. Periplasmic flagella.
E. Ability to survive outside an animal host.
E

Which one of the following laboratory tests is not used for diagnosis of primary syphilis :
A. Dark field microscopic examination
B. VDRL
C. TPHA
D. FTA – ABS 19S-IgM
E. FTA – ABS
4

Treponema pallidum pertenue causes:


A. Syphilis.
B. Bejel.
C. Yaws.
D. Pinta.
E. Periodontal disease.
C

Treponema carateum causes :

A. Syphilis
B. Bejel
C. Yaws
D. Pinta
E. Periodontal disease
4

One of the following is not a clinical feature of secondary syphilis:


A. Piyriasis rosea-like.
B. Generalized papulosquamous.
C. Pink papules.
D. Bullous.
E. Lesions of the palms and soles can have a collarette of scale.
4
The venereal disease with shortest incubation period is:
a. Syphilis
b. Chlamydia urethritis
c. Pinta
d. Chancroid
e. Mycoplasmal infection
4

One of the followings is not correct about causes of genital ulcer diseases
A. Genital herpes, HSV2>HSV1
B. Primary syphilis, Treponema pallidum
C. Chancroid, haemophilus ducreyi
D. LGV, Chlamydia trachomatis type D-K
E. Donovanosis, calymmatobacterium granulomatis.
4

Plasma cells are not conspicuous in the infiltrate of:


A. Syringocytadenoma papilliferum
B. Early syphilis
C. Granuloma inguinale
D. Rhinoscleroma
E. Lichen planus
5

The SUBCUTIS:
A. Is absent in eyelids and female genitalia
B. The thickness of the layer is independent of race
C. Up to the 14th week is made of mesenchymal lipobalsts
D. Matures into lipolytes by the 20th week
E. Some of the primitive cells mature into macrophage
3

Vaginal discharge smear is not useful to screen?


A. Gonnococcal infection
B. Trichomoniasis
C. Candidiasis
D. Syphilis
E. Bacterial vaginitis
D

Hemophilus ducreyi is the causative organism of:


A. Granuloma inguinale.
B. Hard sore.
C. Lymphogranuloma venerium.
D. Urethritis.
E. Soft sore
E

One of the following is not complications of granuloma inguinale:


A. Permanent scarring
B. Inguinal adenitis
C. Squamous cell carcinoma
D. Depigmentation
E. Genitals destruction
B

A 9 year boy with scabies did not respond to the topical treatment. The best drug to prescribe
will be:
A. Albendazole
B. Piperazine
C. Dapsone
D. Banocidee
E. Ivermectin
E

P –acnes can induce monocytes that activate:


A. TLR-4
B. IL-8
C. TNF δ
D. CDI b
E. Histone H4
B
ONE of the following IS NOT expressed by BASOPHILS :
A. Leukotriens
B. Il-4
C. Il-3
D. IgE
E. Prostaglandin
C
Which of the following drugs inhibits de novo purine synthesis as a mechanism of action?
F. Azathioprine
G. Mycophenolate mofetil
H. Cyclosporine
I. Cyclophosphamide
J. Methotrexate
2

Which one of the following cytokines is a macrophage activator:


A. IFN Y.
B. IL-2
C. IL-4.
D. TNF-alpha.
E. IL-12.
A
Which of the following is involved in stimulating adaptive immunity:
A. IL-1
B. IL-2
C. IL-10
D. IL-12
E. IFN-alpha
B
Keratinocytes have been shown to secrete all of the following cytokines except:
1. IL-1
2. IL-6
3. IL-8
4. TNF-alpha
5. IL-2
5

One of the followings is not correct about immune cells as itch mediators and modulators:
IL-2 causes pruritus
Topical tacrolimus and pimecrolimus are known to inhibit itch.
IL-2-induced pruritus is reduced by antihistimanes
Role TNF-α in itch is unclear
IL-6 and IL-31 have possible involvement in itch.
3

One of the following cytokines is not produced by keratinocytes :


a. IL-1
b. TNF -beta
c. IL- 6
d. TNF ALFA
e. TGF –beta
B

One of the following CYTOKINES is not released by Th0:


A. INF- beta
B. IL-2
C. TNF- alpha
D. IL-10
E. IL-13
A

The proliferation of T cells after activation is primarily controlled by:


A. IL-2 receptor
B. IL-12
C. IL-4
D. IL-4 receptor
E. INF gama
2makki
One of the following cytokines is involved during innate immune response:
A. IL-1
B. IL-6
C. TNF- beta
D. TNF- alpha
E. INF- beta
C except
One of the following cytokines has beneficial effect on wound healing:
F. IL-1.
G. TGF-B.
H. IL-8.
I. IL-6.
J. TNF.
2

Which one is a well recognized Th 1- promoting factor :


a. IL-17
b. IL-23
c. IL-2
d. IL-12
e. IL-4
D
Regarding the cellular mechanisms of vascular leakage, which of the following is INCORRECT:
A. The ‘immediate transient response’ Only Occurs in Venules of 20-6- µm Diameter.
B. Sunburn Can Aause a Delayed and Prolonged Vascular Leakage.
C. The Endothelial Injury Caused By Leukocytes is Due to Free Radical-induced Damage.
D. The Cytoskeletal and Junctional Retraction Mechanism is Mediated by The Complement
System.
E. The Most Common Mechanism For Increased Vascular Permeability is “Endothelial
Contraction” (“immediate transient response”).
D

Complement component C3 is cleaved by:


A. C3b
B. C3bBb
C. Factor B
D. Factor D
E. Factor H
B
Complement receptor 1 ( CR1, CD35 ) IS NOT EXPRESSED ON ONE OF THE FOLLOWING :
A. NEUTROPHILS
B. MACROPHAGE
C. T cells
D. B cells
E. Follicular dendretic cells
E
Which complement component is composed of three separate proteins:
A. C1
B. C2
C. C3
D. C4
E. C5
A
In the complement system, which one of the following is not true?
A. C3a, C5a cause vasodilation, and increased vascular permeability.
B. C5a is a powerful chemotactic agent.
C. C5a activates the cyclo-oxygenase pathway of arachidonic acid metabolism.
D. C5 can be activated by kallikrein.
E. C3b, C3bi can act as opsonins.
C

One of the following shares the same mechanism of action as clindamycin:


A. Azithromycin
B. Doxycycline
C. Ciprofloxacin
D. Rifamycin
E. Penicillin
A
One of the following is a 4th generation cephalosporin?
A. Cefapime
B. Cephalexin
C. Cefaclor
D. Cefotaxime
E. Ceftriaxone
A

1. A young woman with acne taking an oral medication developed polyarthritis and raised liver
enzymes test. The most likely drug she was taking is:
A. Minocycline
B. Erythromycin
C. Oxytetracycline
D. Trimethoprim
E. Isotretinoin
E
A 65 y old man with atypical pneumonia caused by mycoplasma pneumonia will most likely be
treated with:
A. Benzyle pencillin
B. Methicillin
C. Erythromycin
D. Primaquine
E. Thiabendazole
C

For monitoring cyclosporine, Which one of the following is not correct:


a. At least 2 baseline blood pressure reading
b. BUN and creatinie every 2 weeks for 1 – 2 months then monthly
c. Urine analysis monthly
d. Fasting lipid panel every 2-4 weeks for 1 – 2 months then monthly
e. CBC and platelets count every week
E
Which one of the following is CONCERNED with heat regulation:
a. Arteries
b. Veins
c. Lymphatics
d. Venous plexus
e. Capillaries
D
SKIN DAMAGE DURING IMMUNE- COPLEXE REACTION IS MEDIATED BY :
a. histamin
b. neutrophil activation
c. macrophage activation
d. T cell activation
e. NK cell activation
D

Which specific B cell depleting agent has been associated with the development of Kaposi
sarcoma?
A. Prednisone
B. Cyclosporine
C. Methotrexate
D. Cyclophosphamide
E. Rituximab
E
Bone morphogenic proteins (BMPs) are express by cells:
A. Without fibroblast growth factors (FGFs).
B. With FGFs.
C. With activation of Wnt signaling.
D. Without activation of Wnt signaling.
E. With activation of molecular signals.
A

The smallest (in size) DNA virus is:


A. Parvovirus.
B. Adenovirus.
C. Papovavirus.
D. Herpes virus.
E. Togavirus.
A
Which of the following is not true regarding human papilloma virus:
A. It is double standed DNA virus.
B. It is approximatelly 55nm in diameter.
C. It is enveloped virus
D. L1 represent 95% of virion protein.
E. HPV 16 and 18 are ptentially oncogenic.
C

The cause of myiasis is:


A. Tunga penetrans.
B. Mites .
C. Clostiduim tetani.
D. Onchocercariasis.
E. Dermatopia hominis
5
The most common site for flea bite causing tungiasis is:
A. Face.
B. Tongue.
C. Feet
D. Back.
E. Thigh.
C
Tear –drop microconidia is seen in:
A. Trichophyton mentagrophytes.
B. Trichophyton rubrum
C. Epidermphyton floccosum.
D. Microsprum gypseum.
E. Trichphyton violaceum.
B

Hortaea werneckii causes:


A. Tinea nigra
B. Black piedra.
C. Trichomycosis nodularis.
D. White piedra.
E. Madura foot.
A

The site for rotation flap is:


A. Lateral nasal sidewall.
B. Upper lateral lip.
C. Eyebrows.
D. Upper cheek
E. Mid helix.
D
Which of the following is not feature of niesseria gonorrhoea:
A. Is a Gram negative bacteria.
B. Grows in hemoglobin – containing medium.
C. It needs 3-5% CO2.
D. It has polysaccharide capsule
E. It canot tolerate dryness and low temperature.
D

Which is true about autosomal recessive:


A. The disease is seen in sibling of proband not in parent or offsprings
B. The disease is seen in every generation.
C. Recurrence risk is 1 in 2.
D. Affected males have no affected sons.
E. Risk factors is denovo mutation.
A

Which of the following is not ingradient of facial powders:


A. Talc.
B. Kaolin.
C. Ceramides
D. Zinc oxide.
E. Magnesium carbonate.
C
Regarding basic genetic, which of the following is false:
A. Nuclear DNA is packed into chromosomes.
B. Somatic and germline cells contains 2 m of DNA.
C. The paternal and maternal gamates are haploid cells.
D. Telomere constitutes the centre of each chromosome.
E. A gene on chromosome is referred as locus
D

Which of the following is not included among the guidelines of monitoring of antimalarial
therapy:
A. CBC.
B. G6PD.
C. Urine analysis
D. Slit lamp and fundoscopy.
E. Liver function test.
C

Which of the following is not a complication of snake bite:


A. Metabolic acidosis
B. Renal failure.
C. Cardiac arrhythmia.
D. Respiratory failure.
E. Disseminated intravascular coagulation
A

"Flame thrower" appearance is histologically characteristic for:


A. Eccrine gland.
B. Nail bed.
C. Telogen hair
D. Dermal vasculature.
E. Sebaceous gland.
C

Which of the following does not cause high anion gap metabolic acidosis:
A. Renal failure.
B. Diabetic ketoacidosis.
C. Ethylene glycol poisoning.
D. Diarrhoea
E. Metformin intoxication.
D

One of the following statements is not correct about azathioprine:


A. It has immunosuppressive and anti-inflammatory effects
B. It is capaple of influencing both humoral and cell mediated immune functions
C. It inhibits B -cell proliferation
D. It suppresses IgG and IgM formation
E. It has inhibitory effects on suppressor or helper T- cells more than B- cells
E
Which of the following is not correct about mechanism of action of azathioprine:
A. Reduced NK cell function
B. Reduction In CD8+ T cell
C. INHIBITION of lymphocyte proliferation
D. Activation of antibody responses
E. Impaired neutrophil chemotaxis
D
Keratinization of the hair follicle begins at:
A. 8 weeks of EGA.
B. 12 weeks of EGA.
C. 4 weeks of EGA.
D. 28 weeks of EGA.
E. 22 weeks of EGA.
B

The most serious complication of SJS/TEN is related to :


a. Kidney
b. Liver
c. Spleen
d. Eye
e. Pancreas
D
The recommended daily dose of grieofulvin in cutaneous fungal infection is:
A. 10 mg/kg body weight
B. 50 mg/kg body weight
C. 2 mg/kg body weight
D. 100 mg/kg body weight
E. 20 mg/kg body weight
E ‫جواب مكي‬

Flap valves are features of:


A. Glomus bodies.
B. Arterioles.
C. Skin lymphatic vessels
D. Venules.
E. Motor nerves.
C

Phenolic glycolipid-1 is important in identification of:


A. Mycobacterium Kanasasi.
B. Dermatophytes.
C. Moulds.
D. Mycobacterium Leprae
E. Box virus.
D
One of the following is a Third generation antihistamine:
A. Hydroxyzine
B. Loratidine
C. Levocertizine
D. Triproline
E. Ciproheptadine
C

Regarding the arterial pulse and pulse rate:


A. The Pulse at the wrist is due to the flow of blood through the artery
B. The radial Pulse is synchronous with the 1st heart sound
C. Hardening and tortuousity of the radial artery is an indication of atheroma elsewhere
D. Sinus bradycardia may result from a rising intracranial pressure
E. Pressure on the carotid Sinus may not reduce the rate in atrial flutter.
D

Which medium is used for fixation and transport of biopsy specimen to be used for
immunofluorescence technique:
A. Neutral buffered formalin solution.
B. Michel solution
C. Zenker’s fluid.
D. Formal saline solution.
E. Frozen tissue.
B

Poor skin tests to tuberculin indicate a deficiency of


A. NK cells
B. T cells
C. Phagocytosis
D. B cells
E. Mast cells
B

Viruses multiply by:


A. Binary fission
B. Mitosis
C. Meiosis
D. A complex life cycle
E. Conidia
D
HIV receptor on the host cell is:
A. Plasma membrane
B. Mitochondria
C. CD+4
D. Ribosomes
E. Plasmids
C
Which One of the following disorders is the most likely to be associated with helicobacter pylori
infection:
A. NON- ulcer dyspepsia
B. Reflux esophagitis
C. Coeliac dis
D. Gastric lymphoma
E. Achalasia of the cardia
D

The surface density of epidermis is approximately:


A. 10.000 nucleated cell/ mm.
B. 20.000 nucleated cell/mm.
C. 30.000 nucleated cell/mm.
D. 40.000 nucleated cell/mm.
E. 50.000 nucleated cell/mm
E

Very common loss of function mutations in the fillagrin gene causes all of the followings
EXCEPT:
A. Genetic disorder of Ichthyosis vuglaris.
B. Risk factor for the development of atopic dermatitis.
C. Atopic asthma.
D. Contact dermatitis
E. Systemic allergies.
D
One of the following is not a function of IL-1:

A. Suppression of migration and activation of Langerhans cells.


B. T cell proliferation.
C. Increase expressions of adhesion molecules on endothelium.
D. Fever.
E. Induction of chemotaxis and cytotoxicity of monocytes.
A

In a female patient presenting with acne which of the following indicates to the testing for
hyperandrogenism ?
A. Premenstrual flare
B. Submarine comedones
C. Pitted scars in the checks
D. Location of lesions in the trunk
E. Sudden onset of severe lesion
E

The epidermis begins to develop at:


A. 4th week of intrauterine life
B. 7th week of intrauterine life
C. 14th week of intrauterine life
D. 10th week of intrauterine life
E. 22nd week of intrauterine life
A

Which of the following is NOT true about elliptical incision biopsy:


A. It is indicated when pathology is epidermal
B. The skin should be anesthesized
C. The scalpel blade no. 15 is used
D. It is closed using rule of halves
E. The length of incision is 3 times of the width
A

Phenolic glycolipid-1 is important in identification of:


A. Mycobacterium Kanasasi.
B. Dermatophytes.
C. Moulds.
D. Mycobacterium Leprae
E. Pox virus.
D

Which of the following suture type has the most persistent tensile strength over time?
a. Braided polyester
b. Polygdioxanone
c. Polyglactin 910
d. Poliglecaprone
e. Silk
A

Psoriasis is characterizsd by keratinocyte hyperproliferation maintained by:


A. Genetically aberrant keratinocyte progenitor cells
B. Persistent stimulation of B- cell by immunogen
C. Accumulation of activated CD8+ and CD 4+ T-Cell in skin
D. Differentiation and expansion of type 2 T-Cell
E. Activated macrophage in the skin
C

Chronic HBV infection is marked by:


A. Incubation period
B. HBs Ag
C. Anti –HBc (IgM)
D. Anti- HBs
E. Anti –HBc (IgG)
B

Hutchinson nail sign is periungual extension of pigment to the proximal and lateral folds of the
nail and is noted in:
A. LP
B. Traumatic purpura
C. Onychomycosis
D. Subungual melanoma
E. Periungual fibroma
D

In which of the following causes of pruritus there is dysregulation of central opiod peptides:
A. Urticarial
B. Scabies
C. Cholestasis
D. Small fibre neuropathy
E. Delusional parasitosis
C

One of the following is not correct about vit B7 (biotin) deficiency:


A. Alopecia
B. Seborrheic Dermatitis
C. Nummular eczema
D. Erythroderma in neonatal form
E. Atopic Dermatitis
E

The epidermis is a dynamic tissue in which cells are in constant motion. The kinetics of this
motion is characterized by:
A. Direction and speed are synchronous
B. Keratinocytes pass each other but not melanocytes
C. Movement is unidirectional
D. Stability for Direction and flow is provided by the basal membrane complex
E. Lateral motion is slower than forward
D

Combining tetracycline and isotretinoine for the treatment of the severe acne should be avoided
because of Which of the following ?
A. Multiple exuberant bone growth
B. Intracranial hypertension
C. Severe chilostomatitis
D. Nasal bleeding
E. Myalgia and arthralgia
B

Hortaea werneckii causes:


A. Tinea nigra
B. Black piedra.
C. Trichomycosis nodularis.
D. White piedra.
E. Madura foot.
A
the incubation period of tinea nigra is :
a. 3 – 5 days
b. 1 week
c. 10 – 15 days
d. 21 days
e. 60 – 90 days
C

.
One of the following is not correct about mast cells:
A. They possess a central round to ovoid dark staining nucleus
B. The cytoplasm contain small granules
C. In normal skin can occur around blood vessels with dendritic morphology
D. They stain methachromatically with alcian blue
E. Commonly seen in various benign and malignant tumors
D

In which one of the following glycosaminoglycans there is no core protein?


A. Dermatan.
B. Chondroitin.
C. Keratan.
D. Heparan.
E. Hyaluronan.
E

Concerning wound healing, which of the following statement is not correct:


A. If the injury is limited to the epidermis it restores itself to a structure similar to the preinjury
state.
B. In case of injury to the dermis, regeneration does not occur normally.
C. Partial thickness wound involves the epidermis and part of the epidermis.
D. In full thickness wounds, the adnexal structure serves as reservior of epithelial cells for
repopulating the epidermis
E. In full thickness wound, epithelium can only migrate the ulcer edge.
D

Which cell type is most susceptible to cold injury during cutaneous cryosurgery:

A. Perineural
B. Endothelial
C. Fibroblast
D. Melanocyte
E. Keratinocyte
D

Cyanosis:
A. Is yellowish discoloration of the sclera.
B. Is produced by high pCO2.
C. Can easily occur in Anemia.
D. Occurs more with polycythemia.
F. Is due to high oxygenated.
D
‫ألحل النهائي‬

MAST cell granules do not contain:


A. Histamine
B. Heparin
C. Tryptase
D. TNF-alpha
E. TNF-beta
E

The first cell type to migrate into a new wound in great numbers is the:
1. Neutrophil
2. Monocyte
3. Macrophage
4. Lymphocyte
5. Mast cell
A

SKIN DAMAGE DURING IMMUNE- COPLEXE REACTION IS MEDIATED BY :


a. histamin
b. neutrophil activation
c. macrophage activation
d. T cell activation
e. NK cell activation
D

Irrevesible cellular injury does not cause:


A. Progressive loss of phospholipids.
B. Decreased activity of Na+ / K+ AT pase causing cellular swelling.
C. Damage to the cellular cytoskeleton.
D. The presence of reactive O2 species.
E. The loss of the intracellular amino acids glycine and L-alanine.
E

Antimicrobial substances on the surface of the skin include:

A. Special peptides
B. Phospholipids
C. Pyrilladone
D. Alphacarboxylic acids
E. Glycolipids
A

Prostaglandins are derived from


A. Lysosomes
B. Golgi apparatus
C. Mitochondria
D. Phospholipids of cell membrane
E. As a constituent of the cytoplasm
D

Antigen presenting cell (APC) IS NOT associated with induction of:


A. IL-1 B
B. IL-10
C. IL-12
D. IL-6
E. Chemokines
A

One of the following cytokines is involved in stimulating adaptive immunity:?


F. IL- 1
G. IL- 2
H. IL- 10
I. IL- 12
J. IFN- alpha
2.4
Adaptive 2-4-12-17-22-23-ifn ¥

The following cytokines are involved during an innate immune response EXCEPT:
IL-1.
IL-6.
TNF-ß
TNF-α
IFN-ß
3
Innate 1–6–18–tnf a—ifn a-b—chemokines

One of the following is not an oxygen-dependant killing mechanism in Neutrophils:

A. Lysozyme
B. Lactoferrin
C. Major basic proteins
D. Cationic proteins
E. Hydrogen peroxidase

Which of the following substances has the greatest permeability across the basement
membrane:
A. Glucose.
B. Albumin.
C. Aliphatic Alcohol.
D. Dextran.
E. Sodium Chloride.
E

Cyanosis:
A. Is yellowish discoloration of the sclera.
B. Is produced by high pCO2.
C. Can easily occur in Anemia.
D. Occurs more with polycythemia.
F. Is due to high oxygenated.
D

One of the following adhesion proteins is not a component of hemidesmosome anchoring


filament complex:
A. Laminin 311.
B. Plectin.
C. Bullous pemphigoid antigen 1.
D. Integrine.
E. Tetraspan CD 151.
A

One of the following adhesion proteins is not observed in lamina densa:


K. Perlecan
L. Lamin 311
M. Lamin 332
N. Elastin
O. Type IV collagen
D

The most important proteoglycan in the dermis:


A. Versican
B. Perlecan
C. Biglycan
D. Keratocan
E. Syndecan
A

One of the following proteoglycans is a component of the dermo-epidermal junction?


A. Decorin
B. Versican
C. Syndecan
D. Chondroitin sulphate
E. Perlecan
E

The main permeability barrier in the lamina densa is:


A. Heparin sulphate proteoglycan.
B. B-Collagen IV.
C. Laminin 5.
D. Nidogen.
E. Alpha-6-beta-4 integrin.

The immediate tanning of skin is mainly caused by:


A. UVA
B. UVB
C. UVC
D. IR
E. Visible
A

Concerning UV light, which statement is not true:


A. Wave lengh of 360 nm is 1000-fold less erythemogenic than 300nm.
B. An immediate erythema reaction is rarely seen I UVB –induced sunburn.
C. DNA is hypothesized to be chromophore for UVB erythema.
D. Delayed tanning is usually as result of exposure to UVA.
E. Fair skin usually tans only with UVB doses above the erythema threshold.
4

Palmoplantar eccrine sweat glands are fully developed by:


A. End of the 1st trimester
B. During the 1st trimester
C. During the 3d trimester
D. During the 2d trimester
E. End of the 3d trimester
D

One of the following is not correct about special stains in Dermatology ?


A. Alcian blue (PH 0.05) stains sulfated mucopolysaccharide
B. Masson”s trichrome stains smooth muscle green
C. Periodic acid Schiff (PAS ) stains fungal walls red
D. Pagoda stains amyloid orange
E. Myeloperoxidase stains immature myeloid cell orange
B

One of the following is not correct about staining:


a. Masson trichrome stains elastic fibers
b. Alcian blue PH2.5 stains acid MPS
c. Verhoeff Van gieson stains elastic fibers
d. Scarlet red stains lipid
e. Oil red O stains lipid
A

The best stain to demonstrate collagen is:


A. Scarlet red
B. Toluidine blue
C. GMS
D. Masson trichrome
E. Hyaluronidase
D

Alertnate morning glucocorticosteriod therapy is not useful in reduce risk in which of the
following:
A. Myopathy.
B. Growth suppression.
C. Cataract.
D. Hypertention.
E. Opportunistic infection.
C

A minimal erythema dose (MED) phototest should be read at:


A. Two hours.
B. 24 hours
C. 48 hours.
D. 12 hours.
E. 96 hours.
B

The first therapeutic dose of NB-UVB therapy is:


A. 40% of MED.
B. 70% of MED.
C. 20% of MED.
D. 100% of MED.
E. 10% of MED.
B

Profillagrin is evident in:


A. Stratum corneum.
B. Stratum granulosum.
C. Stratum lucidum
D. Stratum basale.
E. Stratum spinosum.
B

In which of the following filaggrin break-down amino acids are involved?


A. Corneodesmolysis.
B. White blood cell chemotaxis to the epidermis.
C. Skin antimicrobial defense.
D. Vasodilatation.
E. Water retention in the epidermis.
E

Melanocortin MC-4 receptor is found in:


A. Pripherial tissue.
B. Placenta.
C. Adrenal cortex.
D. Melanocyte.
E. Brain.
E
1. Melanocytes
2adrenal cortex
3brain
4brain
5 peripheral tissues
Which one of the following melanocortin receptors is majorly distributed in cortex?
A. MC1R
B. MC2R
C. MC3R
D. MC4R
E. MC5R
B

A child present with 2 circular lesions that show scales only with partial loss of scalp hair. What
is the most likely diagnosis?
A. Alopecia Areata
B. Tinea capitis
C. Folliculitis
D. Favus
E. Kerion
B

One of the following statements about keratins is false:

A. Keratin filaments are the hallmark of keratinocytes.


B. All keratins are within 30-40 Kd molecular mass.
C. Most acidic keratin genes localize to chromosome 12 q11, q13
D. Basal cells has a large nucleus with prominent nucleolus.
E. Not all basal cells have the potential to divide.
C

Which of the following is a substrate for matrix metalloproteinases?


A. Corneosome.
B. Keratin.
C. Collagen
D. Cornifin.
E. Filaggrin.
C

Keratin filaments in basal cells insert into:


1. Desmosomes
2. Adherens junctions
3. Connexins
4. Lamellar granules
5. Odland bodies
A

Early cornification can be observed within the hair canal at approximately:


A. 12 weeks estimated gestational age
B. 18 weeks estimated gestational age
C. 15 weeks estimated gestational age
D. 22 weeks estimated gestational age
E. 8 weeks estimated gestational age
C

One of the following is not correct about Favus:


A. It caused by T.schoenleinii.
B. Hyphae and air spaces are observed within the hair shaft.
C. By Wood’s light there is bluish-white fluorescence.
D. Scutula is characteristic clinical feature.
E. Non scarring alopecia may develop in chronic infections.
E

Which of the following is characteristic symptom of hypomania:


A. Irritability.
B. Inability to concentrate.
C. Exaggerated feeling of well-being.
D. Increased sleepiness.
E. Tiredness.
C

The arrector pili muscle:


A. Develop from a bulb on the lateral side of hair canal
B. Function as a support for sebaceous gland
C. Participate in thermoregulation
D. Contracts in response to raised temperature
E. Is attached to basement membrane
C

One of the following lipids is not produced by sebaceous glands:


A. Triglycerides
B. Phospholipids
C. Esterified cholesterol
D. Free cholesterol
E. Waxes
B

Sebaceous glands open directly to the surface of the skin in :


A. Eyelids
B. Scalp
C. The external auditory meatus
D. Upper trunk
E. Face
A

Which of the following is not a specialized type of sebaceous gland?


1. Moll's gland
2. Meibomian gland
3. Gland of Zeis
4. Montgomery's tubercle
5. Fordyce spot
A

Regarding sebaceous glands:


1. These glands are present at birth at their adult size
2. Size of the gland is proportional to the size of the associated hair follicle
3. Are always associated with a hair follicle
4. Are found everywhere on the skin except palms and soles
5. Are unilobular glands
D

Sebaceous glands secrete sebum through which of the following secretory mechanisms?
1. Holocrine
2. Merocrine
3. Apocrine
4. Holocrine and Merocrine
5. Holocrine and Apocrine
A

Which of the following antimicrobial peptides is expressed in sebaceous gland:


A. Antileukoprotase
B. Cathelicidin
C. Psoriasis
D. β- defensing 1
E. β- defensing 2
C

Sebaceous glands:
1. Respond to chemical stimuli such as hormones
2. Respond to cholinergic neural activity, exclusively
3. Respond to adrenergic neural activity, exclusively
4. Respond to both adrenergic and cholinergic stimuli
5. Respond to the local release of cytokines from inflammatory cells
A

Sebaceous glands are located in each of the following locations except:


1. Nipple
2. Labia minora
3. Palms
4. Eyelids
5. Buccal mucosa
3

Proliferation and secretion of the sebaceous glands are controlled by:

A. Insulin
B. Estrogen
C. Thyroxine
D. Testosterone
E. Cortisol
D

Which of the following glands is not under neural control?


1. Sebaceous glands
2. Apocrine glands
3. Eccrine glands
4. Salivary glands
5. Ceruminous glands
A

Human sebum is distinguished from lipids of internal organs by the presence of:
1. Cholestrol
2. Cholestrol esters
3. Squalene
4. Wax esters
5. Glycerides
4

What is the most commonly mood stabilizer used in hypomania:

A. Lithium
B. Benzodiazepines
C. ECT
D. Antipsychotics
E. Phenothiazines
A

Which of the following is not a feature of major depression?


A. Anhedonia.
B. Irritability.
C. Insomnia or hypersomnia.
D. Psychomotor agitation or retardation
E. Suicidal ideation.
D

Which of the following is false about depression:


A. Feeling of guilt.
B. Suicidal thinking.
C. Loss of interest.
D. Weight changes.
E. Flight of ideas.
E
Meibomian glands are:
1. Eccrine glands localized to the vermillion border of the lips
2. Sebaceous glands found on the areola of the breast
3. Sebaceous glands found on the eyelids
4. Apocrine glands found in the anogenital regions
5. Apocrine glands found on the eylelids
C

Hair perforation test is useful to distinguish Trichophyton rubrum from:


A. Epidermophyton floccosum.
B. Trichophyton schoenleinii.
C. Microsporum gypsum.
D. Trichophyton mentegrophytes
E. Trichophyton verrucosum.
D

Positive hair perforation test is caused by:


A. T. mentagrophytes.
B. M. audouinii.
C. T. schoenleinii.
D. T. concentricum.
E. T. rubrum.
A

All the following are features of epidermophyton conidia EXCEPT:


A. They are smooth walled
B. Micro conidia are numerous and diagnostic
C. It is club shaped
D. May be solitary or clustered
E. The cell wall thickness is intermediate.
B

One of the following is a zoophilic dermatophyte:


A. T.rubrum.
B. T.tansurans.
C. T.concentricum.
D. T.verrucosum.
E. Epidermophyton floccosum.
D

One of the following is not bactericidal:


A. Aminoglycosides.
B. Penicillins.
C. Sulphonamides.
D. Quinolones.
E. Vancomycin.
C

The labeling index equals the proportion of cells in which phase:


A. The Mitotic.
B. S phase.
C. G1.
D. G2.
E. G0.
2

A patient of herpes zoster developed granuloma annulare at the same site. What is this
phenomenon/sign?
A. Pathergy.
B. Koebner.
C. Reverse Koebner.
D. Autosensitization.
E. Isotopic.
E

One of the following is class D2 corticosteroids:


A. Betamethasone dipropionate
B. Betamethasone 17 valerate
C. Hydrocortisone valerate
D. Clobetasol dipropionate
E. Mometasone furoate
C

One of the following is not a clinical feature of secondary syphilis:


A. Piyriasis rosea-like.
B. Generalized papulosquamous.
C. Pink papules.
D. Bullous.
E. Lesions of the palms and soles can have a collarette of scale.
D

,Fleeting arthritis is characteristic clinical feature of:


A. Ankylosing arthritis.
B. Gouty arthritis.
C. Rheumatic arthritis.
D. Septic arthritis.
E. Rheumatoid arthritis.
C

One of the following is not true about treponema pallidum:

A. Generally shows 8 – 12 spiral convulsions


B. Stained by silver stain
C. Septrin sensitive
D. Protected from phagocytosis by polysaccharide capsule
E. It does not produce exotoxin
D

Which of the following is not characteristic in treponema pallidum:


A. 6-20 µm in length.
B. Regular tight spirals.
C. Limited capacity for DNA repair.
D. Periplasmic flagella.
E. Ability to survive outside an animal host.
E

Which of the following listed facts is incorrect during patient education of Herpes Zoster?
A. Most people recover over several weeks.
B. Workup for immunodeficiency is unnecessary.
C. All adults age 50 and older should get a vaccine.
D. The vaccine is live attenuated virus.
E. This is not sexually transmitted.
2
,Which of the following is not true about newborn skin:
A. Stratum corneum thickness is 9-10 µm.
B. Spinous cell glycogen content is abundant.
C. Similar number of melanocytes to adult.
D. Elastic fibers are small and immature.
E. The epidermal surface has vernix.
2

Keratinization of the hair follicle begins at:


A. 8 weeks of EGA.
B. 12 weeks of EGA.
C. 4 weeks of EGA.
D. 28 weeks of EGA.
E. 22 weeks of EGA.
B

In the embryo when the periderm is lost it is replaced by:


A. Stratum Corneum
B. Basal Layer.
C. Transient Layer.
D. Granular Layer.
E. Basement Membrane.
A

Which of the following is true regarding scarlet fever?


A. The incubation period is usually short, lasting only a few days.
B. Forchheimer spots, seen on the tongue and gums, are signs of this disease.
C. The typical rash spares folds of skin, which are known as Pastia lines.
D. Mediated by pyogenic exotoxin E.
E. The rash typically appears on the dorsal hands and spreads to others parts of the body.
A

Main inflammatory mediator in inflammatory phase of wound healing is:


A. IL-6.
B. IL-12.
C. TGF-alpha.
D. PDGF.
E. IL-4.
A

Dopamine:
A. Is secreted from the anterior pituitary gland.
B. Stimulates prolactin secretion.
C. Is the precursor for adrenaline and nor adrenaline.
D. Transported to the pituitary gland via axons.
E. Can be released by parasympathetic nerves.
C

Regarding intrauterine HSV infection, which of the following findings would LEAST LIKELY be
present in the aff newborn during the neonatal period?
A. Microcephaly.
B. Cutaneous lesions.
C. Chorioretinitis.
D. Sezures.
E. Sepsis.
5

One of the following drugs is not an inducer of psoriasis:


A. Lithium.
B. B-blockers.
C. Antidepressants.
D. IFNs.
E. Antimalarial.
C

In the complement system, which one of the following is not true?


A. C3a, C5a cause vasodilation, and increased vascular permeability.
B. C5a is a powerful chemotactic agent.
C. C5a activates the cyclo-oxygenase pathway of arachidonic acid metabolism.
D. C5 can be activated by kallikrein.
E. C3b, C3bi can act as opsonins.
3
A test which detects the presence of a multiple number of antigens:
A. Complement fixation test
B. Passive latex agglutination
C. Direct fluorescent antibody test
D. Indirect fluorescent antibody test
E. Immuno-electrophoresis
5

The best drug for treatment of cytomegalovirus (CMV) infection is:


A. Valacyclovir.
B. Ganciclovir.
C. Foscarent.
D. Acyclovir.
E. Cidofovir.
C??

The plasma half-life 19-35 h is characteristic of the following antihistamine:,,,24-36hr

A. Hydroxyzine
B. Desloratadine
C. Loratadine
D. Levocetrizine
E. Fexofenadine
B

One of the following cytokines has beneficial effect on wound healing:


F. IL-1.
G. TGF-B.
H. IL-8.
I. IL-6.
J. TNF.
2

Which of the following is the action of Glutathione?


A. Degrader of keratin filaments.
B. Solvent for the cornified envelop.
C. Antioxidant.
D. Sebum emulsifier.
E. Stabilizer for gap junction.
C

Identification of HSV by culture usually requires:


A. 24-36 hours.
B. 2-5 days.
C. 7 days.
D. 2-3 weeks.
E. Not cultured.
B

Which tattoo color produces the most photosensitivity reactions?


A. Red
B. Blue
C. Green
D. Yellow
E. White
D

Interferon beta is produced by:


A. Fibroblast
B. Macrophage
C. Activated T cell
D. B – cell
E. All of the above
A

Which of the following is not a feature of anxiety neurosis:


A. Confusion.
B. Intial insomnia.
C. Panic attacks.
D. Tremor.
E. Frequency of micturition.
C

-Which of the following is not true about systemic lupus erythromatosus:


A. It is more common in women.
B. Remissions is commonly occur in pregnancy.
C. Anti-DNA antibodies are absent in drug induced lupus.
D. Antimalarial can reduce frequency of exacerbations.
E. Renal involvement carries the worest prognosis.
3??2?.
B

Primary cell in creating granulation tissue is:

A. Neutrophil
B. Macrophage
C. Fibroblast
D. Keratinocyte
E. Myofiboblast
3

A pregnant lady was referred from the antenatal care to dermatology clinic with the diagnosis of
severe pruritus gravidarum. What are the type of lesions that consolidate the diagnosis?
F. Excoriations.
G. Oozing plaques of dermatitis.
H. Urticated papules and plaques.
I. Erythematous follicular papules.
J. Blisters.
1

The proliferation of T cells after activation is primarily controlled by:

A. IL-2 receptor
B. IL-12
C. IL-4
D. IL-4 receptor
E. INF gama
B
Which of the following medications is concentrated in the eccrine glands?
1. Cyclophosphamide
2. Cytarabine
3. Ciprofloxacin
4. Cephalexin
5. All of the answers are correct
5

Human sebum is distinguished from lipids of internal organs by the presence of:
1. Cholestrol
2. Cholestrol esters
3. Squalene
4. Wax esters
5. Glycerides
4

Which keratins are upregulated in hyperproliferative disease such as psoriasis?


1. Keratins 1 and 10
2. Keratins 2e and 10
3. Keratins 5 and 14
4. Keratins 6 and 16
5. Keratins 8 and 18
4

Tinea manuum :
A. Dermatophyte infections on the dorsal aspect of the hand
B. Dermatophyte infection of the palm and interdigital spaces have a clinical presentation similar
to Tinea corporis
C. The reason for the two different clinical pictures is thought to be related to the lack of
sebaceous glands on the palms
D. Interdigital type Tinea pedis is often present in patients with Tinea manuum
E. Tinea manuum is usually non-inflammatory and often bilateral
C

Highest density of hairs is usually in:


A. Forehead
B. Eyebrow
C. Scalp
D. Chin
E. Upper lip
A

Side effect of Bleomycin includes:


A. Raynaud’s phenomenon.
B. Sedation.
C. Discoloration.
D. Xerosis.
E. Ocular irritation.
A

Regarding nail development , proximal nail fold is visible at:


A. 8 weeks of EGA
B. 10 week of EGA
C. 13 week of EGA
D. 15 week of EGA
E. 18 week of EGA
C

Regarding Th3 cells, One of the following is not true:


A. Th3 cells are a poorly characterized type of CD4+ Th cell.
B. Primerily secretes TNF-beta........//secrete TgfB
C. Provides help for IgA production
D. Has suppressive properties against Th1
E. Has suppressive properties against Th
B

Which of the following is a fibril-forming collagen?


A. COL.III
B. COL IV
C. COL V
D. COL VI
E. COL VII
C

In cases of shock, which of the following compensatory factor is responsible for increasing blood
pressure:
A. Atrial stretch receptors.
B. Barao receptor reflex.
C. Bainbridge reflex.
D. Carotid body chemo receptors
E. Ischemic brain response.
B baro

Tetracyclines have anti-inflammatory effects which include:


A. Increasing matrix metalloproteinase activity
B. Inhibit leukocyte chemotaxis
C. Increase production of TNF a
D. Increase production of IL 1B
E. Decrease production of IL 1
B

The following causes vasodilatation in acute inflammation:


A. Nitric oxide
B. Leukotriene B4
C. Prostacycline
D. IL8
E. Fibronectin
A

An elderly patient presents with recurrent attack of painful unilateral papulovesicular lesions on
the trunk with a dermatomal distribution. What is the next appropriate step?
A. Swabs for bacterial culture.
B. Neurological clinical evaluation.
C. Topical acyclovir and rest at home.
D. Investigation for underlying malignancy.
E. Punch biopsy.
D

Higher risk for toxicity from glucocorticoid therapy does not occur in Which of the following ?
A. Female patients
B. Patients with rheumatoid arthritis
C. Patients with dermatomysitis
D. Patients with hyperalbuminemia
E. Patients who smoke
D

Which of the following antimicrobial peptides is not expressed in sebaceous gland:


A. Antileukoprotase
B. Cathelicidin
C. Psoriasis
D. β- defensing 1
E. β- defensing 2
C
‫ بدون‬not
Psoriasin ‫هو الوحيد بسبيشيس‬

The highest incidence of drug reaction is with:


A. Penicillin G
B. Barbiturates
C. Cephalosporins
D. Ampicillin
E. Packed red blood cells
A
Which of the following has a protective role in urticarial:
A. Leukotriene C4
B. TNF-β
C. Prostaglandins E2
D. Platelet-activating factor
E. IL-4
C

Complications of bacterial endocarditis include one of the following:


A. Renal abcesses
B. Splenic rupture
C. Immunocomplex glomerulonephritis
D. Heart failure
E. DIC
C

Which part of the nail unit most closely resembles the epiderm histologically?
A. Lunula
B. Cuticle
C. Nail Bed
D. Nail Plate
E. Hyponychium
E

Sweat ducts are not affected in:


A. Lichen Planus
B. Ichtyosis
C. Psoriasis
D. Palmoplantar Pustulosis
E. Atopic Dermatitis
E

Which of the following retinoids has THE LONGEST elimination half life:
A. Isotretinoin
B. Alitretinoin
C. Etretinate
D. Acitretin
E. Bexarotene
C

Which of the following is the most abundant anaphylatoxin?


M. C2a
N. C3a
O. C3b
P. C5a
Q. C5b
4
MHC class I-restricted T-cell-mediated cytolosis appears to be involved in the followings
EXCEPT:
A. Lichen planus
B. Fixed drug eruption
C. Cutaneous GVHD.
D. Psoriasis
E. Herpes simplex virus infections.
E

The chemical mediators that plays a role in cross-linking of wound healing?


M. Kinin
N. Serotonin
O. Histamine
P. Prostaglandins
Q. Complement
Makki3
Book2

Eccrine and apocrine epithelium are stained by One of the following immunohistochemical
stains:
A. CEA
B. Bcl-2
C. Epithelial membrane antigen EMA
D. P53
E. P63
C
Which of the following facts about histamine is true:

A. Histamine is a potent activator of mast cells.


B. The major source of histamine in the skin are keratinocytes
C. There are currently 6 known histamine receptors
D. The H2 receptors is the receptor mainly responsible for pruritus
E. In addition to H1, H3 and H4 receptors can also modulate pruritus
E

A 42 y old housekeeper has a chronic hand dermatitis and is complaining of severe pruritus.
One of the following is not class I corticosteroid?
A. Halobetasol o.o5% cream
B. Fluocinonide o.1% cream
C. Clobetasol o.o5% lotion
D. Diflorasone diacetate o.o5% oint
E. Mometasone o.1% cream
C

Antigen presenting cell (APC) IS NOT associated with induction of:


A. IL-1 B
B. IL-10
C. IL-12
D. IL-6
E. Chemokines
B

Antigen presenting cells (APC) activation is not associated with induction of the following
cytokine:
A. IL-1&.
B. IL-6.
C. IL-12.
D. Chemokine’s.
E. MHC class I&II.
A

Cyanosis:
A. Is yellowish discoloration of the sclera.
B. Is produced by high pCO2.
C. Can easily occur in Anemia.
D. Occurs more with polycythemia.
F. Is due to high oxygenated.
C

Cell cycle of basal cells:


A. G1 phase has the greatest variability in duration
B. Duration of G1 phase is not governed by the physiological state
C. S phase lasts from 2 – 4 hours in most human cells
D. G2 phase lasts from 6 – 16 hours
E. G0 phase is a must for all cells before going into G1
A

Melanocytes first appear in:


A. Upper extremities.
B. Trunk.
C. Head.
D. Genitalia.
E. Axilla.
C

Which of the following is not true about newborn skin:


A. Stratum corneum thickness is 9-10 µm.
B. Spinous cell glycogen content is abundant.
C. Similar number of melanocytes to adult.
D. Elastic fibers are small and immature.
E. The epidermal surface has vernix.
B

Which of the following is not a component of the epidermal differentiation complex:


A. Loricrin
B. Involucrin
C. S – 100 calcium-binding proteins
D. Small proline- rich peptides
E. Transglutaminase
E

One of the following is not an indication for electrofulguration/electrodessication (superficial skin


ablation ):
A. Actinic keratosis
B. Epidermal nevus
C. Angiofibroma
D. Lentigo
E. Seborrheic keratosis
C

One of the following is not correct about eccrine apparatus anatomy:


A. They are absent on nail beds
B. Secretary layer has large vesicular cells and small columnar cells
C. Myoepithelial layer is absent in eccrine ducts
D. They empty directly into epidermis
E. Dermal duct is situated within reticular dermis
B

One of the following histopathologic findings is characteristic for dg of BCCarcinoma:


A. Solid lobules of basaloid cells with jigsaw-puzzle appearance and thickened basement
membrane
B. Masses of basaloid cells with shadow keratinization and foreign body reaction
C. Solid lobules of basaloid cells composed of individual nodules of 2 cell type.
D. Basaloid lesion with mature and immature hair follicle formation.
E. Solid lobules of basaloid cells with sebaceous differentiation
C

Neutrophil polymorphs:

A. Have multilobed nuclei


B. Are the predominant cell type in chronic inflammation
C. May fuse to form multinucleate giant cells
D. Have phagocytic abilities
E. Have numerous eosinophilic granules in their cytoplasm
A

Retinoid topical preparation pass into te keratinocytes through the process of :


A. Attachment to special cell wall receptors
B. Phagocytosis
C. Liquefaction of the cell membrane
D. Osmosis
E. Diffusion gradient
E

Elastic connective tissue:


A. Extends from papillary dermis to reticular
B. Is absent in hair follicules sheathes
C. mature Elastic fibers contain 75% elastin——-/90%
D. oxytalan fibers form microfibrils containing soluable Elastin
E. mature Elastic fibers are most abundant in middermis
D

Human dendritic cells are generated from:


A. Spleen
B. Bone marrow
C. Peripherial blood
D. Regional lymph nodes
E. Thymus
C 👍 👍 bolognia

Gene signature is:


A. The relative position of genes along the chromosome
B. The expression level of a gene
C. A pattern of genetic markers which infer a biologic state
D. The effect of one gene’s expression level on other genes
E. A measure of a gene’s relative importance in a cell type
C
Corneal layer cells are:
A. Firmly attached to each other by desmosomes
B. Metabolically active
C. Surrounded by lamellated lipids
D. Have an outer protein envelope
E. Cytoplasm is filled with fibrillar keratin and reticulin
D

The epithelial cells are derived from


A. Monocytes
B. Lymphocytes
C. Astrocytes
D. Eosinocytes
E. Basocytes
E
‫تأكدي‬

Which one is not a manifestation of Epstein – Bar virus :


a. Gianoti – Crosti syndrome
b. Genital ulcers
c. The papularpurpuric gloves and socks syndrome
d. EM
e. Oral hairy leukoplakia
B
,Which of the following provides the principal barrier to transepidermal water loss:
A. Involucrin.
B. Profillagrin.
C. Lamellar bodies.
D. Lorocrin.
E. Fillagrin
Thalidomide downregulates:
A. Prostaglandin D2.
B. β2- Intregrin and β1 &α4 integrin
C. Calcineurin.
D. CYP3A4.
E. IL-8.
B

The chemical mediators that plays a role in cross-linking of wound healing?


M. Kinin
N. Serotonin
O. Histamine
P. Prostaglandins
Q. Complement
2

1 18
Regarding Passini’s corpuscle:
A. Possesses a characteristic capsule but no lamellar wrapping.
B. Capsule is arranged in 20 layers of cells
C. Slowly adapt to mechano-stimuli
D. Subcapsular zone contains collagen and fibroblasts
E. Capsule is wrapped in lamellar Schwan cells
A

1-When examining a pathology slide with subepidermal blisters with eosinophils, Which of the
following is the most probable diagnosis:
A. PV
B. EBA
C. DH
D. Linear IgA Dermatosis
E. BP
5

2-What is the initial mainstay therapy for severe Pemphigus Vulgaris:


A. Topical corticosteroid
B. Rituximab
C. Systemic corticosteroid
D. Minocycline
E. Azathioprine
C
3-The primary focus of pulmonary tuberculosis is characteristic by:
A. Cavitation
B. Occurs in the apex of the lung
C. Is mostly asymptomatic
D. Is complicated by military tuberculosis
E. Presents with haemoptasis
C

4Psoriasis has many features involving the nails. Which one is most frequently seen:
A. Discoloration
B. Nail plate dystrophy
C. Onycholysis
D. Pitting
E. Subungueal hyperkeratosis
D

5-A 65 y old man with atypical pneumonia caused by mycoplasma pneumonia will most likely be
treated with:
A. Benzyle pencillin
B. Methicillin
C. Erythromycin
D. Primaquine
E. Thiabendazole
C

6-Which of the following vitamine deficiencies has no mucocutaneous findings:


A. B1
B. B5
C. B3
D. B6
E. B7
B

7-Regarding skin Prick test, One of the following is not true:


A. Cheap
B. Results are available in 45 mn
C. Can be used with ad hoc allergens (e.g. fresh, foods, drugs)
D. Cannot be done in patients receiving antihistamines and calcium channel blockers
E. Requires skilled staff for reproducible Results
A

8-Free Radicals:
A. Are not a byproduct of metabolism.
B. Can only form in the presence of oxygen
C. May arise by absorption of radiant energy.
D. Are removed with the aid of vitamin A derivatives.
E. Contain an extra protein that may bind to and destroy organic molecules.
B
9-The so called veil cells are located around:
A. Sebaceous gland
B. Hair follicules
C. Lymphatics
D. Arterioles
E. Capillaries
5

10-Fibrillin is:
A. A macroprotein.
B. Distributed Closely Parallel to Collagen Fibres.
C. Assists in Destruction of Elastin.
D. Plays an Adhesive Role between Matrix Components
E. Abundant in Reticular Layer.
4

11-The following is true about the dermis:


A. Undergoes an obvious sequence of differentiation
B. Its structure and organization are not predictable with regard to depth
C. Its matrix components Undergo remodeling in normal skin
D. Remodeling of matrix is not affected by external factors
E. Pathological factors do affect its organization.
5

12The elastic fibres of the dermis are characteristic by:


A. A central core of elastin
B. A peripheral part of fibronectin
C. Elaunin fibres anchor the basement membrane
D. Oxytalin is abundant in reticular dermis
E. Mature fibres are restricted to middermis
A

13The elastic tissue of the dermis is associated in :


A. Horizontal bundles
B. Transverse bundles
C. Vertical bundles
D. Continuous network
E. Irregular loose fibres
D

14One of the following proteoglycans is a component of the dermo-epidermal junction?


A. Decorin
B. Versican
C. Syndecan
D. Chondroitin sulphate
E. Perlecan
E
15One of the following is true about corpuscular skin receptors:
A. Are composed of a fibrous core and a capsule
B. They mediate pain
C. They form touch domes
D. Meissner”s corpuscule is situated in the deep dermis
E. Paccini corpuscule is located in the papillary dermis
Aa

16One of the following is true about dermal collagens:


A. Constitue 60% of skin dry weight
B. Type I account for 50%
C. Type III surrounds structures with basal lamina
D. Type III fibrils are smaller in diameter than Type I
E. Fibrils are visible by light microscopy
D

17One of the following is mainly composed of collagen VII:


A. Hemidesmosomes
B. Anchoring filaments
C. Lamina densa
D. Anchoring fibrils
E. Lamina lucida
D

18The most important proteoglycan in the dermis:


A. Versican
B. Perlecan
C. Biglycan
D. Keratocan
E. Syndecan
A

19One of the following is not correct about nail plate :


A. Keratin account for 80% of dry weight of nail plate
B. Contain hard and soft Keratin
C. Less permeable to water than is the epidermis
D. It Contain lipids (primarily cholesterol) and minerals (Fe, Zn, Ca)
E. Calcium does not influence the hardness of the nail
B

20Which of the following is false, about catagen:


A. It divided into 6 substages
B. Catagen last about 2 w in human regardless of the site and follicule type
C. Characterized by a nearly 50% reduction in the volume of the demal papilla
D. Melanocytes in the hair bulb also begin to down-regulate Melanogene
E. There is cessation of mitotic activity within the matrix cells of the bulb
A

21Melanocyte connections with neighboring keratinocytes is via:


A. Cadherins
B. Desmosomes
C. Hemidesmosomes
D. Tight junction
E. No connection
A 👍
Ch.65 melanocytes biology
P.1077

22When do the majority of neonatal herpes simplex virus infections take place?
A. Perinatally
B. Gestational (in-utero)
C. Postnatally
D. During breast-feeding
E. Neonatal herpes simplex virus infections occur with equal incidence at B+C
A

23Intraepidermal blister at the granual layer is not seen in:


A. Friction blister
B. Pemphigus foliates
C. Subcorneal pustular dermatosis
D. Herpes Virus infection
E. SSSS
D

24One of the following statement regarding bullous Impetigo is true?


A. Impetigo is transmitted via fomites
B. Impetigo most commonly occurs in childs> 6 y of age
C. Peak incidence is in winter
D. Non-contact sports like running, increase the risk of transmition
E. Streptococcus pyogenes is more often the implicated bacterial species in Impetigo
A

25Which of the following is true regarding Anthrax?


A. Is likely caused by a gram negative spore forming rod.
B. The causative agent produces two types of exotoxins
C. Azithromycin is a first line treatment.
D. There are two clinical forms of this disease: inhalational a cutaneous.
E.None of the above
B

26One of the following laboratory abnormalities would best aid diagnosis of Parvovirus B - 19:
A. Elevated hemoglobine
B. Elevated reticulocyte count
C. Decreased Leukocyte count
D. Elevated liver transaminases
E. Seropositive ANA
B
27One of the following dermatophytes is not ectotrix type:
A. M. ferruginum
B. M. distortum
C. T. gourvilli
D. M.gypseum
E. M.audouini
C

28Generalized pigmentation occurs in :


A. Xeroderma Pigmentosum
B. Tattooing
C. Iron deficiency
D. Addison’s dis
E. Cronn’s synd
D

29HEPATITIS B:
A. Has an incubation period of 15 – 45 days
B. Is spread by ingestion of contaminated seafood
C. Most patients recover completely
D. 50% become healthy carriers
E. Is a SSRNA virus.
D

30One of the following chemical mediators play a role in stimulation of fibroblast proliferation?
G. Kinins
H. Serotonin
I. Histamine
J. Prostaglandins
K. Complement
3

31Regarding viral vectors for skin gene therapy, One of the following viruses has transient (no
integration) duration of expression:
A. Retrovirus
B. Poxvirus
C. Adenovirus
D. HPV
E. Parvovirus
C

32The skin is THICKER in palms and soles because of:


A. A thicker stratum cornum
B. The presence of special sense organs
C. Abundance of sweat glands
D. Presence of an additional cellular layer
E. Frequent friction with external factors
D
33Body louse is a vector for:
A. Endemic typhus
B. Epidemic typhus
C. Yellow fever
D. HIV
E. All of the above
B

34One of the following is not a feature of epidrmophyton conidia:


A. They are smooth walled
B. Microconidia are numerous and diagnostic
C. It is club shaped
D. May be solitary or clustred
E. The cell wall thickness is intermediate
B

35EBV :
A. Approximately 75% of young adults with primary infection develop infectious mononucleousis
B. EBV is commonly associated with B-cell lymphoma
C. The virus enters cell by specifically binding to cell surface complement CR4
D. The humoral immune response, although it generates Ab to a variety of EBV Ag , appear to
be protective
E. Splenomegaly is found in >50% of patients and can be accompanied by hepatomegaly.
E

36Which of the following drugs is the safest option for pregnant female with acne?
A. Dapsone gel
B. Sulfacetamide wash
C. Topical tretinoin
D. Erythromycin gel
E. Adapalene cream
D

37which of the medication below prompts the cyclosporine A dose adjustment:


A. Ciprofloxacin
B. Ketoconazole
C. Carbamazepine
D. Gemfibrozil
E. Furosemide
C

38A 5 y old boy, followed for molluscum contagiosum, presents with an accidental 1 degree
burn. His only medication is cimetidine. He received a prescription for silver sulfadiazine cream.
One of the following is true?
A. The use of silver sulfadiazine cream is contraindicated due to the child’s age.
B. The use of silver sulfadiazine cream is contraindicated in patients with molluscum
contagiosum.
C. when silver sulfadiazine cream is used in conjunction with cimetidine, there is increased risk
of leukopenia.
D. silver sulfadiazine should only be used for 3d degree burns.
E. None of the above are true.
C

39One of the following Antifungal drug blocks DNA synthesis:


A. Polyenes
B. Imidazoles
C. Flucytosine
D. Griseofulvin
E. Allylamines
C

40One of the following is a Third generation antihistamine:


A. Hydroxyzine
B. Loratidine
C. Levocertizine
D. Triproline
E. Ciproheptadine
C

41One of the following is not a sedating antihistamine:


A. Promethazine
B. Misolastine
C. Cyproheptadine
D. Hydroxyzine
E. Clemastil
B

42What anesthetic would you give to a patient with an amide allergy for a punch biopsy?

A. Lidocaine.
B. Benzocaine.
C. Prilocaine.
D. Procaine
E. Mepovacaine.
D

43Histamine exerts its effect during inflammation by:


A. Vasoconstriction of post Capillary sphincters
B. Constriction of Large Arteries.
C. Acting on H2 Receptors on Mast Cells.
D. Causing Venular Endothelial Contraction.
E. Its Direct Effect on Macrophages.
A

44Dopamine:
A. Is secreted from the anterior pituitary gland.
B. Stimulates prolactin secretion.
C. Is the precursor for adrenaline and nor adrenaline.
D. Transported to the pituitary gland via axons
E. Can be released by parasympathetic nerves.
C

45Kallikrein:
A. Directly converts C5 to C5a
B. Negatively inhibits activation of Hageman factor
C. Has high molecular weight kininogen as a precursor
D. Is inactivated by kininase
E. All of the above
A

46Fat embolism:
A. Complicates fatty liver
B. Occurs within 6 h following fracture
C. Dg is done by finding fat globules in the urine
D. Occurs in decompression sichness
E. Causes thrombocytosis
B

47Phenolic glycolipid-1 is important in identification of:


A. Mycobacterium Kanasasi.
B. Dermatophytes.
C. Moulds.
D. Mycobacterium Leprae
E. Pox virus.
D

48Which of the following is not true:


A. Melanocytes are derived from neural crest.
B. The different stages of Melanosomes correlate with the degree of Melanization.
C. Melanosomes that are involved in brown Melanin synthesis are spherical
D. Keratinocytes produce GF which are mitogenic to Melanocytes
E. Proliferation of Melanocytes rely on vit. D synthesis within the Epidermis.
C

49The hypodermis is condidered amongst fonctions to be an endocrine organ. The hormonal it


release is:
A. Amylase
B. Lipase
C. Glucocidase
D. Fructocidase
E. Leptin
E

50Regarding features of premature skin, One of the following is not true:


A. Skin thickness : o.9 mm
B. Epidermal thickness: 20-25microns
C. Stratum cornum thickness: 9-10 microns
D. Size of collagen fiber bundles are small
E. Few mature melanosomes
C

51Which one of the following is not immunologic etiology of urticarial lesions :


A. Direct mast cell-releasing agents
B. autoimmune
C. IgE - dependent
D. vasculitis
E. Complement and kinin-depnendent
A

52Which one of the following melanocortin receptors is majorly distributed in cortex?


A. MC1R
B. MC2R
C. MC3R
D. MC4R
E. MC5R
B

53Complete replacement of toenail requires:


A. 3 months
B. 6 months
C. 12 months
D. 15 months
E. 18 months
E

54One of the following is not true about CD1a:


A. It is the most useful marker for detecting human Langerhans cells.
B. It is exclusively expressed on Langerhans cells in inflamed tissue only.
C. It does not apply to HLA-DR
D. It does not exist in the murine system.
E. It is exclusively expressed on Langerhans cells in normal and inflamed tissue.
B

55Which of the following is involved in stimulating adaptive immunity:


A. IL-4
B. IL-2
C. IL-23
D. IL-12
E. IFN-beta
B
56
The cytokine which is reduced in psoriasis is :
IFN-α
IL-10
IL-2
IL-12
IL-23
2

57One of the following is not true about IgG:


A. It is the most abundant Ig.
B. It is the major Ig of the innate immune
C. IgG 1 and IgG3 are potent activators of the classic pathway
D. IgG2 is less effective
E. Most of the autoimmune dermatosis which autoantibodies are mediated by IgG, most often.
B

58Regarding Th3 cells, One of the following is not true:


G. Th3 cells are a poorly characterized type of CD4+ Th cell.
H. Primerily secretes TNF-beta
I. Provides help for IgA production
J. Has suppressive properties against Th1and Th2
K. Th3 cells are tri
2

59Activation of C5a results in:


A. Opsonization
B. Increased inflammation
C. Endotoxin shock
D. Activation of interferon
E. Activation of NK cells
B

60There are 5 types of human Ig antibodies, IgA, IgD, IgE, IgG, IgM. Each of them has a typical
physical conformation. What type of physical conformation does the IgM usually have?
A. Monomeric
B. Dimeric
C. Trimeric
D. Pentameric
E. Hexameric
D
61
The classical and alternative pathway meet at complement component:
A. c3
B. c4
C. c4b
D. c5
E. factor d
A

62A complement component which is strongly chemotactic for neutrophils is:


A. C3
B. C3b
C. C5a
D. C5b
E. C9
C

63When performing a salt – split DIF of a skin biopsy from a patient with BP, WHAT IS the
expected result?
A. Granular IgA in the dermal papillae
B. Intercellular IgG antibodies located in the epidermis
C. Linear IgG antibodies located on the epidermal side of the BM
D. Linear IgA antibodies located on the BM
E. Linear IgG antibodies located on the dermal side of the BM
C

64Interferon beta is produced by:


A. Fibroblast
B. Macrophage
C. Activated T cell
D. B – cell
E. All of the above
A

65Which of the following is not true about natural killer cells:

A. They compromise 2-15% of circulating lymphocytes.


B. They have no TCR.
C. They have surface Immunoglobulin receptor.
D. They have CD4 and CD8 receptors.
E. They can be distinguished by CD16.
D

66One of the followings is not a feature of leukocyte adhesion with transmigration:


A. Migration of leukocytes as a result of stasis of blood flow in the microvasculature.
B. Interaction of integrins with immunoglobulin found on endothelial cells.
C. The selectins found only on endothelial cells
D. Icam-1 and vcam-1 adhesive molecules on the endothelial cells.
E. Neutrophils in the first 6-24 hours of acute inflammation.
C

67One of the following induces the release of inflammatory mediators from mast cells:
A. C2a
B. C3b
C. C4a
D. C5b
E. C2b
C

68Interferons production is usually induced by which type of Toll-like receptors (TLRs):


A. TLR3
B. TLR5
C. TLR7
D. TLR8
E. TLR9
C👍

69|The greatest density of mast cells is found in the:


A. Stratum spinosum
B. Stratum basale
C. Reticular dermis
D. Subcutaneous fat
E. Papillary dermis
E

70-Tinea manuum :
A. Dermatophyte infections on the dorsal aspect of the hand
B. Dermatophyte infection of the palm and interdigital spaces have a clinical presentation similar
to Tinea corporis
C. The reason for the two different clinical pictures is thought to be related to the lack of
sebaceous glands on the palms
D. Interdigital type Tinea pedis is often present in patients with Tinea manuum
E. Tinea manuum is usually non-inflammatory and often bilateral
C

‫ أيام ماموجوده‬٨ ‫نقاط أول‬

9/2018

4”18
Fibrillin is:
A. A macroprotein.
B. Distributed Closely Parallel to Collagen Fibres.
C. Assists in Destruction of Elastin.
D. Plays an Adhesive Role between Matrix Compnents.
E. Abundant in Reticular Layer.
D

10l18
One of the following is a fibril-forming collagen?
A. Col IX
B. Col VIII
C. Col V
D. Col VI
E. Col VII
C

9/18
Which of the following statements about the direct immunofluorescence pattern in lichen planus
is correct?
1. The DIF is negative in the vast majority of cases
2. Deposition of IgG is within cytoid bodies in the superficial dermis
3. The DEJ deposition is granular
4. Deposition of fibrinogen is within cytoid bodies in the deep dermis
5. There is prominent deposition of IgM within the spinous layer of the epidermis.

124

Makki 17
With Masson,s trichrome , the keratin stains:
• Yellow
• Black
• Green
• Pink
• Red
5👍 👍

One of the following LACKS desmosomal contacts with surrounding cells:


• Keratinocytes of stratum spinosum
• Keratinocytes of stratum granulosum
• Langerhans cells
• Merkel cells
• Keratinocytes of stratum basale
3

Histamine in not present in the following cells:


A. Mast cell
B. Platelets
C. Basophil
D. Free nerve ending
E. Macrophage
E

Factor 8 is positive in :
• Angiosarcoma
• Leiomyosarcoma
• Melanoma
• Atypical fibroxanthoma
• SCC
1

BCG is selective contraindication in the following biologic agent used for the treatment of
psoriasis :

A. Ustekinumab
B. Alefacept
C. Etanercept
D. Infliximab
E. Adalimumab
A

In varicella infection, Which of the following is not true:


A. After viremia the virus persists in sensory ganglion cells
B. Centripetal
C. Droplets are the usual way of transmission
D. Vesicle fluid is usually sterile and does not contain the virus
E. Tzank smear cannot differentiate varicella from HS lesions
D

A pregnant woman has varicella 2 days before delivery, concerning the effects on the neonate ,
which of the following recommendation is most appropriate ???
K. No further recommendation at this time
L. The mother should be started on IV acyclovir as soon as possible
M. The neoborn should receive varicella Immunoglobolines as soon as possible
N. The neoborn should receive the varicella vaccin as soon as possible
O. The neoborn should receive IV acyclovir as soon as possible
C

Which of the following is the greatest risk factor for SCC?


F. History of heart transplant
G. History of liver transplant
H. History of systemic lupus erythematosis.
I. History of diabetes mellitus.
J. History of Hypertension.
1

The incubation period of varicella is:

A. 3 days.
B. 7 days.
C. 14 days
D. 21 days.
E. 50 days.
C

Which virus is single stranded RNA :


a. Measles
b. HSV
c. Adenovirus
d. Parvovirus
e. HPV
A
One of the following is not live attenuated vaccine:

A. BCG
B. Rubella
C. Poliomyelitis
D. Yellow fever
E. Pertussis
E

The commonest clinical feature of infant with rubella is:


a. Cataract
b. Heart defects
c. Deafness
d. Purpura
e. Mental retardation
C

Which disease is associated with epidermolysis bullosa acquisita?


A. Hyperthyroidism
B. Tuberculosis
C. Bronchial carcinoma
D. Diabetus Mellitus
E. Amyloidosis
D

Necrobiosis is not seen in Which of the following:

A. Granuloma Annulare
B. Necrobiosis Lipoidica
C. Rheumatoid nodule
D. Actinic Granuloma
E. Subcutaneous Granuloma Annulare
D

Failure of phagolysosome formation in neutrophils is well documented in :


a. Skin fragility syndrome
b. POEMS syndrome
c. COSTELLO syndrome
d. Leopard syndrome
e. Chediak- higashi syndrome
E

Which of the following is implicated in the pathogenesis of keratinocytes formation?


A. IL-1
B. IL-2
C. IL-3
D. IFN-gamma
E. TGF-beta
5

One of the following is not correct about fibronectin:


A. It is a glycoprotein
B. Found at cell surface and in the body fluids and connective tissue
C. It has no role in cell replication
D. It acts as anchor for cells
E. It appears to be involved in adhesion of cells
C

Which of the following is not high – potency topical steroids:


A. Betamethasone dipropionate ointment 0.05 mg
B. Flucinomide gel 0.05%
C. Dexamethasone spray 0.2%
D. Flucinolone acetonide cream 0.2%
E. Clobetasole ointment
C
The smallest genetic unit of genetic material which produces a phenotypic effect upon mutation
is:
• Allele
• Nucleic acid
• Recon
• Gene
• Muton
5

Elastin is not produced by one of the following:


a. Fibroblasts
b. Smooth muscle cells
c. Macrophages
d. Endothelial cells
e. Condroblasts
C
A skin biobsy from a 35 year old patient shows suprabasal acantholytic cleft and negative
immunofluorescence. Which of following clinical features is most likely present in the patient ?
a. Tense bullae in the abdomen
b. Crusted erosions in axillae
c. Greasy papular lesions in chest
d. Non blanchable erythematous macules
e. Dermatitis reaction in the face due to aftershave lotion
B

One of the following receptor molecules is not expressed by APC:


A. CD8
B. B 7-2
C. CD 58
D. CD 265
E. CD 30
A

Which of the following is not advantage of foam occlusive dressing:


A. Absorbant.
B. Creates bacterial barrier
C. Permeable to water vapor.
D. Conforms to wound shape.
E. Comfortable.
B
Which of the following is not true about clofazimine:

A. It is a phenazine dye.
B. Its half life is 6-12 days.
C. It is mainly absorbed from jejunum.
D. It may cause red - brown discoloration of skin.
E. It can be used in pregnancy.
B

Pitting edema
A. MAY not be clinically detectable until the extra-cellular fluid volume has increased by 1% or
more
B. Not developed in a paralyzed patient
C. In cardiac failure is mainly due the rise in venous pressure
D. Develops in all normal subjects from time to time
E. Is a characteristic feature of myxedema.
C
Prostaglandins are derived from
A. Lysosomes
B. Golgi apparatus
C. Mitochondria
D. Phospholipids of cell membrane
E. As a constituent of the cytoplasm
D
Higher risk for toxicity from glucocorticoid therapy does not occur in Which of the following ?
A. Female patients
B. Patients with rheumatoid arthritis
C. Patients with dermatomysitis
D. Patients with hyperalbuminemia
E. Patients who smoke
D

Stem cell can be identified by high expression of:


A. Langerin.
B. Β1-integrins
C. Desmoglein 1.
D. Factor XIIIa.
E. Keratin 14.
B

Dermcidin source is :
a. Keratinocytes
b. Fibroblast
c. Langerhans cells
d. Sweet glands
e. Sebaceous glands
D
The cellular source of antibacterial peptide Dermcidin is:
K. Keratinocytes .
L. Airway epithelia
M. Granulocytes
N. Intestinal tract
O. Sweat glands
5
The major function of the Golgi apparatus is :
A. Endocytosis.
B. Secretion.
C. Diffusion of small molecules.
D. Protein synthesis
E. Energy generation.
B

The suffix ximab in the nomenclature of biological agents indicates to What of the following ?
A. Blocker of interaction between interleukin- 1 and its receptor
B. Chimeric monoclonal antibody
C. Humanized monoclonal antibody
D. Human monoclonal antibody
E. Receptor protein Blocker
B

Nocireceptors repond to:


A. Temperature changes
B. Harmful stimuli
C. Chemical solutions
D. Mechanical forces
E. Light
B

Which of the following is used to treat pubic lice infestation of eyelashes:


A. Septrin
B. Ivermectin
C. Petrolatum
D. Pyrethin
E. Malathion
C

Which of the following has the highest risk for gastrointestinal bleeding:
A. Aspirin
B. Ibuprofen
C. Diclofenac
D. Piroxicam
E. Indomethacin
E
Which of the following is NOT true:
A. Gene consists of DNA packed in chromosome
B. Skin fibropblast can be cultured in the laboratory
C. Chromosomes are present in all nucleated cells
D. In AD, usually only one generation is affected
E. In AR, both parents are carriers
D

For a patient of chancre and allergy to penicillin, What is the alternative therapy ?
A. Metronidazole 2 g single dose
B. Terbinafine 250 mg x 2 daily for 1 month
C. Tetracycline 500 mg x 4 daily for 1 week
D. Erythromycin 500 mg x 4 daily for 2 weeks
E. Clindamycin 150 mg x 4 daily for 3 days
D

Itching migrating in distribution and time and without lesions is most likely secondary to ?
A. Aging
B. Hepatic disease
C. Renal disease
D. Parasitophobia
E. Internal malignancy
D

The keratinocyte is protected from attack by complement by:


A. CD 35
B. CD 21
C. CD 88
D. CD 55
E. CD 59
E
The structural base of the UV protection function of strateum corneum is :
A. Corneodesmosome
B. Lamellar bilayers
C. Corneocyte and extracellular matrix
D. Corneocyte Cytosol
E. Cornified Envelope
C

One of the following is not correct regarding skin homing of memory T cells:
A. It depends on interaction of CCL 17 with CCR4
B. It depends on interaction of CCL 22 with CCR4
C. It depends on interaction of CCL 27 with CCR8
D. CCL 17 is synthesized by activated keratinocyte
E. CCL22 is produced by macrophage
C

One of the following is a UVA sun blocker:


A. Padimate
B. Meradimate
C. Octiroxate
D. Trolamine salicylate
E. Ensulizole
B

One of the following is an uncommon cause of drug-induced acne:


A. Danazol
B. Lithium
C. Phenytoin
D. Azathioprine
E. Bromides
D

One of the following is not correct about the markers of NK cells:


A. CD 56+
B. CD 16+
C. CD 154+
D. CD 94+
E. CD 161+
C

One of the following is a less cause of erythroderma in adults:


A. PSO
B. Atopic der
C. PRP
D. Drug reaction
E. Idiopathic
C

One of the following is polyenes antifungal drugs:


A. Ketoconazole
B. Itraconazole
C. Terbinafine
D. Amphotericine B
E. Tolnaftate
D

One of the following is not correct about chronic pain:


A. Peripheral sensation of C Nerve fibers
B. Central sensation are allokness, punctuate hyperknesis
C. Neuromediator is neurotropin 4
D. Chemical mediator is bradykinin
E. CNS areas activated is anterior cingulate cortex
B
.
One of the following plants cause urticarial dermatitis:
A. Dumb cane
B. Garlic
C. Celery
D. Stinging nettle
E. Chrysathemums
D

The corneal layer role in protection against UV radiation is fulfilled through:

A. Reflection of light from the surface


B. Light scattering
C. Reflection from the surface and filtring through acid metabolites present in the corneal layer
D. Absorption of light through keratin
E. Refraction
C

Under dermoscopy, a sebaceous keratosis has Which of the following feature?


A. Hairpin vessels
B. Milia – like cysts
C. Glomerular vessels
D. A and B
E. A, B and C
D

Prolyl hydroxylation reaction is the initial step in the biosynthesis in WHICH of the following :
a. Collagen
b. Elastin
c. Keratin sulphate
d. Hyaloronic acid
e. Keratin
A

One of the following mucocutaneous features occur more often in girls with Turner syndrome
than in normal girls:

A. Halo nevi
B. Scrotal tongue
C. Vitiligo
D. Psoriasis
E. Macrocheilitis
2

Hyperemia of acute inflammation is due to:


A. Interleukin- 1
B. Fibrinogen
C. Indreased hydrostatic pressure
D. C5a
E. Emigration of leucocyte
A

Select the incorrect statement about the remoudeling of keratinocytes into corneal layer:
A. Keratinocytes become less permeable to Ca
B. Profilagrin is processed into filagrin
C. Keratohyaline granules are broken down
D. Glutaminazes are activated
E. Filagrin is degraded into amino acids
A

Regarding development of melanocytes, Which of the following is not true:


A. They are arising from neural crest ✔
B. They are present in the epidermis by the end of 1st trimester✖ 50 day
C. Melanin production does not begin until 3-4 months of EGA✔
D. Melanosome transfer to keratinocyte is not seen until 5 months of EGA✔
E. At birth, neonates are not fully pigmented✔

Regarding development of dermis, the demarcation between the dermis and underlying skeletal
condensations becomes distinct at:
A. 7 weeks of estimated gestational age
B. 9 weeks of estimated gestational age
C. 11 weeks of estimated gestational age
D. 13 weeks of estimated gestational age
E. 15 weeks of estimated gestational age
B

One of the following is not part of a pillosebaceous unit:


• Sebaceous gland
• Hair follicule
• Arrector pilli
• Apocrine sweat gland
• Eccrine sweat gland
5

Which of the following is the longest acting systemic corticosteroid:


A. Cortisone
B. Prednisone
C. Methyl prednisone
D. Dexamethasone
E. Triamcinolone
D

Each of the following is true of the occurrence of malignancy in patients with Pyoderma
Gangrenosum except:
A. Occurs in 7% of patient
B. Leukemia is the most common reported malignancy
C. The prognosis of Pyoderma Gangrenosum is good
D. IgA myeloma has been associated with Pyoderma Gangrenosum
E. Bullous Pyoderma Gangrenosum is the most commonly associated
C

Regarding breast cancer in males, mark the WRONG statement:


A. The incidence approaches that in women in cases of klinefelter’s syndrome.
B. It is usually related to wearing braces (suspenders)
C. It can be associated with previous exposure to radiation.
D. It is likely to produce nipple inversion.
E. As in women the essential features is a breast mass
B

One of the following has an inhibitory effect on hair growth:


A. IL- 1
B. INSULIN- like growth factor
C. Hepatocyte growth factor
D. Keratinocyte growth factor
E. Vascular endothelial growth factor
A

.Which of the following mycobacteria is rapid grower in culture?


A. Mycobacterium leprae
B. Mycobacterium kansasi
C. Mycobacterium smegmatis
D. Mycobacterium scofulaceum
E. Mycobacterium tuberculosis
C

C-T CC-TT DNA mutations are characteristic for:


F. Oxidative damage
G. UVA absorption
H. P 53
I. XP
J. UV photodamage
5

One of the following is not a physiologic role of IL - 10:


K. Limiting the inflammatory response
L. Supporting humoral immunity
M. Preventing overwhelming immune response
N. Increasing macrophage Ag presenting
O. Upregulating Th2 immune pathway
D

0
One of the following IS NOT correct about Langerhans cell:
A. Murine Langerhans cells are derived from radio-resistant hematopoietic precursor cells.
B. The Formation of Langerhans cells depends on TGF-B2
C. After UVB irradiation the cells appear to repopulate from the hair follicule.
D. Langerin is not an exclusive marker for epidermal Langerhans cells.
E. Formation of Langerhans cells depends on MC-SF receptor ligands.
B

Which disease is associated with epidermolysis bullosa acquisita?


A. Hyperthyroidism
B. Tuberculosis
C. Bronchial carcinoma
D. Diabetus Mellitus
E. Amyloidosis
D

The following mucosal manifestation are known to occur in sarcoidosis except?


A. Xerostomia
B. Salivary gland swelling
C. Gingival swelling
D. Leukoplakia
E. Swelling of the lachrymal and parotid glands and facial palsy
D
At how many weeks does the fetal basement membrane begin to develop :
A. 3 weeks
B. 7 weeks
C. 9 weeks
D. 12 weeks
E. 16 weeks
B

8
Sebaceous dermatitis:
A. Common disorder occurring in 1-3% of general population
B. Marginal blepharitis is a complication of Seb dermatitis
C. A unilateral injury to the innervation of the face or a stroke lead to decrease incidence of
Sebaceous dermatitis of the affected side
D. Patient with Sebaceous dermatitis show downregulation of gamma interferone
E. Patient with AIDS has decreased incidence of Sebaceous dermatitis
B

8
The major difference between darker skin and lighter skin is:?

A. The relative concentration of melanocytes per unit area of skin surface.


B. The relative activity of cutaneousmelanocytes in creation of melanosomes
C. The relative action of cutaneousmelanocytes in transferring melanosomes to keratinocytes
D. The fate of the melanosomes once they are within the keratinocyte.
E. Number of melanosomes is higher in lighter skin.
2

One of the following stains blue by Giemsa:


A. Amyloid
B. Mast cell granules
C. Nuclei cell granules
D. Acid – fast organisma
E. Sulfared mucopolysaccharides
C

Which of the following is in photo aged skin:


A. Collagen type I.
B. Collagen type III.
C. Collagen type IV
D. Collagen type VII.
E. Febrillin.
C

10 17
After the skin, the 2d organ mostly involved by mast cell disease is :
A. Bone
B. Gastro-intestinal tract
C. Brain
D. Liver
E. Kidney
A

8
Biopsy of suspected DLE would show inflammation around which portion of the hair follicle.:

A. Hair bulb
B. Infundibulum
C. Isthmus
D. Bulge
E. Matrix
C

Sister Joseph’s nodule of the umbilicus represents a metastasis from primary cancer in the
following organs. EXCEPT:

A. The stomach.
B. The ovary.
C. The intestines.
D. Pancreas.
E. Kidney
5

In SCC of lower lip, mark the wrong ?

A. It is rare in negroes
B. Very rarely metastatises
C. Undue exposure to the sun in the major cause
D. Clay – pipe smoking can prove it
E. Poor dentition and syphilis can be important predisposing causes
2

In chronic renal failure by renal dialysis What is the commonest complication?

A. Gynecomastia
B. Calciphylaxis
C. Sensorimotor neuropathy
D. Perforating skin disorders
E. Cutaneous necrosis
2👍 👍

Mark the WRONG statement about pityriasis aminatacea:

A. It can be a manifestation of sebrrhoeic dermatitis of the scalp.


B. It can be due to psoriasis.
C. It can be a manifestation of eczema.
D. It can be caused by fungal infection.
E. The average age incidence is 5 -40 years
5

Basic organogenesis is complete by the end of:

A. 12 weeks EGA.
B. 8 weeks EGA
C. 6 weeks EGA.
D. 14 weeks EGA.
E. 16 weeks EGA.
2

one of the following is not correct about NK cells:

A. the major task of NK cells is to eliminate infected or malignant cells


B. they expresse Fc receptors that bind IgM
C. They recognized involves killer- activating and killer-inhibitory receptors
D. NK cells Carry receptors that recognize MHC class 1 molecules
E. Tumor cells and viruses often down regulate MHC class 1 molecules
2

One of the following is NOT correct about melanocytes:


A. In premature there is high number of cells.
B. In newborn similar number of cells to young adults.
C. In adult their number decrease with age.
D. In premature high number of mature malenosomesx
E. In adult, melanin production dependent on skintype and body area.
4

Among the following lesions, which of the most likely to turn malignant?

A. Nevus comedonicus.
B. Verrucous epidermal haevus.
C. Nevus sebaceous
D. Sebaceous adenoma.
E. Steatocytoma multiplex
3

Toll-like receptor (TLR) 6 is:

A. Diacylated lipoprotein
B. Flagellin.
C. Lipopolysaccharide.
D. Ds RNA.
E. Triacylated lipoprotein.
A

One of the following occupations may not be infected by anthrax:

A. Farmer
B. Wool sorter
C. Veterinarian
D. Dock worker
E. Butcher
3

The mediator that has no activity on pain is:

A. Histamine
B. Prostaglandin E
C. Substance P
D. Tryptase
E. Opiod peptide
5

Which of the following is not the most frequently subjective side effects of methotrexate ?
A. Nausea
B. Abdominal pain
C. Headache
D. Loss of libido
E. Fatigue
4

Which of the following is not atypical form of PR :

A. Psoriatic like
B. Urticarial
C. Vesicular
D. Pustular
E. Purpuric
1

Dendritic epidermal cells express which of the followings markers:

A. CD 5
B. CD 4
C. CD 45
D. CD 8
E. MHC class II molecules
5

The major difference between darker skin and lighter skin is:

A. The relative concentration of melanocytes per unit area of skin surface.


B. The relative activity of cutaneousmelanocytes in creation of melanosomes
C. The relative action of cutaneousmelanocytes in transferring melanosomes to keratinocytes
D. The fate of the melanosomes once they are within the keratinocyte.
E. Number of melanosomes is higher in lighter skin.
2!3

Keratohyalin granules:

A. Stain intensely with eosin.


B. Contain glycosphingolipids, phospholipids and cermaides that are released into the
extracellular spaces between the stratum granulosum and stratum corneum.
C. Mark the last living layer of the epidermis.
D. Contain unmodified profilaggrin that is easily visible in the stratum corneum.
E. Have s100 as a surface marker.
3

One of the following plants does not cause CD:

A. Garlic
B. Peruvian lily
C. Poison ivy
D. Poison oak
E. Celery
1
One of the followings is not a biologic effect of retinoids :

A. Inhibition of apoptosis
B. Inhibition of keratinization
C. Inhibition of tumor promotion and malignant cell growth
D. Differentiation and maintenance of epithelial tissue
E. Regular expression and activation of TLRs
1

Which of the following is a vasoconstrictor in the absence of epinephrine?

A. Lidocaine.
B. Procaine.
C. Cocaine
D. Bupivacaine.
E. Prilocaine.
3
In electron microscopy, the cell that demonstrates cytoplasmic projections and secretory
granules is:
-[]
- [ ] A. Langerhans cell.
- [ ] B. Keratinocyte.
- [ ] C. Mast cell
- [ ] D. Melanocyte.
- [ ] E. Macrophage.
-[]3

High-frequency electrosurgery method DOES NOT include:

A. Electrolysis
B. Electrodesiccation.
C. Electrofulguration.
D. Electrosection.
E. Electrocoagulation.
1

Regarding cryotherapy which of the following statement is NOT TRUE:

A. The freezing time is shorter for benign lesions than malignant.


B. 10 seconds with open spray technique will cure 80% of actinic keratosis.
C. The freeze time is the same for both cryoprob or spray technique
D. 2-3 nm lateral spread is sufficient for successful treatment of warts.
E. Keloids can be treated with open spray technique alone or in combination with intralesional
steroids.
3

The smallest genetic unit of genetic material which produces a phenotypic effect upon mutation
is:
• Allele
• Nucleic acid
• Recon
• Gene
• Muton
5

With Masson,s trichrome , the keratin stains:


• Yellow
• Black
• Green
• Pink
• Red
5

One of the following is not correct about nail plate and its layers:
• It has thicker intermediate lamina
• It has thin dorsal lamina
• The pink color is due to the longitudinally oriented subungual capillaries
• Ca+ significantly contribute to the hardness of the nail
• Contain significant amount of hard keratin
4

MOUSY(MUSTY ) odor of urine is characteristic of:


• Hunter disease
• Alkaptonuria
• Maple syrup urine disease
• Hurler disease
• Phenylketonuria
5

All layers of the keratinized epidermis identifiable at:


• 24 week gestational age
• 20 week gestational age
• 16 week gestational age
• 12 week gestational age
1

Natural killer cells are stained with :


• CD 1a
• CD11
• CD30
• CD56
• CD68
4

The pigment deposits in ochronosis are accentuated with


• Cresyl violet and Silver nitrate
• Bodian
• Silver nitrate
• Methyl – green pyronin
• Cresyl violet
5

“One of the following is not part of a pillosebaceous unit:


• Sebaceous gland
• Hair follicule
• Arrector pilli
• Apocrine sweat gland
• Eccrine sweat gland
5

Silver preferentially deposits in :


• Eccrine glands and apocrine glands
• Fat
• Hair follicules
• Apocrine glands
• Eccrine glands
1

Which one is not true with respect to IgG antibodies


• They mediate allergic reaction
• They are glycoproteins
• They predominate in the primary response to antigen
• They occur on the surface of lymphocytes
• They fix complement
3

Factor 8 is positive in :
• Angiosarcoma
• Leiomyosarcoma
• Melanoma
• Atypical fibroxanthoma
• SCC
1

Two antibodies that have the same antigenetic recognition sequence are called:
• isotypes
• auto
• idio
• haptene
• allo
3
Idio

The most abundant sensory receptor of the skin is :


• Meissner,s corpuscle
• Krause,s end bulbs
• Pacinian corpuscle
• Ruffini,s corpuscle
• Free nerve endings
5

Langerhans cells does not express :


• S- 100
• Birbeck granules
• CD 1a
• HLA -DR
• Chromagranin
5
Which tumor is S- 100 negative immunostaining :
• Melanoma
• Schwannoma
• Adenoid cystic carcinoma
• Cellular neurothekoma
• Granular cell tumor
4

Which one is not an advers effect of topical corticosteroids?


• Pseudoscarring
• Hypopigmentation
• Connective tissue reduction
• Retention hyperkeratosis
• Impaired angiogenesis
4

The bacterial cell wall is not the site of action of :


• Carbapenems
• Vancomycin
• Sulfonamide
• Monobactams
• Penicillin
3

Which chemical modification is linked to increase steroid potency ?


• Hydroxylation
• Carboxylation
• Acetylation
• Myristylation
• Fluorination
5

Which one is not correct regarding laser terms :


a. Pulse duration : seconds
b. Spot size : mm
c. Flounce : watt / cm2
D. Power : watts
E. Energy : joules
3

Postauricular adenopathy is commonly associated with :


a. Rubella
b. Mumps
c. Erythema infectiosum
D. Scarlet fever
E. Measles
1

Which one is a monofilament suture :


a. Polyglactin 910
b. Poliglecaprone
c. Lactomer ( polysorb )
D. Polyglycolic acid
E. Chromic surgical gut
2

One is not involved in adaptive immune response:


a. IL-2
b. IL-10
c. IL-18
D. INF-BETA
E. IL-23
4

The first site in body that shows yellowish pigmentation in carotenoderma is :


a. Face
b. Trunk
c. Palms and soles
d. Nails
e. Sclera
A

Regarding viral vectors for skin gene therapy, one of the following viruses has transient (no
integration) duration of expression:
a. Retrovirus
b. Poxvirus
c. Adenovirus
d. HPV
e. Parvovirus
3

Skin lesions with cutaneous larva migrans are typically seen on :


a. Distal lower extremities
b. Scalp
c. Face
d. Trunk
e. Perianal area
1

Which one is not effective for treatment of CMV :


a. Cidofovir
b. Acyclovir
c. Foscarnet
D. Ganciclovir
E. Valganciclovir
2

Which one is not correct about HIV- associated dermatological disease when CD4 > 500 cell/
mm3 :
a. HZ
b. Acute retroviral syndrome
c. Oral hairy leukoplakia
D. Vaginal candidosis
E. Seborreic dermatitis
A

Which one is not correct about IgG:


a. It is the most abundant Ig.
b. IgG is the major Ig of the innate immune response
c. IgG1 and IgG3 are potent activators of the classical complement pathway
D. IgG2 is less effective.
E. Most of the autoimmune dermatoses which are caused by autoantibodies are mediated by
IgG, most offten IgG4.
2

Which one is not correct about venereal species of treponema and diseases caused by them :
a. T.pallidum, venereal syphilis
b. T. Pertenue, Bejel
c. T.carateum, pinta
d. T. denticola, periodontal disease
e. T. socranski, periodontal disease
2

Which antifungal drug blocks DNA synthesis : :


a. Polyenes
b. Imidazoles
c. Flucytosine
d. Griseofulvin
e. Allylamines
3

P-Phenylenediamine is found in one of following allergens:


a. Preservatives
b. Hair allergens
c. Textile allergens
d. Adhesive
e. Components of rubber products
2

Which laser does not has crystal media :


a. Alexandrite
b. Ruby
c. Erbium- doped YAG
d. Krypton
e. Holmium doped YAG
4

Which one is not second generation antihistamine :


a. Misolastine
b. Fexofenadine
c. Desloratadine
d. Acrivastine
e. Doxepin
5

The cause of white piedra is :


a. Hortea werreckii
b. Trichosporon beigelii
c. Piedraia hortae
D. Trichophyton rubrum
E. Epidermophyton
2

which one is not ectotrix type dermatophytes:


a. M. audouini
b. M. distortum
c. M. gypseum
d. T. soudanense
e. M. ferrugineum
4

TLR 5 recognizes :
a. DsRNA
b. Diacyclated lipoprotein
c. Fillagrin
d. Peptidglycans
e. Lipopolysaccharide
3

Dermcidin source is :
a. Keratinocytes
b. Fibroblast
c. Langerhans cells
d. Sweet glands
e. Sebaceous glands
D

HMB – 45 is POSITIVE in one of the following tumors:


a. SCC
b. Atypical fibroxanthoma
c. Melanoma
d. Leiomyosarcoma
e. Angiosarcoma
3

Which one is not correct about color of staining of the folowing elements:
a. Masson trichrome stains collagen blue- green
b. Verhoeff van gieson stains elastic fibers black
c. Alcian blue PH2.5 stains acid MPS blue
d. Toluidine blue stains acid MPS green
e. Oil red O stains lipid red
4

Which one is not correct about types of collagens:


a. Types 1 is found in papillary dermis
b. Types 2 largelly restricted to cartillage
c. Types 3 is found in developing tissues
d. Types 4 is basement membrane
e. Types 8 is characterized by discontinuities in the triple helix
5

Which one is not correct about phagocytes :


A. Derived from blood-born monocytes
B. They carry receptors for mannose
C. Their major function is protection from parasitic infection
D. Activared phagocyte release G-CSF
E. It can also present processed Ag to B and T cells
3

The average density of scalp vellus and terminal hair in neonate is?

A. 250 / cm2
B. 1135/ cm2
C. 615/ cm2
D. 485/ cm2
E. 300/ cm2
2

Which of the following is adhesive allergen:

A. Colophony
B. P- Phenylenediamine
C. Diazolidinyl urea
D. Nickel sulfate
E. Balsam of Peru
A

Natural killer cells can adhere and kill target cells that are coated by ?

A. IgE
B. IgM
C. IgA
D. C5
E. IgG
5

Which of the following is not true :


A. Basophils and mast cells exhibit similar functional and morphological features
B. Basophils are found in the blood
C. mast cells are found in tissues
D. both express the low – affinity receptors for IgE ( FceR II )
E. mucosal mast cells contain only trypsin
4

Which of the following in pediculosis capitis is not true:

A. More common in girls


B. Uncommon in African – americans
C. Oral ivermectin is used to treat resistant cases
D. The parasire aon hair keratin
E. Blow – dryers allow lice to become airborn
4

The following dermatophytes are ectothrix type except :

A. M. ferrugineum
B. M. distortum
C. T. gourvilli
D. M. gypseum
E. M . audouini
3

Which of the following is not tick – borne disease?

A. Lyme dis.
B. Rocky mountain spotted fever
C. Babesiosis
D. Human monocytic ehrlichiosis
E. Yellow fever
5 mosquito borne

Which of the following normal skin flora is Gram negative:


A. Staph epidermis
B. Acintobactor spp
C. Propionibacterium acne
D. Corynebacterium minutissimum
E. Malassezia furfur
2

Which of the following mycobacteria is rapid grower in culture?

A. Mycobacterium leprae
B. Mycobacterium kansasi
C. Mycobacterium smegmatis
D. Mycobacterium scofulaceum
E. Mycobacterium tuberculosis
3

Which of the following can not be seen in acromegaly?

A. Cutis verticis gyrate


B. Hypotrichosis and hypohydrosis
C. Enlargement of hands and feet
D. Skin tag
E. Cardiovascular, metabolic and respiratory complications
2

One of the following is not essential assays in diagnosis of primary HIV infection:

A. ELISA
B. Western blot
C. HIV 1. P24 Ag testing
D. HIV DNA or RNA polymerase chain reaction
E. T- Cell subset enumeration ( PCR )
5

Which of the following stains is not used to identify Langerhans cells:

A. ATP
B. Aminopeptidase
C. OKT6
D. Argentaffin
E. Gold chloride
4

The only granuloma which develop in papillary dermis is :

A. Sarcoidosis
B. Lichen nitidus
C. Tuberculoid leprosy
D. T. B. granuloma
E. Ruptured hair follicule granuloma

Which of the following is not a feature of cytologic atypia:

A. Pleomorphism.
B. Hyperchromasia
C. Infiltative growth
D. Abnormal mitosis
E. Prominent nucleoli
3

In anthrax, Which of the following is not true:


A. It is caused by anthracis bacillus.
B. The skin lesion is associated with occupational exposure
C. It may present with bloody diarrhea
D. Inhalational anthrax is the commonest type
E. Prophylaxis with ciprofloxacin for any one at risk is recommanded
4

In varicella infection, Which of the following is not true:

A. After viremia the virus persists in sensory ganglion cells


B. Centripetal
C. Droplets are the usual way of transmission
D. Vesicle fluid is usually sterile and does not contain the virus
E. Tzank smear cannot differentiate varicella from HS lesions
4

Which of the following is inactivated toxin (toxoid ):

A. Measles
B. Yellow fever
C. Rabies
D. Typhoid
E. Diphtheria
5

The most common cause of death in treated diabetics is :


A. Infection
B. Diabetic ketoacidosis
C. Hypoglycemia coma
D. Cardiovascular disease
E. Renal failure
4

Which of the following is not true:


A. The normal sum of O2 consumption in 70 kg adult is 250ml/mn
B. Shock in Diabetic ketoacidosis is hypovolemic
C. Anaphylactic Shock usually associated with cold peripheries
D. JVP is elevated in cardiogenic Shock
E. Neurogenic Shock may be caused by major spinal cord injury
3

Which of the following interferons is fibroblasts :

A. Interferon a.
B. Interferon β.
C. Interferon Y.
D. Interferon k.
E. A and b.
B
Alfa <<— leukocytes
Beta <<— fibroblasts
Gamma <<—T/NK

Which of the following is not among the components of hemidesmosomes:

A. Plectin.
B. BP Ag-1.
C. BP Ag-2.
D. Type IV collagen.
E. Type XVII collagen.
4

Soft keratin is present in:

A. Cortex of the hair.


B. Cuticle of the hair.
C. Hair matrix.
D. Nail plate.
E. Medulla of the hair.
5

Which of the following is deep pressure receptor:

A. Ruffini’s end organ.


B. Meissner’s corpuscle.
C. Pacinian corpuscle.
D. Merkel’s disc.
E. Free nerve endings.
C

Which of the following is not ectodrm in origin:


A. Sebaeous gland
B. Nerves
C. Blood vessel
D. Nails
E. Epidermis
C

Histamine in not present in the following cells:


A. Mast cell
B. Platelets
C. Basophil
D. Free nerve ending
E. Macrophage
5

Active melanocytes At birth disappear from the following site:


A. Trunk
B. Presacral area
C. Head
D. Dorsal aspects of distal extremities
E. Neck
A

Which of the following does not cause false positive patch test:
A. Irritant vehicule
B. Insufficient amount of the test substance is applied
C. Recent dermatitis at the site
D. Adhesive tape reaction
E. Stains or discoloration of the skin
2

Regarding development of melanocytes, Which of the following is not true:


A. They are arising from neural crest
B. They are present in the epidermis by the end of 1st trimester
C. Melanin production does not begin until 3-4 months of EGA
D. Melanosome transfer to keratinocyte is not seen until 5 months of EGA
E. At birth, neonates are not fully pigmented
2

Merkel cell is detected as early as 6 – 8 weeks at the epidermis of :

A. Scalp
B. Scrotum
C. Palms
D. Trunk
E. Face
C

The immediate tanning of skin is mainly caused by:

A. UVA
B. UVB
C. UVC
D. IR
E. Visible
A

Urocanic acid is found in:

A. Stratum basale
B. Stratum spinosum
C. Dermis
D. Stratum Corneum
E. Stratum Granulosum
4

Necrobiosis is not seen in Which of the following:

A. Granuloma Annulare
B. Necrobiosis Lipoidica
C. Rheumatoid nodule
D. Actinic Granuloma
E. Subcutaneous Granuloma Annulare
4👍

Interface dermatitis not seen in which of The following

A. EM
B. Large plaque parapsoriasis
C. PR
D. Paraneoplasic pemphigus
E. Lichen sclerosus
3

The histological feature of decreased to absent granular layer is seen In which of the following?

A. X – linked ichtyosis
B. Lamellar ichtyosis
C. Congenital Ichtyosiform erythroderma
D. Ichtyosis vulg
E. Epidermlytic hyperkeratosis
4

One of the following has an inhibitory effect on hair growth:

A. IL- 1
B. INSULIN- like growth factor
C. Hepatocyte growth factor
D. Keratinocyte growth factor
E. Vascular endothelial growth factor
A
Fingernail growth is:

A. 0.1 mm / day
B. 0.2 mm / day
C. 0.3 mm / day
D. 0.4 mm / day
E. 0.5 mm / day
A
3mm/month
3/30 = 0.1 mm/day
Toenails 1mm/month
1/30 = 0.03mm/day

Which of the following is not correct about mechanism of action of azathioprine:

A. Reduced NK cell function


B. Reduction In CD8+ T cell
C. INHIBITION of lymphocyte proliferation
D. Activation of antibody responses
E. Impaired neutrophil chemotaxis
4

MAST cell granules do not contain:

A. Histamine
B. Heparin
C. Tryptase
D. TNF-alpha
E. TNF-beta
5

In the 1st year of life, one of the following disease is difficult for the immune system to
recognize:

A. Neisseria meningitis
B. Chicken pox
C. HS
D. Impetigo contagiosum
E. Bacterial folliculitis
A

Which of the following is not correct about side effects of antimalarial :

A. Irreversible keratopathy
B. Irreversible retinopathy
C. Aplastic anemia
D. Bleeching of the hair
E. Lichenoid eruptions
A
One of the following is not histologic changes in DLE:

A. Hyperkeratosis
B. Follicular plugging
C. Hydrophilic degeneration of basal cells
D. Colloid bodies
E. Elongated rete-ridges
E

Which of the following is not included in the Herpes Virus group :

A. Epstein Bar virus


B. Cytomegalo virus
C. Varicella zoster virus
D. coxackie virus
E. HHV 8
D

Which of the following is not high – potency topical steroids:

A. Betamethasone dipropionate ointment 0.05 mg


B. Flucinomide gel 0.05%
C. Dexamethasone spray 0.2%
D. Flucinolone acetonide cream 0.2%
E. Clobetasole ointment
C

The chromophore of pulsed dye laser is:

A. Oxyhemoglobine
B. Melanine
C. Carboxy hemoglobine
D. Collagen
E. DNA
A

Vaginal discharge smear is not useful to screen?

A. Gonnococcal infection
B. Trichomoniasis
C. Candidiasis
D. Syphilis
E. Bacterial vaginitis
4

Which of the following is not true regarding hair biology:

A. The hair matrix gives rise to hair and the inner root sheath
B. The outer root sheath represents a downward extension of the epidermis
C. The keratin in cortex represent hard keratin
D. The inner root sheath contain melanin
E. The three layers of inner root sheath keratinized by thricohyaline granules
D

All of the followings vascular changes during pregnancy are physiological except:

A. Spider angiomas
B. Hemorrhoids
C. Pyogenic granuloma
D. Striae
E. Gingival hyperemia
D 👍 stria CT not vascular

Which of the following is a single stranded DNA virus?


A. Hepatitis B virus.
B. Retrovirus.
C. Herpes virus.
D. Parvovirus
E. Coxackie virus.
D

All of the following are TNF- a inhibitors, except:

A. Ustekinumab.
B. Golimumab.
C. Etanercept.
D. Adalimumab.
E. Infliximab.
A

Oral papillomas associated with which type of HPV?

A. HPV 28.
B. HPV 32
C. HPV 39.
D. HPV 52, 53.
E. HPV 42, 43.
2

In which one of the following conditions Maculae caeruleae are seen?

A. Jelly fish bites.


B. Pediculosis pubis.
C. Multiple mosquito bites.
D. Sexually transmitted vulvo-vaginal candidosis.
E. Sexually transmitted anogenital warts in children.
B
Telangiectases and acneiform lesions are features of a disease that cause which one of the
following?

A. Submarine comedones.
B. Oozing.
C. Excoriation marks.
D. Flushing.
E. Liquefaction degeneration.
D

Which of the following is not preservative allergens?

A. Methyldibromoglutaronitrile
B. Phenoxyethanol
C. Methylchroroisothiazolinone
D. Methylisothiazolinone
E. DMDM hydantion
B

One of the following IS NOT characterized of tuberculoid leprosy:

A. Hypohydrotic.
B. Anesthetic.
C. Atrophic
D. Asymmetrical.
E. < 5 lesions.
C

Which of the following is major criteria for dg of Behcet “s dis?

A. Recurrent genital ulceration


B. Eye lesions
C. Recurrent oral ulceration
D. Cutaneous lesions
E. Pathergy test
C

One of the following is a common cause of erythroderma in adults:

A. Cutaneous T-cell lymphoma.


B. PRP.
C. Pemphigus foliaceus.
D. Atopic dermatitis
E. Congenital ichthyoses.
D

The majority of patients with DH have ?


A. HLA- A5
B. HLA- B51
C. HLA- CW
D. HLA- B8
E. HLA- DR4
D

Which one of the following is NOT small vessel vasculitis?

A. Henoch – schonlein purpura


B. Takayasu” s syndrome
C. Urticarial vasculitis
D. Erythema Elevatum Diutinum
E. Allergic vasculitis
B

The target protein in bullous SLE is?

A. TYPE XVII collagen


B. TYPE VII collagen
C. Integrin subunit beta 4
D. Laminin 5
E. Laminin 1
B

Bartonella hensella can cause all the following diseases except:

A. Cat – scratch disease


B. Bacillary angiomatosis
C. Carrions disease
D. Bacillary Peliosis hepatitis
E. Endocarditis
C

Which one of the following laboratory tests is not used for diagnosis of primary syphilis :

A. Dark field microscopic examination


B. VDRL
C. TPHA
D. FTA – ABS 19S-IgM
E. FTA – ABS
D

Desmoplakins are the target antigen in which of the following :

A. PV
B. Paraneoplasic pemphigus
C. BP
D. Pemphigus Erythematous
E. Pemphigus foliaceus
B

Localized itching occurs in

A. Iron deficiency
B. Polycythemia vera
C. Thyroid dis
D. Uraemia
E. Diab mell
5

Acquired hypertrichosis lanuginose is not associated with

A. Lung cancer
B. Breast carcinoma
C. PPK
D. Colon cancer
E. Brain tumor
E

Gallstones most commonly occur in?

A. Acute intermittent porphyria


B. Porphyria cutanea tarda
C. Variegate porphyria
D. Hereditary coproporphyria
E. Erythropoitic protoporphyria
5

The specific test for the diagnostic of neurosyphilis is?


A. TPHA
B. MHA-TP
C. SPHA
D. FTA-ABS
E. FTA-ABS 1gs-IgM
C

One of the following is NOT CORRECT about acute retroviral syndrome in HIV infection?

A. The earliest cut. manifestation of HIV infection may be an exanthema


B. The presence of HIV – 1 in the plasma is detected by viral RNA by PCR and / or P24
C. Development of HIV –1 antibodies is detected by ELISA or western blot test
D. The generalized morbillifom exanthema , that typically involved the palms and soles usually
lasts 4 – 5 days
E. A decline in CD 4+T cells can observed, occasionally to levels that allow the Development of
opportunist infections
D👍 👍
Face & trunk
Elastin is not produced by one of the following:
a. Fibroblasts
b. Smooth muscle cells
c. Macrophages
d. Endothelial cells
e. Condroblasts
C

Hair follicule start to appear at :


a. 10 weaks of gestation.
b. 12 weaks of gestation.
c. 14 weaks of gestation.
d. 9 weaks of gestation
e. 16 weaks weaks of gestation.
D

Which Porphyrin is responsible for the fluorescence under wood’s light lamp examination of
erythrasma?
A. Porphobilinogen
B. Protoporphyrin IX
C. Coproporphyrin III
D. Uroporphobilinogen
E. Coproporphyrinogen
C

One of the following is not correct about staining:


a. Masson trichrome stains elastic fibers
b. Alcian blue PH2.5 stains acid MPS
c. VERHEOFF Van gieson stains elastic fibers
d. Scarlet red stains lipid
e. Oil red O stains lipid
A

Under dermoscopy, a sebaceous keratosis has Which of the following feature?


A. Hairpin vessels
B. Milia – like cysts
C. Glomerular vessels
D. A and B
E. A, B and C
D

What is the 1st stage of carcinogenesis?


A. Induction
B. Progression
C. Initiation
D. Promotion
E. Conversion
C
Which histologic subtype of BCC is considered most likely to recur?
A. Pigmented
B. Superficial
C. Nodular
D. Adenoid
E. Sclerosing
E

Complement receptor 1 ( CR1, CD35 ) is NOT expressed on one of the following :


A. Neutrophils
B. Macrophages
C. T cells
D. B cells
E. Follicular dendretic cells
E

One of the following cytokines is not produced by keratinocytes :


a. Il-1
b. TNF -beta
c. Il- 6
d. TNF ALFA
e. TGF –beta
B

ONE of the following IS NOT expressed by BASOPHILS :


A. Leukotriens
B. Il-4
C. Il-3
D. IgE
E. Prostaglandin
C

. Gap junctions in the epidermis are connections between adjacent cells through:
A. Claudine
B. Cadherins
C. Connexins
D. Plakoglobins
E. Plakophilins
C

One of the following is not correct about cyclosporin :


a. It binds to cyclophilin , that inhibits cytoplasmic calcineurin
b. Dephosphorylation prevents the normal translocation of NF-AT ( NUCLEAR FACTOR OF
ACTIVATED T- CELLS ) from cytoplasm to nucleus
c. Activate production of IL-2
d. NK cells and B- cells are affected by cyclosporine
e. The effects of Tacrolimas are similar to those of cyclosporin
C
Witch One of the following is negative in wood,s light :
a. Microsporumauduinii
b. MicrosporumGypseum
c. Trichophytonschoenleinii
d. Porphyreas
e. Trichophytontonsurans
E

One of the following is not correct about the side effect of amphotericin B :
a. Hypersensitivity reaction including anaphylaxis
b. Febrile reaction
c. Microchromic anemia
d. Decreased potassium and magnesium serum level
e. Renal dysfonction
C

Non inflammatory types of tineacapitis is caused by :


a. MicrosporumGypseum
b. TrichophytonViolaceum
c. TrichophytonMentagraphytes
d. MicrosporumCanis
e. Trichophytonrubrum
B

The venereal disease with shortest incubation period is :


a. Syphilis
b. Chlamydia urethritis
c. Pinta
d. Chancroid
e. Mycoplasmal infection
D

One of the following is alkylamine class of antihistamine:


a. Triprolidine HCL
b. Diphenhydramine HCL
c. Hydroxyzine HCL
d. Azatadine maleate
e. Promethazine HCL
A

Which one is not correct about wet ( rural ) type of cutaneous leishmania :
a. Incubation period ranges from few weeks to 2 months
b. Ulcerates rapidly
c. Self healing take 12 months
d. Appears as red furuncle like nodule
e. Ulcer with raised red margin is observed
C
Tin- tack sign is observed in :
a. SLE
b. DLE
c. Sarcoidosis
d. Psoriasis
e. Darier
B

Phage 2 type 71 staphylococcus aureus is not found in patient with :


a. Lichen plan
b. Psoriatic scales
c. Atopic dermatitis
d. Dermatomysitis
e. Hemodialysis
B

One of the following is not RNA containing:


a. Picorna v.
b. Retrovirus
c. Rhabdo v.
d. Paramyxo v.
e. Parvo v.
E

In which of the following disease Kaposi,s varicelliform eruption does not occur :
a. DLE
b. Severe seborrheic dermatitis
c. Scabies
d. Darier disease
e. Burns
A

Which one is not a manifestation of Epstein – Bar virus :


a. Gianoti – Crosti syndrome
b. Genital ulcers
c. The papularpurpuric gloves and socks syndrome
d. EM
e. Oral hairy leukoplakia
C

Which one is the major protein of tight junction :


a. Integrin
b. Laminin
c. Connexin
d. Claudin
e. Nidogen
D
The envelop of glycoprotein HIV virus is :
a. P24
b. Gp41
c. P17
d. Gp120
e. Reverse transcriptase
D

The most serious complication of SJS/TEN is related to :


a. Kidney
b. Liver
c. Spleen
d. Eye
e. Pancreas
D

Leptin is involved in:


a. Keratinisation
b. Corneodesmolysis
c. Energy homeostasis
d. Induction of hair telogen phase
e. Repair of dermoepidermaljonction
C

Large clear cells, small dark cells and myo-epithelial cells make WHICH of the following :
a. Peripherial cells of sebaceous gland
b. Main duct of sebaceous gland
c. Secretory coil of the eccrine sweat gland
d. Secretory coil of the apoccrine sweat gland
e. Duct of the eccrine sweat gland
D

At the surface of earth, the sense of wormth results from WHICH of the following :
A. 100-290 nm
B. 290-400 nm
C. 400-760 nm
D. 760-1440 nm
E. 1440 nm – 1 mm
E

Which cell is a major source of angiotensin converting enzyme?


a. Pericyte
b. Veil cell
c. Merkel,s cell
d. Endothelial cell
e. Smooth muscle cell
D

Prolyl hydroxylation reaction is the initial step in the biosynthesis in WHICH of the following :
a. Collagen
b. Elastin
c. Keratin sulphate
d. Hyaloronic acid
e. Keratin
A

A skin biobsy from a 35 year old patient shows suprabasalacantholytic cleft and negative
immunofluorescence. Which of following clinical features is most likely present in the patient ?
a. Tense bullae in the abdomen
b. Crusted erosions in axillae
c. Greasy papular lesions in chest
d. Non blanchable erythematous macules
e. Dermatitis reaction in the face due to aftershave lotion
B

Which of following cysts contains lobules of sebaceous glands?


a. Milium
b. Pilomatricoma
c. Epidermal cyst
d. Tricholemmal cyst
e. Steatocystoma
E

CD207 is a sensitive and specific marker of :


a. Helper T cell
b. Cytotoxic cell
c. Merkel cell
d. Langerhans cell
e. Vascular endothelial cell
D

Which one is a clinical presentation of spongiosis:


a. Erythema
b. Wheal
c. Oozing (weeping)
d. Lichenification
e. Lichenoid papule
B

.which one is accurately matched ?


a. karyorrhexis – fragmentation of cllagen fibers
b. Polymorphism – variability in the apperance of nuclei of the same cell type.
c. kamino bodies – eosinophilic rounded body resulting from degeneration and death of
keratinocytes
d. theque – collections of naevus cells at and in region of the dermoepidermal junction
e. Munro microabscesses – collection of neutrophil polymorphs in thesratumcorneum
E
Multiple epidermoid cysts, fibrous tissue tumors, osteomas and polyposis of the colon are
features of ?
a. Cowden disease
b. Gardner,s syndrome
c. Papillon – Lefevresyn
d. Cronkhite- Canada syndrome
e. Goltz syndrome
B

Which one is a feature of Netherton syndrome?


a. Distinctive face
b. Premature ageing
c. Trauma – induced blistering
d. Congenital erythroderma
e. Conical teeth
D

A biopsy from a lesion on the face of an adult patient shows dermal palissades of cells and
stroma. Which one is most likely to be present in the lesion ?
a. Positive diascopy
b. Ichtyosiform scale
c. Foamy histocytes
d. Positive congo red stains
e. Telangiectasia
A

Which disorder is transmitted through healthy female carriers?


a. Phenylketonuria
b. Xeroderma pigmentosum
c. Acrodermatitis enteropathica
d. Hereditary haemorrhagic telangiectasia
e. Fabry,s diseaes
E

A 20 y. patient, with genetic disease, with greyish small flaky scals, most distinct on the extensor
surfaces of the arms and lower legs, sparing the flexural creases. The lesion developed early in
childhood. Biopsy showes hyperkeratosis and absent granular layer. Which one is defected ?
a. Connexins
b. Keratin1 & 10
c. Cholesterol sulphate
d. transglutaminase
e. Filaggrin
E

A 10 y. child with yellowish plaque with velvety surface on scalp. Biopsy shows Mature
sebaceous glads, papillomatous hyperplasia of overlying epidermis, inconspicous hair follicles
and buds of undifferentiated epithelial cells. What is the appropriate management?
a. Ct scan brain
b. Removal of the lesion
c. Potent topical steroid
d. Topical nitrogen mustard
e. Reassurance
B
A 50 y. patient, with multiple nodular lesion on the face, trunc and proximal limbs , of long
duration,. Biopsy shows aggregates of epitheliod cells, mulinucleate cells and few surronding
lymphocytes. What is the next management step?
a. X-ray chest
b. Treat by potent topical steroid
c. CBC
d. Treat by topical nitrogen mustard
e. Confirm the diagnostic by detailed exam of urogenital tract.
A

A 4y. old child presents with a 3 cm red nodular lesion on the back. Biopsy shows spindle
shaped granulated cells staining with toluidine blue. Which one is the most expected course?
a. Dissemination to internal organs
b. Breaking down into an ulcero- vegetative lesion
c. Spreading to lymph nodes
d. Spontaneous resolution over childhood period
e. Diffuse cutaneous involvement
D

Lamellar granules are developed in :


a. Spinous layer
b. Granular layer
c. Stratum corneum
d. horny layer
e. Basal layer
B

Which one is significanty increased in DLE:


A. HLA- B7
B. HLA- B8
C. HLA- A2
D. HLA- Dr3
E. HLA- Dr4
E

Which one is resulting from disruption of desmoglein – 4:


a. naxos disease
b. autosomal recessive hypotrichosis
c. bullous impetigo
d. lethal acantholyticepidermolysisbullosa
e. striate palmoplantarkeratoderma
B

Which one has more expression of desmoglein –1:


a. Granular layer
b. Dermoepidermal junction
c. Hair follicles
d. Blood capillaries
e. Basal layer
A

When interpreting a biochemistry of a loricrin , you will find :


a. A cysteine- rich protein
b. A lysine- rich protein
c. An arginine- rich protein
d. A histidine- rich protein
e. A prolin- rich protein
A

The presence of a mixed population of cells bearing different genetic characteristic leading to
phenotypic diversity is referred to as:
A .balanced translocation
b.mosaicim
c.reciprocal translocation
d. Heteroplasty
e. uniparental meroisodisomy
B

Failure of phagolysosome formation in neutrophils is well documented in :


a. Skin fragility syndrome
b. POEMS syndrome
c. COSTELLO syndrome
d. Leopard syndrome
e. Chediak- higashi syndrome
E

Th 17 is distinguished by production of :
a. Tnf- alfa
b. Il- 17
c. Il – 4
d. Il- 23
e. Interferone – gamma
B

A napped claires is a well recognized feature of :


a. Parapsoriasis
b. PRP
c. Darier disease.
d. Lymphomatoid papulosis
e. Reiter syndrome.
B

A central parakeratosis without hypergranulation is a characteristic histopathologic feature of :


a. Lichen nitidus
b. Pityriasis lichenoide
c. GVHD
d. PRP
e. Parapsoriasis
B

Which one is a well recognizedTh 1- promoting factor :


a. IL-17
b. IL-23
c. IL-2
d. IL-12
e. IL-4
C

A neonate presented with disseminated infection. Which CSF Test is more sensitive to detect
herpes simplex meningitis ?
A. CSF protein analysis
b. HSV PCR
C. HSV culture
D. Tzanck smear
e. HSV IgG antibody
B

Lactoferrin is well recognized component in :


a. Keratinocyte
b. Melanocyte
c. Neutrophil
d. Fibroblast
e. T- cell lymphocyte
C

A 38 year man , HIV + , He asked about the chance of him progressing to symptomatic AIDS .
whitch test would be most usefull?
a. HIV antibody test
b. HIV RT PCR
c. Neopterin
d. HIV
e. CD4 Lymphocyte count
E

A 5 year BOY known to have sickle cell disease presented with an acute febrile rash and
transient aplastic crisis, which virus is well recognized to cause this presentation ?
a. Herpes simplex
b. Varicella- zoster
c. Parvovirus
d. Rubella
e. Rubeola
C
Which cells secret CYTOKINES that stimulate IgE production by B cells, recruit and activate
mast cells during induction phase of type 1 hypersensitivity reaction:
A. Melanocytes
B. NK cells
C. Macrophages
D. Neutrophils
E. CD4+ lymphocytes
E

Which one is true regarding IgM :


a. Cross the placenta
b. Produced in a secondary immune response
c. Found as a diametric form linked by a J chain
d. Can activate complement
e. Usually found on the surface of mast cells
D

Which virus is single stranded RNA :


a. Measles
b. HSV
c. Adenovirus
d. Parvovirus
e. HPV
A

Which CYTOKINE is not secreted by T-helper lymphocytes:


A. IL12
B. IL4
C. IL10
D. IL2
E. IFN- gamma
A

Which one of the following side effects is not caused by thalidomide:


a. Somnolence
b. Constipation
c. Motor neuropathy
d. Thromboembolism
e. Irregular heartbeat
C

In localized acute cutaneous LE, of following is characteristically spared :


a. forehead
b. nasolabial fold
c. chin
d. v- area of the neck
e. ears
B
Which hormone is increased in female patients of SLE :
a. Tesstosterone
b. DIHYDRO TESTOSTERONE
c. HIGH POTENCY ESTROGEN
d. DHEA
e. PROLACTIN
C

the carpet tack sign is a prominent feature in one of following skin disease :
a. Sarcoidosis
b. DLE
c. Follicular mucinosis
d. nodular amyloidosis
e. Ashy dermatosis
B

A 34 y old female patient presented with extensive LP and liver disease . Which one of the
following drug is recommended to treat this patient :
a. Prednisolone
b. Prednisone
c. Acitretine
d. Methotrexate
e. PUVA
A

. Which one of the following does not cause scleroderma – like disorder :
a. Bleomycine
b. Vinyl chloride
c. Silicosis
d. Tolerization
e. Fosinopril
D

Which one of the following is not true regarding Doxepin :


a. It is a tricyclic antidepressant
b. It is a potent H1 receptor - blocker
c. It is a potent H2 receptor - blocker
d. It is a mast cell stabilizer
e. It is a inhibits he reuptake of serotonine
D

Which one of the following drugs cause chronic myelogenous leukemia :


a. Imatinib
b. Interferon
c. Hydroxyurea
d. Busulfan
e. Dapsone
D
A 15 year old boy presented with dermatitis involving the whole face. His dermatologist gave
him betamethasone valerate cream twice daily for 1 week. How much grams he should apply for
each application:
a. 1 gram
b. 3 grams
c. 5 grams
d. 7 grams
B

Which one of the following does not cause immunologic granuloma:


a. beryllium
b. silica
c. zirconium
d. cadmium
e. mercury
E

. Which one of the following collagen types is increased in scleredema :


a. collagen type I
b. collagen type II
c. collagen type III
d. collagen type IV
e. collagen type XI
A

Which one of the following returns leukocytes and interstitial fluids :


a. arterioles
b. lymphatics
c. subcutaneous venous plexus
d. capillaries
e. arteriovenous anastomosis
B

Which one of the following binding protein is essential in the mechanism of action of Tacrolimus
:
a. Fibronectin- binding protein
b. Immunophilin- binding protein
c. Lipopolysaccharide- binding protein
d. Retinol - binding protein
e. Albumin
B

Which one of the following is CONCERNED with heat regulation :


a. arteries
b. veins
c. lymphatics
d. venous plexus
e. capillaries
D

......
3/17

6/17
One of the following is not associated with eosinophilia:?

A. Schistosomiasis
B. Filariasis
C. Amaebiasis
D. Hydatid disease
E. Helminthiasis

7.17
One of the following is NOT side effect of fumerate:
A. Renal impairement
B. Flushing
C. Headache
D. Fatigue
E. Hepatotoxicity
E

5/17
One of the following is derived from neural crest:
A. Keratinocytes
B. cells
C. Langerhans cells
D. Face dermis
E. Endothelial cells
D
Face and scalp dermis

4/17
Which one of the following is 3d line treatment of physical urticaria:

A. Chlorpheneramine
B. Desloratidine
C. IV. IG. infusion
D. Prednisone
E. Thyroxine
C

A pregnant woman has varicella 2 days before delivery, concerning the effects on the neonate ,
which of the following recommendation is most appropriate ?
A. No further recommendation at this time
B. The mother should be started on IV acyclovir as soon as possible
C. The neoborn should receive varicella Immunoglobolines as soon as possible
D. The neoborn should receive the varicella vaccin as soon as possible
E. The neoborn should receive IV acyclovir as soon as possible

.6/17
One of the following is not responsible for systemic corticosteroid induced osteoporosis:
A. SECONDARY hyperparathyroidism and bone resorption
B. Increased gastrointestinal absorption of calcium
C. Increased osteoclast activity
D. Decreased osteoclast activity
E. Increased renal excretion of calcium
‫ اكو خطأ‬s

6/17
One of the following is not sexually transmitted:

A. scabies
B. moll. contagiosum
C. condylomata accuminata
D. eczema herpeticum
E. herpes progenitalis
D

6-17
One of the following is not true about treponema pallidum:

A. Generally shows 8 – 12 spiral convulsions


B. Stained by silver stain
C. Septrin sensitive
D. Protected from phagocytosis by polysaccharide capsule
E. It does not produce exotoxin
D

7=17
Which of the following is not a major feature of Tuberus Sclerosis ?

A. Gingival fibroma
B. Periungual fibroma
C. Hypomelanotic macules
D. Facial angiofibroma
E. Lymphangioleiomyomatosis
A

7=17
A 47-y-old female developed an acneiform facial eruption at the time of elective ambulatory
surgery when she was treated with pre and post-operative antibiotics. Occasionally, the patient
will develop painfull nodules as well. Topical tretinoin cream and doxycycline have not been
effective. Culture of a pustule reveals growth of serratia marcescens. What is the best next step
in the management of the patient?
A. Skin biopsy with tissue culture, as serratia marcescens is normal Skin flora
B. Switch to minocycline
C. Add benzoyl peroxide as it possesses antibacterial properities
D. Switch to metronidazole gel
E. Obtain baseline lab work, including a beta-HCG
C,!! E

7/17
Hair shaft production rate (scalp):
A. ~0.6 mm/day, 0.5cm/month
B. ~0.5 mm/day, 1.5cm/month
C. ~0.7 mm/day, 2cm/month
D. ~0.8 mm/day, 2.5cm/month
E. ~0.35 mm/day, 1cm/month
E

7/17
Which of the following has the highest risk for gastrointestinal bleeding:
A. Aspirin
B. Ibuprofen
C. Diclofenac
D. Piroxicam
E. Indomethacin
E

8/17
In atopic dermatitis :
A. White dermographism is blanching of the skin at the site of stroking with a blunt instrument
Which is similar to the triple of Lewis
B. 50%-70% of chronic eczematous lesions contain staphylococcus aureus in large numbers
C. Acute atopic dermatitis is a Th-1 response which turns to Th-2 in its chronic phase
D. Keratoconus is uncommon finding occurring in 1% of Atopic individuals
E. Vaccination against smallpox can be given savely when the dermatitis in remission.
D
7 17
One of the following is mesodermal lineage:
A. Keratinocytes
B. Melanocytes
C. Merckel cells
D. Neurons
E. Langerhans cells
E

Fluconazole is not FDA approved in:


A. Vaginal candidiasis
B. Cryptococcus meningitis
C. Oropharyngeal and esopharyngeal candidiasis
D. Cutaneous and chronic mucocutaneous andidiasis
E. Prophylaxis against candidiasis in bone marrow transplant
D
7 17
Brown fat differs from white fat by being:
A. Unilocular
B. Metabolically less active
C. Its cells contain less mitochondria
D. Most prominent retroperitoneally
E. Responds to oral noradrenaline
D

12 17
Which of the following is not correct about itraconazole?
A. Highly lipophilic
B. Cimetidine decreased serum level of itraconazole
C. It persists in nails up to 7 – 12 months
D. The drug must be taken with fatty food
E. It is caregory CI in pregnancy
D

Select the incorrect statement about the remoudeling of keratinocytes into corneal layer:
A. Keratinocytes become less permeable to Ca
B. Profilagrin is processed into filagrin
C. Keratohyaline granules are broken down
D. Glutaminazes are activated
E. Filagrin is degraded into amino acids
1

Makki15+16
The following are contraindications for topical tazarotene in psoriasis Except:
A. Gulfate psoriasis
B. Unstable plaque psoriasis in a phase of progression
C. Erythrodermic psoriasis
D. Allergic contact dermatitis to tazarotene
E. Pregnancy and lactation

One of the followings is not correct about ivermectin:


A. It is a macrocyclic lactone derived from ivermectin B1
B. It blocks the transmission of signals from interneurones to excitatory sensory neurons
C. It is effective against sarcoptes sabiei
D. It is used for treatment larva migrans
E. Side effects include fever, itching and headache
B

One of the following is not a function of IL-1:


A. Suppression of migration and activation of Langerhans cells.
B. T cell proliferation.
C. Increase expressions of adhesion molecules on endothelium.
D. Fever.
E. Induction of chemotaxis and cytotoxicity of monocytes.
A
Which of the following retinoids has the shortest elimination half-life:
A. Etretinate.
B. Bexarotene.
C. Acitretin.
D. Isotretinoin.
E. Tretinoin
5
. Which of the following does not cause false positive patch test:
A. Irritant vehicule
B. Insufficient amount of the test substance is applied
C. Recent dermatitis at the site
D. Adhesive tape reaction
E. Stains or discoloration of the skin
B
Interface dermatitis not seen in which of The following
A. EM
B. Large plaque parapsoriasis
C. PR
D. Paraneoplasic pemphigus
E. Lichen sclerosus
C

Which one of the following does not cause immunologic granuloma:


a. berylium
b. silica
c. zirconium
d. cadmium
e. mercury
E

Granulomas as clinical features suggesting an irritant or toxic etiology may be caused by the
followings EXCEPT:
A. Cotton fibers
B. Talc
C. Beryllium
D. Silica
E. Woods
E

. Pyogenic granuloma: which of the following statements is FALSE?

A. Commonly occurs in the gingival, lips or fingers.


B. Usually develops at the site of preexisting injury.
C. May appear within a preexisting nevus flammeus.
D. With multiple lesions has a predilection for the interscapular region.
E. Drug-indeced pyogenic granuloma tend to persist after withdrawal of the causative drug.
E

A 4 year old child presents with a 3 cm red nodular lesion on the back. Biopsy shows spindle
shaped granulated cells staining with toluidine blue. Which one is the most expected course?
A. Dissemination to internal organs
B. Breaking down into an ulcero- vegetative lesion
C. Spreading to lymph nodes
D. Spontaneous resolution over childhood period
E. Diffuse cutaneous involvement
D
Dx: Solitary mastocytoma
Which of the following suture type has the most persistent tensile strength over time?
a. Braided polyester
b. Polygdioxanone
c. Polyglactin 910
d. Poliglecaprone
e. Silk
A

A napped claires is a well recognized feature of :


a. Parapsoriasis
b. PRP
c. Darier disease
d. Lymphomatoid papulosis
e. Reiter syndrome
B

Which one of the following about genetics is not true.


a. nuclear DNA is packed into chromosomes
b. somaitic and germline cells contain approximately 4 m of DNA
c. the maternal and paternal gamates are haploid cells
d. X and Y chromosomes share only two small regions (pseudoautosomal regions)
e. Karyotype describes the constitution of an individual
B

Which of the following is false about head louse:


A. It is highly host specific insect.
B. Its size 2-3mm.
C. Male louse lives for 30 days.
D. Eggs located within 0.6 mm of the scalp are unhatched.
E. Rarely live more than 36 hours away from the host.
C

About chrome gut suture, all are true except:


A. It is the first synthetic absorbable suture
B. It is processed by chromium salts to increase its resistance to enzymatic degeneration.
C. It is tensile strenght lasts for 10-14days.
D. It is still widely used for suturing of mucosal surfaces.
E. None of the above.
A

Which of the following dressings is the least to absorb wound fluid:


A. Foams.
B. Hydrogels.
C. Films
D. Alginates.
E. Hydrocolloides
C

- [x] Concerning UV light, which statement is not true:


A. Wave lengh of 360 nm is 1000-fold less erythemogenic than 300nm.
B. An immediate erythema reaction is rarely seen i UVB –induced sunburn.
C. DNA is hypothesized to be chromophore for UVB erythema.
D. Delayed tanning is usually as result of exposure to UVA.
E. Fair skin usually tans only with UVB doses above the erythema threshold.
D

- [x] Which of these molecules plays a major role in adhesion of basal cell extracellular matrix:
A. Fibronectin
B. Perlecan
C. Envoplakin
D. Desmoglein
E. Heparin
A

- [x] - Dermal dendrocyte hamartoma shows positive immunohistochemical staining of which of


the following?

A. HLA-DR
B. CD1a
C. CD34
D. S100 protein
E. Factor IX
C

- [x] Stem cell can be identified by high expression of:


A. Langerin.
B. Β1-integrins.
C. Desmoglein 1.
D. Factor XIIIa.
E. Keratin 14.
B

The major component of basal lamina is:


A. Collagen IV
B. BP230.
C. Α6β4 integrin.
D. Collagen VII.
E. Plectin.
A

. A 15 year old boy presented with dermatitis involving the whole face. His dermatologist gave
him betamethasone valerate cream twice daily for 1 week. How much grams he should apply for
each application:
a. 1 gram
b. 3 grams
c. 5 grams
d. 7 grams
e. 10 grams
B

The average daily requirement of vitamin C is:

A. 5 mg
B. 10 mg
C. 20 mg
D. 40 mg
E. 50 mg
E

. Which of the following is not high – potency topical steroids:

A. Betamethasone dipropionate ointment 0.05 mg


B. Flucinomide gel 0.05%
C. Dexamethasone spray 0.2%
D. Flucinolone acetonide cream 0.2%
E. Clobetasole ointment
C

Which of the following is not a sebaceous gland:


A. Ceruminous gland.
B. Meibomian glands.
C. Montgomery tubercles.
D. Tyson glands
E. Fordyce spots.

. One of the following drugs does not precipitate Erythema multiforme primarily:
A. B-blockers
B. Non-steroidal anti-inflammatory drugs
C. Sulfonamides
D. Antibiotics
E. Anti-epileptics
1

: Melanocyte develops at:


A. 4 weeks of EGA.
B. 12 weeks of EGA.
C. 16 weeks of EGA.
D. 20 weeks of EGA.
E. 24 weeks of EGA.
B

. Which one of the following laboratory tests is not used for diagnosis of primary syphilis :

A. Dark field microscopic examination


B. VDRL
C. TPHA
D. FTA – ABS 19S-IgM
E. FTA – ABS
2

Transglutaminase enzymes are involved in the formation of Which of the following ?


A. Sebum
B. Keratin 1&10
C. Oxytalan fibers
D. Cornified cell envelop
E. Intercellular lipid in the stratum corneum
D

. A 10 y. child with yellowish plaque with velvety surface on scalp. Biopsy shows Mature
sebaceous glads, papillomatous hyperplasia of overlying epidermis, inconspicous hair follicles
and buds of undifferentiated epithelial cells. What is the appropriate management?
a. Ct scan brain
b. Removal of the lesion
c. Potent topical steroid
d. Topical nitrogen mustard
e. Reassurance
B

Keratohyaline granules:
A. Are acidic in nature
B. Composed primarily of keratin
C. Release filagrin
D. Filagrin is degraded to acid metabolites in the granular layer
E. metabolites of filagrin help filter UV radiation
5

Makki 15
. Concerning wound healing, which of the following statement is not correct:
A. If the injury is limited to the epidermis it restores itself to a structure similar to the preinjury
state.
B. In case of injury to the dermis, regeneration does not occur normally.
C. Partial thickness wound involves the epidermis and part of the epidermis.
D. In full thickness wounds, the adnexal structure serves as reservior of epithelial cells for
repopulating the epidermis.
E. In full thickness wound, epithelium can only migrate the ulcer edge.
4
Tissue contraction begins:
1. At 3rd day of wound healing
2. During the 2nd week of wound healing
3. After the first month of wound healing
4. After the 3rd month of wound healing
5. After the 9th month of wound healing
2

. about scabies , Which one of the following is not correct:


a. it is caused by highly host - specific eight- legged mite
b. mites from animals are not a source of human infestations
c. it is too small to be seen by the naked eye
d. a female mite will lay 6 – 10 eggs
e. eggs require 10 days to mature
D

- one of the following antimicrobial drugs is bacteriostatic:


A. tetracyclines
B. penicillins
C. aminoglycosides
D. quinolones
E. vancomycin
A
In which of the following antibiotic, the site of action is DNA gyrase :
A. Vancomycin
B. Clindamycin
C. Sulfonamides
D. Quinolones
E. Trimethopri
D
Which of the following is bacteriostatic:
A. Aminoglycosides.
B. Penicillins.
C. Sulphonamides.
D. Quinolones .
E. Vancomycin.
C
.
The following mucosal manifestation are known to occur in sarcoidosis except?
A. Xerostomia
B. Salivary gland swelling
C. Gingival swelling
D. Leukoplakia
E. Swelling of the lachrymal and parotid glands and facial palsy
D
One the followings is not cutaneous manifestations of Behcet’s disease:
A. Sweet’s syndrome-like dermatosis
B. Erythema nodosum
C. Neutrophilic lobular panniculitis
D. Pyoderma gangrenosum
E. Pyoderma vegetans.
E

. Which one of the following collagen types is increased in scleredema : :


a. collagen type I
b. collagen type II
c. collagen type III
d. collagen type IV
e. collagen type XI
C

. Which of the following TLR is involved in recognizing lipopolysaccharides:

A. TLR 2
B. TLR 3
C. TLR 4
D. TLR 5
E. TLR 7
C

Which of the following TLR is ACTIVATED by triacylated lipoprotein :


A. TLR 2/6
B. TLR 2/1
C. TLR 3
D. TLR 9
E. TLR 8
B

Toll-like receptor (TLR) 6 is:


F. Diacylated lipoprotein
G. Flagellin.
H. Lipopolysaccharide.
I. Ds RNA.
J. Triacylated lipoprotein.

Which of the following TLR is involved in recognizing lipopolysaccharides:


F. TLR 2
G. TLR 3
H. TLR 4
I. TLR 5
J. TLR 7
3

Which of the following Toll-like receptors is involved in recognizing flagellin:


A. TLR-2.
B. TLR-3.
C. TLR-4.
D. TLR-5.
E. TLR-6.
4

Stem cells have high activity in one of the following conditions:


A. During fetal development and wound healing
B. Cycling in normal epidermis
C. Growth in culture
D. Potential for differentiation
E. Self- renewal
A

.Which of the following mycobacteria is a rapid grower in culture?


A. M. lepra.
B. M. fortuitum
C. M. kanasaii.
D. M. ulcerans.
E. M. tubruculosis.
B
. Which of the following mycobacteria is rapid grower in culture?

A. Mycobacterium leprae
B. Mycobacterium kansasi
C. Mycobacterium smegmatis
D. Mycobacterium scofulaceum
E. Mycobacterium tuberculosis
C

A neonate presented with disseminated infection. Which CSF test is more sensitive to detect
herpes simplex meningitis ?
A. CSF Protein Analysis
b. HSV PCR
C. HSV Culture
D. Tzanck smear
e. HSV IgG antibody
B

. In the 1st year of life, one of the following disease is difficult for the immune system to
recognize:

A. Neisseria meningitis
B. Chicken pox
C. HS
D. Impetigo contagiosum
E. Bacterial folliculitis
A
Which of the following is not a feature of Nisseria gonorrhoae:
A. Is a Gram negative bacteria.
B. Grows in hemoglobin – containing medium.
C. It needs 3-5% CO2.
D. It has polysaccharide capsule.
E. It canot tolerate dryness and low temperature.
D

Histamine exerts its effect during inflammation by:


A. Vasoconstriction of post Capillary sphincters.
B. Constriction of Large Arteries.
C. Acting on H2 Receptors on Mast Cells.
D. Causing Venular Endothelial Contraction.
E. Its Direct Effect on Macrophages.
A

Which of the following facts about histamine is true:

A. Histamine is a potent activator of mast cells.


B. The major source of histamine in the skin are keratinocytes
C. There are currently 6 known histamine receptors
D. The H2 receptors is the receptor mainly responsible for pruritus
E. In addition to H1, H3 and H4 receptors can also modulate pruritus
5

Cells that have eosiniphilic cytoplasm and a pyknotic nucleus represent:


A. Parakeratosis.
B. Acanthosis.
C. Hyperkeratosis.
D. Dyskeratosis.
E. Spongiosis.
D
Which of the following is not advantage of foam occlusive dressing:
A. Absorbant.
B. Creates bacterial barrier.
C. Permeable to water vapor.
D. Conforms to wound shape.
E. Comfortable.
B

The best dressing type for achieving mild hemostasis is:

A. Alginates
B. Collagens
C. Petroleum gauze
D. Hydrocolloids
E. Foams
A
Which of the following dressings is the least to absorb wound fluid:
A. Foams.
B. Hydrogels.
C. Films.
D. Alginates.
E. Hydrocolloides.
C

Which of the following diseases is transmitted by mosquitos:

A. Trypanosomiasis.
B. Bilharziasis.
C. Filariasis
D. Schistosomiasis.
E. Cysticercosis.
C

Which of the following is not a tick-born disease:


A. Lyme disease.
B. Rocky mountain spotted fever.
C. Babesiosis.
D. Human monocyticehrlichiosis.
E. Yellow fever.
E

In vagabond’s disease, There is a heavy infection with what:


A. Mite
B. Lice
C. Tick
D. Parapoxvirus
E. Trichophyton violaceum
B

Each of the following is true about melanosomes except: 1. Most characteristic organelle of the
melanocyte
2. Tyrosinase activity decreases as melanosomoes mature
3. Are transferred to keratinocytes via phagocytosis
4. Are singly dispersed in the basal layer in white skin
5. Are larger in size in black skin compared to white skin
4

Which of the following is FALSE about melanin synthesis:


A. Pheomelanin has oval – shaped structure
B. Melanin precursors are dependant on copper-requiring enzyme
C. Eumelanin is brown or black in color
D. Tyrosinase is the rate-limiting step in melanin synthesis
E. Chronic sun exposure causes larger melanosomes to be created
A
Which of the following is not true:
A. Melanocytes are derived from neural crest.
B. The different stages of Melanosomes correlate with the degree of Melanization.
C. Melanosomes that are involved in brown Melanin synthesis are spherical
D. Keratinocytes produce GF which are mitogenic to Melanocytes
E. Proliferation of Melanocytes rely on vit. D synthesis within the Epidermis.
C

The major difference between darker skin and lighter skin is:
A. The relative concentration of melanocytes per unit area of skin surface.
B. The relative activity of cutaneousmelanocytes in creation of melanosomes
C. The relative action of cutaneousmelanocytes in transferring melanosomes to keratinocytesx
D. The fate of the melanosomes once they are within the keratinocyte.
E. Number of melanosomes is higher in lighter skin.
B

Choose the correct answer regarding melanin and skin color:


1. In black and brown skin the melanosomes are smaller in diameter and length
2. Facultative skin color is the amount of cutaneous melanin pigment generated according to
cellular genetics
3. In white skin the melanosomes form groups within the secondary lysosomes
4. Eumelanin produces a yellow chromophore
5. The number of melanocytes increases with one exposure to UVA/visible light
3

One of the following is NOT correct about melanocytes:


A. In premature there is high number of cells.
B. In newborn similar number of cells to young adults.
C. In adult their number decrease with age.
D. In premature high number of mature melanosomes.
E. In adult, melanin production dependent on skintype and body area.
D

the major difference between darker skin and lighter skin is:
• number of melanosomes is higher in lighter skin
• the fate of the melanosomes once they are within the keratinocytes
• the relative action of cutaneous melanocytes in transforming melanosomes to keratinocytes
• the relative activity of cutaneous melanocytes in creation of melanosomes
• the relative concentration of melanocytes per unit area of skin surface
4

One of the following is not correct about Melanocytes:


A. A key protein involved in melanosome assembly is NCKX5 encoded by the gene SLC 24A5.
B. Chediak- Higashi syndrome is due to reduced melanocytes in skin.
C. They are developed from melanoblasts.
D. Hair graying is due to the elimination of melanoblasts and melanocytes stem cells.
E. Melanocytes possess G-Protein-couple receptors and receptor tyrosine kinases.
B
Which of the following statements about darkly pigmented races versus lighter pigmented races
is correct?
1. The number of melanosomes in melanocytes are the same
2. The individual melanosomes have the same degree of melanization
3. The melanosomes are equal in size melanosomes
4. There are equal numbers of melanocytes
5. There is a faster rate of melanosome degradation
4

Regarding development of melanocytes, Which of the following is not true:


A. They are arising from neural crest
B. They are present in the epidermis by the end of 1st trimester
C. Melanin production does not begin until 3-4 months of EGA
D. Melanosome transfer to keratinocyte is not seen until 5 months of EGA
E. At birth, neonates are not fully pigmented
B

50 day
Stage II melanosome includes:
A. Spherical with no melanin deposition.
B. Oval with minimal melanin deposition and high tyrosinase activity.
C. Oval with high melanin deposition and high tyrosinase activity.
D. Oval with moderate melanin deposition and high tyrosinase activity.
E. Oval with high melanin deposition and minimal tyrosinase activity.
B

Raynaud Disease Characteristically:


A. Occurs Almost Entirely In Young Male Cigarette Smokers.
B. Is Associated With Atherosclerosis.
C. Occurs Primarily In Tropical Climates.
D. Is The Result Of A vasospastic Reaction.
E. Involves Elastic Arteries.
D

.‫ هذا السؤال ورد في دورة سابقة بصيغة مختلفة‬:- Which of the following is true about gap junctions:

A. They are composed of 6 connexins


B. They mediate tight adhesion between cells
C. They mediate calcium-induced calcium release
D. They require several heterotypic connexin subunits for proper function
E. All of the above
A

Calcium absprption:
A. Is not affected by age
B. Is stimulated by hydrochloric acid
C. Occurs in distal ileum
D. Is increased by phosphate in small intestine
E. Is delayed by bile
B

Regarding calcium homeostasis:


A. Plasma Calsium Level is 5 Mmoles Per Liter.
B. Calsium is mainly transported bound to albumin.
C. Calsium requires vitamin C for its absorption.
D. 50% of Calsium in plasma is bound to phosphates.
E. Vegetables contain considerable amounts of calcium.
B

Profilagrin:
A. Is a large calcium binding Glycoproteins
B. Its gene has been mapped 1q21
C. Consists of 15 – 20 repeating copies of filagrin
D. Its terminal domain contains calcium binding region used to assembly the molecule
E. Undergoes hydrolysis to release hydrogen
B

One of the following is not correct about diaper dermatitis:


A. Alkaline PH of urine
B. Enzymes produced by bacteria
C. Residual pancreatic and lipase in the stool
D. Diaper dermatitis is more likely to appear in breast –fed than in cow milk-fed infants
E. Chemical constituents of the diaper
D

Cyclobutane pyrimidine dimers are produced by which of the following processes?


A. Normal flora interaction with corneocytes.
B. Ultraviolet light interaction with the skin.
C. Breaking down of corneodesmosomes.
D. Antigen presentation to lymphocytes.
E. Wound healing.
B

Following UV irradiation, the most prevalent DNA photoproduct is:


A. Cyclobutane pyrimidine dimer
B. Adenine dehyrodimer.
C. Thymine-adenine dimer.
D. Cytosine photohydrate.
E. Pyrimidine 6,4 pyrimidone.
A

Which one of the following accelerates nail growth:


A. Night.
B. Old age.
C. Summer.
D. Fever.
E. Ca, Vit D.
C

Which of the following STATMENT about nails is correct?


A. Fingernails grow at about 0.5mm/month
B. Toenail grow faster than fingernails
C. After avulsion toenails return to former length in 6 months
D. Nails grow faster in winter than summer
E. Nails growth is slower in adults than in children
E

Which of the following neonatal conditions does not require treatment:


A. Milia.
B. Mongolian spots.
C. Umbilical granuloma.
D. Subconjunctival haemorrhage.
E. Erythema toxicum.
B

Which of the following topical drugs is LEAST likely to cause contact sensitization ?

A. Neomycin
B. Silver sulphadiazine
C. Diphenhydramine
D. DNCB
E. Diphencyprone
C

The highest incidence of drug reaction is with:


A. Penicillin G
B. Barbiturates
C. Cephalosporins
D. Ampicillin
E. Packed red blood cells
D

Stem cell can be identified by high expression of:


A. Langerin.
B. Β1-integrins
C. Desmoglein 1.
D. Factor XIIIa.
E. Keratin 14.
B

One of the following is not correct about stem cell:


A. Self renewal is unlimited.
B. Potential for differentiation is multipotent.
C. Cycling in normal epidermis is slow.
D. Proliferative potential is high.
E. Growth n culture is small abortive clone.
E

also

Full term healthy baby, at which age will he/she develop full/mature skin barrier?
a. At birth.
b. 3 days.
c. 3 weeks.
d. 6 week
C

The major reservior of Leishmania major is:


A. Human.
B. Dogs.
C. Foxes.
D. Rodents.
E. Hyraxes.
D

. One of the followings is not correct regarding treatment of Hidradenitis suppurativa:


A. Systemic steroids often leads to dramatic improvement
B. CO2 laser has been proven to be quite effective
C. Isotritinoin has been particularly effective
D. Infliximab may be used
E. Botulium toxin A to reduce sweating

The lipids in Odlands body are secreted in the form of :


A. Cholesterol
B. Ceremides
C. Sphingolipids
D. Glycosylated lipids
E. Triglycerides
D

- The proliferation of T cells after activation is primarily controlled by:

A. IL-2 receptor
B. IL-12
C. IL-4
D. IL-4 receptor
E. INF gama
B

WHICH of the following has been shown to be the most efficacious for Lichen spinolosus ?

A. Topical corticosteroid.
B. Topical retinoid
C. Calcipotriene
D. 12% lactic acid
E. Correct : D radiation bactroban ointment
D

Fluconazole differs from other azoles by:

A. High molecular weight.


B. Low water solubility.
C. Not affected by food or gastric acidity.
D. Poor penetration to CSF.
E. Metabolized in the liver.
C

WHICH CELLS SECRET CYTOKINES THAT STIMULATE IGE PRODUCTION BY B CELLS,


RECRUIT AND ACTIVATE MAST CELLS DURING INDUCTION PHASE OF TYPE 1
HYPERSENSITIVITY REACTION :
A. MELANOCYTES
B. NK CELLS
C. MACROPHAGES
D. NEUTROPHILS
E. CD4+ LYMPHOCYTES
E

One of the following nail anomalies increases significantly after renal transplantation:

A. Absence of lunula
B. Splinter hemorrhage
C. Half-and-half nails
D. Leukonychia
E. Onychomycosis
D

Which one is significanty increased in DLE:


A. HLA- B7
B. HLA- B8
C. HLA- A2
D. HLA- Dr3
E. HLA- Dr4
E

A gardener presented with nonimmunologic contact urticaria. Which one of the following does
not cause nonimmunologic contact urticaria :
a. Benoic acid
b. cinnamic aldehyde
c. nicotinic acid esrers
d. natural rubber latex
e. sorbic acid
D

One of the following drugs is not reported to exacerbate psoriasis:


A. Antimalarials
B. B-blockers
C. Carbamazepine
D. Ibuprofen
E. Cimetidine

Dr.Sabreen O. Jebur
Dream,smile,live
2019

You might also like